Wikipedia:Reference desk/Science

From Wikipedia, the free encyclopedia

This is an old revision of this page, as edited by Freshgavin (talk | contribs) at 09:59, 19 October 2006 (→‎Male marsupial nipples ?). The present address (URL) is a permanent link to this revision, which may differ significantly from the current revision.


Science Mathematics Computing/IT Humanities
Language Entertainment Miscellaneous Archives
How to ask a question
  • Search first. It's quicker, because you can find the answer in our online encyclopedia instead of waiting for a volunteer to respond. Search Wikipedia using the searchbox. A web search could help too. Common questions about Wikipedia itself, such as how to cite Wikipedia and who owns Wikipedia, are answered in Wikipedia:FAQ.
  • Sign your question. Type ~~~~ at its end.
  • Be specific. Explain your question in detail if necessary, addressing exactly what you'd like answered. For information that changes from country to country (or from state to state), such as legal, fiscal or institutional matters, please specify the jurisdiction you're interested in.
  • Include both a title and a question. The title (top box) should specify the topic of your question. The complete details should be in the bottom box.
  • Do your own homework. If you need help with a specific part or concept of your homework, feel free to ask, but please don't post entire homework questions and expect us to give you the answers.
  • Be patient. Questions are answered by other users, and a user who can answer may not be reading the page immediately. A complete answer to your question may be developed over a period of up to seven days.
  • Do not include your e-mail address. Questions aren't normally answered by e-mail. Be aware that the content on Wikipedia is extensively copied to many websites; making your e-mail address public here may make it very public throughout the Internet.
  • Edit your question for more discussion. Click the [edit] link on right side of its header line. Please do not start multiple sections about the same topic.
  • Archived questions If you cannot find your question on the reference desks, please see the Archives.
  • Unanswered questions If you find that your question has been archived before being answered, you may copy your question from the Archives into a new section on the reference desk.
  • Do not request medical or legal advice.
    Ask a doctor or lawyer instead.
After reading the above, you may
ask a new question by clicking here.

Your question will be added at the bottom of the page.
How to answer a question
  • Be thorough. Please provide as much of the answer as you are able to.
  • Be concise, not terse. Please write in a clear and easily understood manner. Keep your answer within the scope of the question as stated.
  • Link to articles which may have further information relevant to the question.
  • Be polite to users, especially ones new to Wikipedia. A little fun is fine, but don't be rude.
  • The reference desk is not a soapbox. Please avoid debating about politics, religion, or other sensitive issues.


October 9

Car, chain

how do you pull a car with a chain and one person

Im not trying to block you, but Im sure that you can tackle this question on your own--Light current 01:33, 9 October 2006 (UTC)[reply]
This guy's trying to subtly point out the magicalness that is the block and tackle. Vitriol 01:44, 9 October 2006 (UTC)[reply]
"Magicalness"... that's interesting :). --liquidGhoul 02:01, 9 October 2006 (UTC)[reply]
Is the person who pulls the car the puller or the pulley ? StuRat 03:10, 9 October 2006 (UTC)[reply]
Ur pulling our legs!--Light current 03:11, 9 October 2006 (UTC)[reply]
If I'm pulling your legs may I be put on the rack, then drawn and quartered. :-) StuRat 00:59, 11 October 2006 (UTC)[reply]
You're making this way too complicated. Get a really big person. Clarityfiend 05:46, 9 October 2006 (UTC)[reply]
Or a really small car. DirkvdM 08:59, 9 October 2006 (UTC)[reply]
Kidding aside, what is the chain for? Or the tackle and stuff? Just push. I've done that (even slightly uphill once) and I'm not even particularly strong (except for my legs, I must admit). DirkvdM 09:03, 9 October 2006 (UTC)[reply]
You just have to remember to let the hand brake off. Richard B 21:28, 9 October 2006 (UTC)[reply]
The car must be in Neutral and the emergency brake released, with a licensed driver at the wheel (so the car does not get away from you and crash into something, or run over you if you are running alongside trying to push and steer). If it has power steering, the steering effort will be much greater than normal if the engine is off. Edison 14:26, 10 October 2006 (UTC)[reply]

I was wondering if you could help me find a source for a piece of information that could go in the article; I don't know if it goes here but their doesn't seem to be a help section for Wikipedia itself. On the talk page it has a section called 'The antidote?'. I don't know a good way to sum it up, so go and read it. The point is, the piece of information removed is true; I saw it on The Life of Mammals. The animals congegrate at a natural patch of a type of clay that neutralises poison in plants they eat. Although this is a fact, my Googling skills are poor so I don't know a reputable webpage I could cite. Any help? (P.S., the section on if they're kosher or not is bizarre and I don't know if it should be in.) Vitriol 01:21, 9 October 2006 (UTC)[reply]

The BBc's own website http://www.bbc.co.uk/nature/animals/features/274feature1.shtml is much more guarded on the reasons for them eating the clay so maybe it is uncitable as a fact. Curse that evil Attenborough. MeltBanana 01:48, 9 October 2006 (UTC)[reply]

Size of Human Epidermis Cell

Around how many micrometers in length is a human epidermis cell? Thanks. --Proficient 01:33, 9 October 2006 (UTC)[reply]

Which bit? Try here. Vitriol 01:42, 9 October 2006 (UTC)[reply]
I'm not sure actually. Whichever would come off if you scraped off a sample and put it under a microscope... --Proficient 05:02, 9 October 2006 (UTC)[reply]
I have another question: What color are the organelles of cells, specifically eukaryotic cells? (I know chloroplats are green, but what about ribosomes, mitochondria, and such?) I would assume that it might depend on the specific type of cell, but I want to make sure. When looking at them in microscopes, it would also depend on the dye, I believe. Whenever I see diagrams, they tend to be different from one another. So what color are they if they were able to be seen clearly without dye? This is a weird question, but thanks if you can answer it. --Proficient 05:08, 9 October 2006 (UTC)[reply]

why cant saltwater fish live in freshwater?

why cant saltwater fish live in freshwater? answer this in referance to teh cell function and stucture— Preceding unsigned comment added by 68.19.209.91 (talkcontribs)

Get up now, get on up, and do your own homework! Vitriol 01:59, 9 October 2006 (UTC)[reply]
I believe they would get bloated. --Proficient 05:15, 9 October 2006 (UTC)[reply]
When fat people swim in the sea, do they shrink? DirkvdM 09:06, 9 October 2006 (UTC) ps, if so, please don't tell them - the sea level is rising enough as it is. DirkvdM 09:08, 9 October 2006 (UTC)[reply]
If they do it for a couple of hours every day, and follow a sensible diet, they should look buff in a few months. Like a Diana Nyad.Edison 18:56, 9 October 2006 (UTC)[reply]
Isn't it freshwater fish suffocate in salt water and saltwater fish drown in fresh water? Vespine 22:47, 16 October 2006 (UTC)[reply]
I don't know if this is the reason they will die but it's probably what they are are looking fore. Has to do with partial pressure of salt in the cells and the semipermeable membrane of the cell. There is a large diffusion gradient between the salty cell and the fresh water. The cell will start to aborb the fresh water to equal out the pressure and balance the "saltiness". This will bloat the cell as it fills with water. The real question is how do fresh water fish survive in fresh water since they have the same saltiness in their cells as the salt water fish. Somehow they adapted but I don't know how. --Tbeatty 02:24, 19 October 2006 (UTC)[reply]

Mucopolysaccharidosis

mucopolysaccharidosis is a genetic disorder in which the person is lacking an enzyme normally found on the lysosomes. plants dont seem to suffer from the disorder. why dont plants have the disorder and what are the results of the disorder on the cell function?— Preceding unsigned comment added by 68.19.209.91 (talkcontribs)

As for plants, diseases rarely affect both people and plants. As for cells, see the first paragraph of Mucopolysaccharidosis where it says "People with a mucopolysaccharidosis either do not produce enough of one of the 11 enzymes required to break down these sugar chains into proteins and simpler molecules or they produce enzymes that do not work properly. Over time, these glycosaminoglycans collect in the cells, blood and connective tissues. The result is permanent, progressive cellular damage that affects the individual's appearance, physical abilities, organ and system functioning, and, in most cases, mental development." Dar-Ape 02:02, 9 October 2006 (UTC)[reply]

thanx

Plants don't break down sugar, they make it by photosynthesis? -THB 10:12, 16 October 2006 (UTC)[reply]

HOMEWORK QUESTION

the nucleus of a eukaryote is often referred to as the control centerof the cell. eukaryotes have a true membrane bound nucleus but prokaryotes just have a nuclear region. what is the advantage to having your dna isolated form the rest of ur cell?— Preceding unsigned comment added by 68.19.209.91 (talkcontribs)

See Vitriol's comment above. -Fsotrain09 02:08, 9 October 2006 (UTC)[reply]
Did you read the top of this page? "Do your own homework. If you need help with a specific part or concept of your homework, feel free to ask, but please do not post entire homework questions and expect us to give you the answers." Please see Cell nucleus. Dar-Ape 02:10, 9 October 2006 (UTC)[reply]

Gene defect

Some people are born with a gene defective for a channel protein. The result is that a channel in the plasma membrane is larger than usual and seems to allow Na ions to enter and leave the cell freely. What are the short and long term health ramifications be and what dietary adjustments could we make to help treat such a person?— Preceding unsigned comment added by 68.19.209.91 (talkcontribs)

I was going to say: "Dietary adjustments are best recommended by medical professionals, not by anonymous users on Wikipedia. Please see a doctor instead." but I see from your question above that this probably a homework question. --Canley 02:44, 9 October 2006 (UTC)[reply]

There is no such defect that affects the primary Na-K channel in all the cells of the body. It likely would be lethal. A number of "channelopathies" are known, affecting specific symptoms. No dietary treatment would be likely to affect an Na channel defect with the sole exception that if the defect is primarily restricted to the tubules and collecting ducts of the kidneys, extra dietary salt would compensate to a significant degree. alteripse 12:51, 9 October 2006 (UTC)[reply]

bursting cells

how come an animal cell will burst if it lets to much water in but plant cell dont? — Preceding unsigned comment added by 68.19.209.91 (talkcontribs)

Cuz plant cells got cell walls of cellulose and animal cells ain't now stop asking homework questions and do your own research. It's probably in that bio book of yours. =O Hyenaste (tell) 02:54, 9 October 2006 (UTC)[reply]

nah.. i looked in my book for about an hour for all of these questions and never found anything. ive been looking on the net for at least 3 hours for an answer to any of these and havent found anything close to an answer. — Preceding unsigned comment added by 68.19.209.91 (talkcontribs)

Have you got the right book? Its the biology one you need! 8-)--Light current 03:10, 9 October 2006 (UTC)[reply]

yes...i hae the right book$$~& — Preceding unsigned comment added by 68.19.209.91 (talkcontribs)

Look here Plant_cell--Light current 03:19, 9 October 2006 (UTC)[reply]

If you are really having problems with your book, this website has some good narrated biology animations. Also, please sign your comments with ~~~~ Dar-Ape 03:20, 9 October 2006 (UTC)[reply]

What's in a plant cell that's not in an animal cell? --Bowlhover 05:52, 9 October 2006 (UTC)[reply]
Like Hyenaste said, cell walls of cellulose. And chloroplasts --GangofOne 09:46, 9 October 2006 (UTC)[reply]
Stop bitching at people who ask for help with homework, we dont do homework for people, we are happy to help with it though. Philc TECI 17:43, 9 October 2006 (UTC)[reply]

cell theory

why did it take 150 years after van leeuwenhoek found "animicules" to come up with a cell theory?

six homework questions in a row? do you have a deadline soon? Xcomradex 03:50, 9 October 2006 (UTC)[reply]
Why dont you learn how to use wikipedia by searching yourself? And please sign your comments!!--Light current 03:53, 9 October 2006 (UTC)[reply]
This seems less homeworky so I'll field it (as in, I don't think it's a homework question and you're just biting a newbie). If you didn't know that animals were made from cells you could argue that macroscopic animals are much like single-celled ones (as in, made from organs/organelles and things but do not scale down further). Vitriol 03:55, 9 October 2006 (UTC)[reply]
Grist for the alternative history mill: van leeuwenhoek noticed that the little animicules he scraped off his teeth were killed when he drank a hot beverage. If he (or say Edward Jenner) had developed the germ theory of disease from this observation, how would medical science have advanced? It seems like a lot of time was wasted between van Leewenhoek and Pasteur.Edison 19:02, 9 October 2006 (UTC)[reply]

About the Big Bang

I was reading about the Big Bang, Speed of light and escape velocity among other topics and I ended kind of puzzled. I tried reasoning all that I read and I was trying to reason the following 'postulates':

1. At some point in time everything in the universe was toghether in a single 'point'. 2. Nothing can move faster than the speed of light. 3. The escape velocity of a black hole is faster than the speed of light wich is why not even light can escape. 4. All matter and energy exploded at the event known as the 'big bang'.

Considering all of those postulates as true seams to lead to a big contradiction because if we consider points 1-3 then point 4 would be impossible because it would imply that everything in the universe at some point was moving (considerably) faster than the speed of light (to achieve the incredible speed i suppose would be need to escape from the gravitational pull from all of the mass in the universe combined).

How could that be?

I'm thinking it has something to do with one of my 'postulates' being wrong or missinterpreted. My limited knowlege in this field doesn't allow me to answer this question so I was wondering if anyone here could help me figure this puzzle out.

No this is not a homework and no i don't have a deadline. It's just a question i've been asking myself for a couple of months now.Nnfolz 04:57, 9 October 2006 (UTC)[reply]

While I'm sure someone else will answer this in more detail, the idea is that all the time, matter and space in the universe "inflated" from that point, rather than "exploded". BenC7 05:24, 9 October 2006 (UTC)[reply]
I also won't go into the full details, but I will point out that while special relativity says that no matter/energy/information can move locally faster than light (i.e. an observer very close by in space will never see anything going faster than light), space itself is neither energy nor matter nor information (not quite, at least), and so the space can expand at whatever speed it pleases. Confusing Manifestation 09:04, 9 October 2006 (UTC)[reply]
Another lame answer is that close to the Big Bang (in time), the laws of physics break down. Another way of saying "we haven't a clue". And of course I can't resist pointing to my alternative to the Big Bang theory. DirkvdM 09:18, 9 October 2006 (UTC)[reply]
The speed-of-light limit is with respect to a metric, i.e. a way of giving a meaning to distance and time in order to define what speed means. This metric comes from spacetime and it is spacetime itself that expands, and it is not limited to the speed of light. Simon A. 17:02, 9 October 2006 (UTC)[reply]
The way I understand it is that at the big bang, spacetime itself expanded very quickly. So nothing actually travelled faster than the speed of light. 23:08, 17 October 2006 (UTC)

Human skin colour

Why is human skin the colour it is (ranging from a very pale pink to a dark brown shade)? I know the variations are caused by melanin levels, but I'm curious as to why we aren't blue or green or something. CameoAppearance orate 05:18, 9 October 2006 (UTC)[reply]

Great question. I don't know the answer, but looking at the box at the bottom of brown, I would say that human skin color is really all shades of brown (not so much pink, really). And brown is considered a "neutral" color, although I admit I don't really know what that means. Looking at nature, it seems that things that come in non-neutral colors tend to have good reason to be those colors -- leaves are green because the pigments for photosynthesis happen to absorb the other frequencies. Frogs and peacocks and butterflies come in crazy colors for defense, or sexual selection or stuff like that. But we humans, though we need some color for UV protection, it doesn't really matter what color it is. And things where color doesn't matter -- tree trunks, roots, worms, dog paws, most mushrooms -- seem to tend to be brownish. Could it have something to do with brown being (according to its article) a tertiary color in the "original sense"? It's what you get when you mix together all three primary colors, so maybe when you throw together various atoms and molecules, in the absence of evolutionary pressure on hue, you tend to get brown? --Allen 06:38, 9 October 2006 (UTC)[reply]
I don't have specific answer, but note ALL MAMMALS are in the range of browns to black, and maybe white hair, with only a few exceptions, like purple-assed baboon, and mandrill. The previous answer sounds good to me. --GangofOne 09:40, 9 October 2006 (UTC)[reply]
It seems that in mammals, skin colouring pigments such as melanin and [what was the other one, again?] evolved primarily to protect the skin from UV radiation, while for purposes of camouflage or mimicri, fur colour is varied. As fur is a common chararteristic of all (or most) mammals there was no evelotuionary pressure for varying skin colour -- it would pointless as one cannot see it underneath the fur, and so everything stayed with the UV protection compounds that worked out well. At least that's my guess. Simon A. 17:07, 9 October 2006 (UTC)[reply]
The "other one" is another form of melanin, orangy rather than brown. The two forms are pheomelanin (red to yellow) and eumelanin (dark brown to black).
Atlant 17:16, 9 October 2006 (UTC)[reply]
In order to be blue or green, wouldn't we have to have copper in our blood instead of iron? User:Zoe|(talk) 02:55, 10 October 2006 (UTC)[reply]
Yeah, I suppose. Well, my question's been answered. Thanks! CameoAppearance orate 16:25, 10 October 2006 (UTC)[reply]
Well, humans who don't have much (of either type) of melanin are pink as a result of our red blood, but that relatively-unsaturated color could be easily over-ridden by other pigments in the skin; note in particular the mandrill[1]. Also, pigments aren't the only way to generate color. Some of the most-brightly-colored feathers and scales are often colored as a result of diffraction patterns. So if there were some evolutionary advantage to us being, say, bright blue, that's probably what we'd be.
Atlant 17:41, 10 October 2006 (UTC)[reply]
Interesting. --Proficient 06:39, 18 October 2006 (UTC)[reply]

international awards in physics.

Nobel prize has been critized(particularly in science)for awarding people after their career has almost ended(as if a lifetime acheivement award) rather than award "to encourage young minds". IS THERE ANY UPCOMING AWARD CEREMONY THAT ASSURES PRIZE TO YOUNG MINDS(IN PHYSICS)?____202.71.153.76 05:22, 9 October 2006 (UTC)ANURAG[reply]

Please don't shout. DirkvdM 09:23, 9 October 2006 (UTC)[reply]

see List of prizes, medals, and awards --GangofOne 09:31, 9 October 2006 (UTC)[reply]

Did you try Ig Nobel Prize ? -- DLL .. T 17:03, 9 October 2006 (UTC)[reply]
The IISEF is one of the highest honors. — X [Mac Davis] (SUPERDESK|Help me improve)02:41, 11 October 2006 (UTC)[reply]

prices of toluene

when looking up the prices of toluene in chemical market reporter, I see "Toluene, spot" and then a per pound price. What does the "spot" mean?

Probably Spot price. Clarityfiend 05:42, 9 October 2006 (UTC)[reply]

Oil absorption into skin

When you rub oil into your skin (such as for massage or to moisturize dry skin) it seems to be absorbed. This is probably a dumb question, but can the oil go into the bloodstream and eventually become part of your body fat? Or if not, what happens to it? The more I think about it, the more I doubt it would work that way, but I don't know why. --Grace 05:46, 9 October 2006 (UTC)[reply]

A lot of the ingredients listed in moisturiser will be broken down and either excreted by the body (eg methylparaben), or used in metabolism (things like glycerol and stearic acid). it is possible for compounds to be stored in the fat layers of your body, but they are usually slowly broken down and excreted. Xcomradex 06:15, 9 October 2006 (UTC)[reply]
I'm talking about actual oil, though, like olive oil, which is sometimes used for massage. Surely you aren't saying that people watching their weight should avoid even external contact with fat? :) --Grace 09:20, 9 October 2006 (UTC)[reply]
The major component in olive oil, oleic acid, does travel through the skin. so at least in theory some of it wil contribute to yur overall energy uptake. however this is dependant on how much of it makes it past the outer dead layers of skin (which won't metabolise it) into the bloodstream. i'd say thats probably not a lot. Xcomradex 09:51, 9 October 2006 (UTC)[reply]

chemical engg

oxygen demand for prod of recombinant protein

production of recombinant protein by a genetically engineered strain of e.coli is proportional to cell growth .ammmonia is used as nitrogen source for aerobic respiration of glucose.the recombinant protein has an overall fortmula C H 1.55 O 0.31 N 0.25. THE YIELD OF BIOMASS FROM GLUCOSE IS MEASURED AT 0.48g/g.the yield of recombination of protein from glucose is about 20% (percent) that for cells..

1.how much ammmonia is required?

2.what is the oxygen demand?

3.if the biomass yield remains at 0.48g/g ,how much different are the ammonia and oxygen requirements for wild type e.coli unable to synthesize recombinant protein.......

Please read the text at the top of this page. Do your own homework. Simon A. 12:03, 9 October 2006 (UTC)[reply]
Please tell us what you understand in the text and what needs a particular help from us. -- DLL .. T 17:01, 9 October 2006 (UTC)[reply]

Skies of the distant past/future

Let's say that I want to know what the skies looked like 50 million years ago. Is there a star chart program that I can use for this? Of course it will need to take into account proper motion, because the stars move a lot over millions of years. --Bowlhover 06:29, 9 October 2006 (UTC)[reply]

There's a number of free astronomy programs that do this kind of stuff; I use Celestia although there are certainly other (and maybe better) programs. Keep in mind, though, that errors accumulate over time; it's possible to get an idea of how the sky looked like to ancient Romans (being only about 2000 years in the past), but I would not really trust any such program to show anything meaningful for a date several million years in the past. Ferkelparade π 06:35, 9 October 2006 (UTC)[reply]

effect of growth on oxygen demand

the chemical reaction for conversion of ethanol to acetic acid (C2H4O2) is

C2H6O + O2------> C2H4O2+H20

Acetic acid produced during growth of Acetobactor aceti, which has the composition C H 1.8 O 0.5 N 0.2; biomass yields from substrate is 0.14g/g. Product yield from substrate is 0.92g/g. Ammonia is used as nitrogen source. How does growth in this culture affect oxygen demand for acetic acid production???

I DON'T MEAN TO BE RUDE, BUT DON'T WRITE IN CAPITALS, PLEASE. Aaadddaaammm 08:41, 9 October 2006 (UTC)[reply]

Thanks for removing the caps, but please also sign your comments by adding four tildes. And do your own homework. Simon A. 12:02, 9 October 2006 (UTC)[reply]
It's perfectly fine to ask for homework help here. It is not, however, perfectly fine to dismiss a question with the rude "do your own homework". That sort of rudeness at the reference desk really should stop. If someone's sole contribution to an answer is "do your own homework", I would suggest either [1] silence, or [2] substituting "If you need help with a specific part or concept of your homework, feel free to ask, but please do not post entire homework questions and expect us to give you the answers."- Nunh-huh 12:10, 9 October 2006 (UTC)[reply]

I agree. Since this is the Science desk, we should have a proper calibrated "Scale of Rudeness", going from 1 to 5, with standardized rudeness templates. 'Very Rude' (5) will be reserved for those who copy their homework assignment directly, and don't sign. 'Slightly Rude' (1) is for those who don't use the search box first. --Zeizmic 13:18, 9 October 2006 (UTC)[reply]

i need help

hi im a student doing a national diplomea of allpied sciences and i have been looking for an atricle everywhere and just can find it its called 'a mind to crime' by Anne Moir and David Jessel. i would greatly apprectiate it if you could provide any revelant info.

thank you very much -demoness_althea66611:33, 9 October 2006 (UTC)

Anybody ever told you about Google Scholar. I just typed in "a mind to crime Anne Moir David Jessel", and it immediatly told me that this is not an article, but a book (Signet, 1995). See here: ISBN 071813768X Simon A. 11:54, 9 October 2006 (UTC)[reply]
http://www.amazon.com/Mind-Crime-Moir/dp/0451196295/sr=8-1/qid=1160394637/ref=pd_bbs_1/104-8179265-6067933?ie=UTF8&s=books. ISBN 0451196295. Click on the ISBN link to look for sources. Presumably you can ask your local library to obtain it through interlibrary loan. - Nunh-huh 11:52, 9 October 2006 (UTC)[reply]

Underground nuclear tests

From what I've read about today's test by North Korea, it was tested underground. So, basically, I'm wondering how they do this underground. Do they have a really big cave that it's done in or what? Do we have an article on it that I didn't see linked at the nuclear testing article? Dismas|(talk) 12:49, 9 October 2006 (UTC)[reply]

They said it was in a mine (now abandoned). The difficult thing would be to seal it before the explosion, so it doesn't leak. --Zeizmic 13:13, 9 October 2006 (UTC)[reply]
Of course now it's abondoned. But Kim Jong Il stuffed babies in around the bomb to see the effects. — X [Mac Davis] (SUPERDESK|Help me improve)
That's not so hard, just use a manhole cover... but don't aim it at the ISS! --Jmeden2000 14:37, 9 October 2006 (UTC)[reply]
[after edit conflict]I think in general they just drill a hole, lower it in, and plug the hole, but they could also use old mineshafts. I think most caves might not be in a suitable type of rock. Wikipedia could do with more information if anyone has it.--Shantavira 13:15, 9 October 2006 (UTC)[reply]
It can vary a lot depending on the purpose for the test. But usually you dig a big hole, drop a bomb into it, and cover the hole. You can use old mines or whatever as well. The most important factors are the type of rock you are detonating it in and taking care that even if the bomb goes over the maximum predicted yield it won't vent. Other issues are whether or not the seismic activity will affect nearby people and wildlife but usually that sort of thing is set to the wayside, especially with small bombs. I imagine they also take into account whether or not there is water running through the area (the rock inside the test area will become quite hot and radioactive). One could create an article on it though personally I find the subject pretty boring — the technical issues are predictable and fairly banal. --Fastfission 14:59, 9 October 2006 (UTC)[reply]
Decoupling may also be a factor. You don't necessarily want all the evolved energy going into seismic activity lest everyone else know exactly how big a bang you produced, so you may excavate a larger cavity than you would need merely to hold the nuke's phsyical volume.
Atlant 17:20, 9 October 2006 (UTC)[reply]
News reports in Korea have said that a horizontal mineshaft is thought to have been used instead of a vertical one. --Kjoonlee 18:50, 9 October 2006 (UTC)[reply]
It was done under a 360-metre high hill/mountain. --Kjoonlee 18:53, 9 October 2006 (UTC)[reply]
Thanks for the responses, everyone! Although, in the case of N. Korea, I would think that they would want as much of the energy as possible to go into seismic activity. It makes for better advertising for themselves if they intend to sell the technology to someone else. Thanks again, Dismas|(talk) 21:48, 9 October 2006 (UTC)[reply]
I agree. Most of the effect they'll get out of a nuke is rhetorical or political; there is little reason for them to want to make their tests look smaller than they are. --Fastfission 00:44, 10 October 2006 (UTC)[reply]

That brings into play a new skill: making the bomb appear bigger seismically than it actually is. Everything I've read in the past few years was about tricks to obscure the test. The near-surface location is good, packing with tons of old munitions would add to the effect. It's not hard to get an M4 explosion. Wouldn't it be funny if it wasn't nuclear at all? --Zeizmic 17:14, 10 October 2006 (UTC)[reply]

That's why the Americans (and, I think, the Russians) have various detectors to detect leaks of radioactive materials which would have been given off by the blast. [2]. If the Americans still had their Velas, they could have found the site and nature of the blast precisely (the ones which found the Gamma ray bursts which America mistook for Soviet nuke tests on the dark side of the moon) by detecting the gamma rays and EM pulse from the blast. Laïka 17:15, 11 October 2006 (UTC)[reply]
Which is why all the news reports are still saying things like unconfirmed or alleged about the test. --WhiteDragon 16:56, 16 October 2006 (UTC)[reply]

Reduced salt area

Sometimes while going on a highway we see a sign saying "Reduced Salt Area". What does this mean?

In Korea, only old people drive on highways with reduced salt... --Jmeden2000 14:08, 9 October 2006 (UTC)[reply]
Maybe one of the scientists assigned to the project has access to Wiki...
In seriousness, a quick google returns that a 'reduced salt area' is one where salt used to deice roadways is reduced or eliminated due to conservation concerns. --Jmeden2000 14:32, 9 October 2006 (UTC)[reply]
Not just conservation concerns. Watershed areas are often marked this way to limit the amount of sodium that ends up in the human drinking water supplies.
Atlant 17:23, 9 October 2006 (UTC)[reply]
That may be true for some areas, but most road salt used in the US is actually calcium chloride, since its more effective than sodium chloride and much less harmful to the local ecology (but still not 100% safe). --Jmeden2000 18:48, 9 October 2006 (UTC)[reply]
Here in my part of New England, it's definitely good-ole sodium chloride, probably because it's cheaper. The usual "winter mix" is sand and NaCl. On the other hand, I use CaCl2 when I spread "salt" at all, primarily because I care about my plants and my concrete.
Atlant 00:39, 10 October 2006 (UTC)[reply]
its CaCl2 of course, which is why it is more effective mole for mole than NaCl. Xcomradex 01:40, 10 October 2006 (UTC)[reply]
Correction accepted -- thanks!
Atlant 17:44, 10 October 2006 (UTC)[reply]
Gotta be careful with practical measurement of "effectiveness" though: to get a mole of ions from NaCl, you'll need 29 g of it, whereas you'll need 37 g if you use CaCl2. DMacks 17:58, 10 October 2006 (UTC)[reply]
true, but there are other reasons to be careful with "real-world" measurements too. dissolving NaCl in water gives neglible temperature change, however dissolving anhydrous CaCl2 in water gives a whopping exothermic reaction (release of heat) as the Ca(H2O)n2+ ions form. so in a simply weight for weight measure, i'd say calcium chloride still wins. Xcomradex 01:28, 11 October 2006 (UTC)[reply]

Refraction of light

[copied from talk page ]
Is the light sensible to changes of pressure? For example, the sound pressure, will affect in any way the direction of the light, even could be very little influence? —The preceding unsigned comment was added by 80.103.32.2 (talkcontribs) 30 September 2006.
[end copied]

According to List of indices of refraction, the answer is yes. --Allen 16:14, 9 October 2006 (UTC)[reply]
I agree. However, in most cases, the change due to temperature is larger, see e.g. twinkling. But I remember that in undergrad lab practice, we did an experiment, where one arm of a Fabry-Perot interfreometer is led though a glass cylinder. When the cylinder is being evacuated one can see the fringes move. A pretty complicated and imprecise way of building a barometer. Simon A. 17:16, 9 October 2006 (UTC)[reply]

narcotics

(1)is alcohol a narcotic. (2)is nicotine a narcotic. (3)is caffine a narcotic. i have spent 2 hours on the internet trying to figure this out and i cant. thank you kevin

No, no, and no. Narcotics are, by scientific definition, derived from opium. None of those are. Even under the slightly-looser US legal defition (which includes coca-derived drugs) they aren't included. -- Plutor talk 11:14, 10 October 2006 (UTC)[reply]
To be nitpicky, narcotics are not derived from opium, but work by binding to the opiate receptor. alteripse 23:05, 9 October 2006 (UTC)[reply]
I believe that that is actually the definition of opioids. From a cursory search and a quick read of our articles, it appears that narcotics are a subset of opioids. -- Plutor talk 11:14, 10 October 2006 (UTC)[reply]
In the early 20th century all known narcotics were derived from opium, but that definition is long obsolete. My point was that synthetic opioids are considered narcotics by doctors, pharmacologists, and US law enforcement people but many are not derived from opium in any direct way, but simply share the ability to bind and activate opioid receptors. The word narcotic is somewhat slippery. In a medical context opioids and narcotics are typically considered interchangeable terms. In a US law enforcement context, the term narcotic is sometimes used more widely for all controlled substances, a usage that makes doctors cringe. alteripse 15:27, 10 October 2006 (UTC)[reply]
They're all psychoactive drugs, though. From the article: "In the West, the most common by numbers of users are caffeine, alcohol, and nicotine, in that order." DirkvdM 08:22, 10 October 2006 (UTC)[reply]

Wood Integrity

What is the integrity of pine wood? I would like to know the exact formulas so that I can use them for a medival seige engine.-- Meteshjj We come from the land of the ice and snow 20:55, 9 October 2006 (UTC) [reply]

I did a google search on "pine wood strength", and looked at our article wood, and it looks like the answer to your question might depend on too many things for an easy answer: species, growing conditions, treatments, and drying temperature may all need to be known to figure out exact strength. You might be better off directly experimenting with samples of the wood you have to figure out its particular strength. And do be careful with any war machines you build. You might want to check with local authorities for permission beforehand. --Allen 23:13, 9 October 2006 (UTC)[reply]
I know that there is a specific formula including the aforementioned variables. i counted on testing for the various variabls before I walked into a crapload of math. By the way, It is legal to build a trebuchet in my area, as we often host thePunkin Chunkin Contest.
Older engineering handbooks, from say before 1960, frequently contain this kind of information, and they can also be found very inexpensively in used book shops. For example, Eshbach's Handbook of Engineering Fundamentals, 2 ed. (1936) gives the modulus of elasticity of northern white pine as 1,280,000 lbs. per sq. in., as well as data on other parameters and species. I have no idea if these numbers, probably measured on old growth timber, will hold up for stuff you can buy today. -- The Photon 01:58, 11 October 2006 (UTC)[reply]

Thank. I will try to find the before sited books.-- Meteshjj We come from the land of the ice and snow 05:47, 11 October 2006 (UTC) [reply]

Is it possible to artificially increase a metal's physical weight to indefinitive amounts?

Monday, 10-9-06; Portland, OR; 1:38pm West Coast Pacific Time;

Original post: http://en.wikipedia.org/wiki/Talk:Heavy_metals#I.27m_searching_for_information_on_how_to_artificially_increase_a_metal.27s_physical_weight_to_indefinitive_amounts.

Is there an authority who can provide an answer on how to artificially increase the physical weight of a small piece of metal (say the size of a 1/2 inch diameter galvanized steel washer)? Is this only theory, or has this concept actually been proven? Bear in mind, that I wish to affect only the weight (or density) of this size and not the physical dimensions of this size; in other words, is it possible to artificially increase the weight of this size to an indefinitive weight such as, for example, 50 lbs., 100 lbs., 150lbs., 200lbs., 500lbs., or even to a weight of 1,000 lbs.? Why would I want to try this? For a private science project I'm cogitating. --MyPresentCPUisTooSlow 21:08, 9 October 2006 (UTC) MyPresentCPUisTooSlow[reply]

Since metals cannot generally be compressed much, I think the answer is a firm NO!--Light current 21:19, 9 October 2006 (UTC)[reply]
You could make it out of Osmium or Iridium - the densest relatively common materials that we have available on Earth - a sphere with a radius of about 22 cm (~9 inches) of either metal has a mass of about 1 tonne. Into the realms of sci-fi - and if you could make it out of the same stuff and density that white dwarf stars are made from - then a sphere only 5 mm (~0.2 inches) in radius has a mass of about 1 tonne. Better still, get a lump of neutron star with the same density as a neutron star in the upper mass range - a sphere 5mm/0.2 inches in radius can have a mass approaching half a billion tonnes - and would have a significant gravitational effect if you got close to it!Richard B 21:49, 9 October 2006 (UTC)[reply]
Accelerate it to relativistic velocity; as it approaches the speed of light, it will gain mass. TenOfAllTrades(talk) 02:20, 10 October 2006 (UTC)[reply]
While the mass of the piece of metal is an inherent property of matter, and can only be altered by relativistic effects as it is accelerated to near the speed of light, the weight is technically an effect of the gravitational attraction. Just as you can make a metal washer weightless by putting it in orbit or dropping it, you can make it weigh more simply by taking it and the spring scale you are weighing it with to a planet where the gravity is greater. If you want a 1 ounce (say) washer to weigh 1000 pounds, which is 16,000 times as much, all you have to do is weigh at some place in the universe where the gravity is 16000 times as much as on earth. On the surface of a neutron star the gravity is at least 2×1011 that on Earth, so the washer which weighs 1 ounce on earth would weigh at least 12,500,000,000 pounds.Edison 14:37, 10 October 2006 (UTC)[reply]
As long as you are talking about pounds-force, of course. You should identify them as such when you use them here, because they aren't the pounds a hardware store uses when it sells washers by weight. Gene Nygaard 18:36, 17 October 2006 (UTC)[reply]

Water...Sily question?

oxygen is h20, so if I took 2 parts hydrogen to 1 part oxygen could they be 'mixed' together to form water? Or does it take some form of binding much more complex than simply 'pouring' the two items together?? I know this is probably silly and that it takes much more than this, but I figure this is the perfect place to ask such a question! ny156uk 22:00, 9 October 2006 (UTC)[reply]

I assume you that water is H2O. All it takes is a little heat (or a spark). Hydrogen and oxygen will react explosively to form water. Not recommended for casual users. Dragons flight 22:04, 9 October 2006 (UTC)[reply]
You can also have a controlled burning of Hydrogen - say from a tap connected to a gas cyclinder to form steam - if you put a cold object above the flame, water will form from the steam produced. The reaction doesn't have to be explosive.Richard B 22:46, 9 October 2006 (UTC)[reply]
(edit con) You have to ignite the hydrogen, but also if they are in liquid, volume(ml) * concentration(m/dm3) = moles(m) and you need correct amount of moles not equal volume. You will need twice as many moles of hydrogen as oxygen (since they are both diatomic the ratios will be the same as in the molecular formula), but generally people would use one in excess to ensure the other is completely reacted, most likely oxygen in this case. Philc TECI 22:06, 9 October 2006 (UTC)[reply]

but if you do the reaction in the gas phase, then you can simply mix one volume of oxygen to two volumes of hydrogen then ignite, since to the first approximation the molar volume of all gases is 22.4 L/mol. the above caveat on this reactions explosive nature does apply of course. Xcomradex 01:38, 10 October 2006 (UTC)[reply]

I believe the splendidly-named Gay-Lussac had something to say about this. --G N Frykman 07:00, 10 October 2006 (UTC)[reply]

We even have an article on oxyhydrogen flame. To add a few more things: As alrady poined out: if you mix hydrogen and oxygen you get highly explosive stuff which in German has the nice name Knallgas ("boom gas"). After you ignite it (from save distance) you get a load, sharp boom noise and water, in gaseous form, of course, because an explosio is hot. In class demonstration, one hence does it in a closed glass container such that one can wait for the water gas to cool down and condense. One might be disappointed to see that a litre of Knallgas condenses to less than a millilitre of liquid water. And in case you don't like explosions, a fuel cell is your device of choice. Simon A. 07:35, 10 October 2006 (UTC)[reply]

There are no silly questions, just silly moments to ask them. You actually stumbled on something big. See Hydrogen economy. DirkvdM 08:28, 10 October 2006 (UTC)[reply]
However, there may be such things as silly answers (not that the above is silly). 8-)--Light current 08:45, 10 October 2006 (UTC)[reply]
At the Exploratorium I saw a nice exhibit in which water was broken up by electricity to hydrogen and oxygen, The hydrogen and oxygen were then allowed to exit small glass tips near one another and burned. As I recall, the burning produced water. It was not perpetual motion, since the electricity input to break down the water was greater than the energy obtained by burning it. Edison 15:03, 10 October 2006 (UTC)[reply]
Youre talking complete hydrolysis 8-)--Light current 23:47, 10 October 2006 (UTC)[reply]
If we include the energy converted to sound and light, then the energy needed to split the hydrogen from oxygen should exactly match that created by burning them together. However, since you will never have a 100% efficient mechanism, there will be energy lost in the process, making a perpetual cycle impossible without the addition of outside energy. StuRat 00:52, 11 October 2006 (UTC)[reply]

Mother gull is double-clutching this year...

It's now October and it's starting to get cold, yet the GGB gull hen that nests on the roof opposite me (and has done for ten years or so) appears to be sitting on another clutch of eggs (her first brood fledged about six weeks ago). Any bird experts here? Do you think the chicks have any chance of making it through the winter? I'll do my best to make sure that the hen bird gets plenty of food... --Kurt Shaped Box 22:07, 9 October 2006 (UTC)[reply]

Perhaps the gull laid more eggs anticipating your urge to help her? --Russoc4 23:37, 9 October 2006 (UTC)[reply]
I was thinking that myself. Ever since I saw she had a nest, I've been lobbing food out of my window onto the roof for her. I guess I've made a commitment now... :) --Kurt Shaped Box 23:39, 9 October 2006 (UTC)[reply]
Yes, you've likely changed the whole local ecological balance. StuRat 00:42, 11 October 2006 (UTC)[reply]

October 10

Super Cannon of Doom

I asked a long time ago if it was possible to send an object wieghing 500 tons hurtiling through the air at 7.2 miles per second and the answer was yes but with the amount of energy that little boy the atomic bomb released. So To further the question is it possible, I'll even be happy if you give me the equations to lead me in the right direction, to build a cannon with a ten foot bore capable of sending an object of this wieght? If so how long would the cannon be and what are some Ideas on what compound to use that would create the required energy? Thanks I know this is really random and of course it was inspired by Jules Vernes.

Project HARP--Light current 02:23, 10 October 2006 (UTC)[reply]

So could Harp be put on some steroids for my purposes or it this totally and uterlly impossible.

I think if Gerald Bull could have made it bigger , he would have.--Light current 14:02, 10 October 2006 (UTC)[reply]
To start, here's the article on kinetic energy, to calculate how much energy is required to accelerate the payload to the required speed. The way I crunch the numbers, the amount of TNT required to provide that amount of energy (assuming 100% efficiency, which of course is not possible) would be a cube 16 metres (roughly 50 feet) to a side, and weighing 7,000-odd tonnes. That should give you an idea of the gargantuan scale of any such cannon. --Robert Merkel 05:43, 10 October 2006 (UTC)[reply]
Some points of reference: Jules Verne's From the Earth to the Moon(1865) had a giant cannon fire a heavy projectile from the Earth to the Moon. Skeptics say "bad science": it would have killed the passengers and fallen back to the ground. Big Bertha (Howitzer) a WWI siege mortar fired shells weighing 820 kg each to a maximum range of 12 km. The Paris Gun, another WWI artillery piece, built for long range bombardment, fired 94-kilogram (210 lb) shell to a range of 130 kilometres (81 mi) and a maximum altitude of 40 km (20 mi). The 16 inch guns on the WWII battleship USS Missouri were 16 inch (406 mm)/50 caliber Mark 7 naval gun. The article does not give range, projectile velocity, or muzzle velocity, but an earlier gun is described at http://www.geocities.com/fort_tilden/16ingun.html For a 16" Mk2 they state 2750 feet/sec, projectile weight 2100 pounds, The largest bore naval guns of the 20th centyury were the Japanese WWII 40 cm/45 Type 94 which had 18.1 inch bore and fired a projectile of 1460 kg or 3220 lb at a velocity of 780 m/s or 2560 ft/s to a range of 42,000 meters or 26 miles. The WWII Schwerer Gustav could fire a projectile weighing 4.8 ton (4,800 kg) at a muzzle velocity of 820 m/s, using 2,500 lb (1134 kg) of propellant. Now you can figure the kinetic energy of your proposal and compare it to that achieved by practical artillery. Graph the KE against the weight of the entire gun, and extrapolate to what the approximate required weight and propellant charge of your gun would be. You specified 7.2 miles/sec = 38,000 feet/sec or 11,600 meters/sec. This is way higher than the achievements of artillery designers, so your projectile would likely melt and lose velocity very rapidly, like a meteor reentering the atmosphere. Certainly you would want to consider having the muzzle be at the top of Mt. Everest, to take advantage of the thinner atmosphere. Consider a lower muzzle velocity with rocket power to boost the projectile to the ultimate velocity when it is up to a region of thin atmosphere. Then consider rail guns and mass drivers, or perhaps acquire some old blueprints and build a Saturn V, which could boost 260,000 pounds (130 tons)(118,000 kg) to low earth orbit. You could get your payload into orbit with 4 such rockets, assemble it there, then send up more boosters to launch it on it way to wherever. Edison 16:08, 10 October 2006 (UTC)[reply]

Awesome thanks for the ideas they are exactly what I was looking for.

Is there a way to calculate the arc or when then the projectile will start going down instead of up?

I'm no good with "miles" so 7.2 m is 11.5872768 km, call it 11.6 km/s, or 11600 m/s. Gravity is close to 10 m/s/s, if you divide initial velocity by the acceleration of gravity you get the 'velocity = 0' point, or the time when the projectile starts going down = 1160 seconds = 19.33 minutes. Because it is accaleration constant, the projectile would travel as far as if it had been going half its initial speed for the full length of the journey, = 5800m/s for 1160 s = 6728000 mhad meters, 6.7 thousand km, about 4 thousand miles. Of course wind resistance and orbital motion screw all that up, the above would only be true in a frictionless environment and shooting straight up. Vespine 06:40, 16 October 2006 (UTC)[reply]

An estimaste of the thermal power output and specific heat capacity of a human being

I am curious how long it would take someone like me to overheat if wrapped up in a ton of blankets. Assume an average sized human in a fairly sedentary state. This human is then put in a perfect insulator. About how long would it take for the human's temperature to be raised by approx. one degree celcius? I originally guessed the specific heat capacity for human tissue would be close to that of water but looking at a table in specific heat capacity and noting that the specific heat capacity for wood was about ten times lower, I am not so sure. The equation should be (energy in)/ (mass * specific heat capacity) = (change in temperature). Sifaka talk 02:06, 10 October 2006 (UTC)[reply]

One average human being produces as much heat as a 100W bulb (ie 100W) when sedentary.--Light current 02:20, 10 October 2006 (UTC)[reply]
Since we're mostly made of water, I suppose using the SH of water would give you a good estimate of the temperature rise per hour etc. Energy = mass*specific heat*temperature rise (if I remember correctly).
This is one of those rare occurrences where the calorie comes in handy (although it is not necessary). 100W is 100 J/s. The article says it takes 4.185 kJ to increase the temperature of 1 kg of water by 1 °C. So if we take a 70 kg person, it takes about 30 kJ. So it would take 30,000/100 = 300 s or 5 minutes. So you'd be dead within half an hour. In other words, don't try this at home (if you'd have such a perfect insulator, that is). Or anywhere else, for that matter. DirkvdM 08:42, 10 October 2006 (UTC)[reply]
THis analysis of course excludes the effects of the body's temperature regulation mechanism for which at present I cant find the page (cos I havent looked!). I believe some tests were done on how many blankets to put over a young baby without overheating it. Cot death?--Light current 08:50, 10 October 2006 (UTC)[reply]

All of this stuff is relevant only to dead bodies. Live human beings are homeothermic and will expend considerable energy to maintain a healthy body temperature. alteripse 09:35, 10 October 2006 (UTC)[reply]

I wasnt aware that dead people could generate any heat 8-)--Light current 14:05, 10 October 2006 (UTC)[reply]
Presumably a corpse would generate some heat, since decomposition is a form of oxidation. If the dead person were named Ernest, then heat might be generated by maggots fighting in dead Ernest over tasty morsels. And I've seen 60 watts as the approximate heat output of a person at rest. (Some have slower metabolism than others) People doing sustained work can burn 500 kcal/hour, or if very athletic up to 800 kcal per hour. Another rule of thumb is that a man can produce about 1/8 horsepower of sustained work, while an athlete can produce perhaps 1/2 horsepower for a short while. These figures are off the cuff and totally unreferenced. The stated problem is extremely complex, because an overheated person wrapped in blankets would sweat profusely, and presumably some cooling would occur as the moisture evaporated with the blankets acting as a wick. Respiration can also produce cooling. Per Specific heat Q = n C ΔT where delta T=1 degree, m=100kg for a good sized person, , and c= 4.18, (for a person like water> it would take 4.18 *10^5 Joules, which at 60 watts of heat produced would take 1.9 minutes. At http://www.engineeringtoolbox.com/human-body-specific-heat-d_393.html they give the specific heat of the human body as 3.47 J/*(g*degK), and at various sites the 100 watt output figure is commonly bandied about, so do the math.Edison 16:58, 10 October 2006 (UTC)[reply]
Calories and horsepowers? What century are you from? Also, there is no such thing as degK. There is K, though. DirkvdM 08:32, 11 October 2006 (UTC)[reply]
From the mid-2oth century, and proud of it. Still have my sliderule. Per K (disambiguation), K represents 21 different things, so perhaps you could edit that page and add degrees Kelvin as the 22nd. Deg K= deg C so far as delta T is concerned. Motors are still rated in HP, so it is an interesting comparison. In many parts of the world, human or animal labor still pumps water for irrigation and cultivates fields. A champion cyclist can put out 1/2 HP for 75 minutes, but in competition such cyclists only produce an average of 1/3 HP continuously. The average person can only produce 1/10 to 1/5 HP continuously, per http://qlipoth.blogspot.com/2006/08/our-slave.html The net result is that one barrel of oil represents one year's hard labor for a person. Such labor consumes food energy, usually measured in kcal. Metabolism produces heat. See http://www.ieer.org/reports/energy/3-power.html for an input-output analysis of bullocks in Joules, and HP. A bullock is about 6% efficient. Are you? Edison 14:22, 11 October 2006 (UTC)[reply]
Edison - I think Dirk is just pointing out that the kelvin scale isn't measured in degrees. Matt Deres 18:46, 11 October 2006 (UTC)[reply]

I appreciate the Earnest joke, which should tell you I am no engineer, but I am relying on the definition of specific heat as simply the heat needed to raise the temperature of a substance 1 degree. It takes a heckuva lot more heat to overcome the homeothermic mechanisms of a live human being and raise the body temperature from 37 to 38 degrees than it would to raise the temperature of a corpse by 1 degree (and before you ask, no I have not tested this personally). Are the engineers at the site you linked to that simple, or are you and they assuming a different definition of specific heat? alteripse 21:15, 10 October 2006 (UTC)[reply]

UG Nuclear tests

In films of American UG nuclear tests, you see a large area of ground in the test area sinking rather than being pushed up. Why is this?--Light current 03:29, 10 October 2006 (UTC)[reply]

I expect it's the collapse of the mine or tunnel or cave they put the bomb in, but I'm just guessing. --Allen 04:32, 10 October 2006 (UTC)[reply]
See nuclear testing and Subsidence crater. Dismas|(talk) 04:57, 10 October 2006 (UTC)[reply]

THanks. But I would still expect the ground to bulge before it sinks back, and it doesnt seem to. 8-|--Light current 13:59, 10 October 2006 (UTC)[reply]

Sometimes it does bulge. I imagine it depends on how large the blast is and how close it is to the actual surface. This file shows a graphic in which it indicates some bulge. But the bulging and the sinking are two different things — one is a shock wave, the other is the result of the molten rock having cooled and falled into the newly created hole. --Fastfission 21:22, 10 October 2006 (UTC)[reply]

A question that relates to UG nuclear tests. How are the effects of Electromagnetic pulse affect above ground? Or do the EM pulses never reach the surface? --Agester 12:26, 10 October 2006 (UTC)[reply]

  • You shouldn't get any real EMP effects unless you are in the atmosphere. If you read our EMP article it discusses why this is (they dissipate fairly quickly). --Fastfission 21:22, 10 October 2006 (UTC)[reply]

fainting at the sight of blood

My boyfriend faints when he see's blood. He always has but to different degrees. He has a vagal responce. He gets pale then cold and sweaty. A doctor told him the name of a syndrome he though start with a V. I am trying to find out info. Thanks Pattibeach

Vasovagal syncope? Sifaka talk 04:01, 10 October 2006 (UTC)[reply]
Vampirism? They are reputed to be pale and cold. Edison 17:05, 10 October 2006 (UTC)[reply]
I think a Vampire would be in trouble if he fainted when he saw blood. That's one thirsty Vampire... Benbread 20:19, 10 October 2006 (UTC)[reply]

weapon testing

North Korean explosion caused earthquake of magnitude 3.58 on ricter scale..Any estimation of weapon power??

see 2006_North_Korean_nuclear_test =--GangofOne 06:25, 10 October 2006 (UTC)[reply]

Lead acid storage batteries

Conventional Lead storage Acid batteries have water as the electrolyte, which has the risk of electrolysis resulting in battery explosion.Can any substitute be used for water?

i guess another system with good solubility for sulfuric acid would work, as long as the sulfuric acid could dissociate. i can't find too much on it though. Xcomradex 07:27, 10 October 2006 (UTC)[reply]

This [3] is the best article I could find. The fanciest lead acid batteries use a glass mat in a tight roll. They do not discharge explosive hydrogen gas under normal conditions. --Zeizmic 12:11, 10 October 2006 (UTC)[reply]

Someone from a battery company once pointed out that if a battery is maintained, it is full nearly to the top with liquid, leaving very little space for enough hydrogen to accumulate to produce an explosion capable of rupturing the battery case. He said cases of batteries blowing up were probably cases where the electrolyte level had been allowed to drop very low.Edison 17:08, 10 October 2006 (UTC)[reply]

Critical MAss

After any Mass goes critical,from where does that one neutron come to create the reaction?Even in refining, what i dont understand is that when the percentage of enrichment goes too high it gets critical and undergoes reaction.But how does that happen without the first neutron to start the process?

it comes from spontaneous fission. Xcomradex 07:18, 10 October 2006 (UTC)[reply]
That's fine for a reactor, where the critical mass is assembled statically and you can wait a fraction of a second before a spontaneous fission supplies the neutron that starts the chain reaction. In a bomb, the critical mass may only exist momentarily. In that case an initiator is provided, which is a combination of substances that will emit plenty of neutrons. The Fat Man bomb used a beryllium-polonium initiator with the two substances kept separate until detonation, so the neutrons wouldn't cause trouble eralier. On mixing them, alpha particles from spontaneous decay of the polonium hit the beryllium atoms and the debris includes neutrons. See section 8.1.1.2 on this page. --Anonymous, 04:30 UTC, October 15.

Transition Metal Chemistry question

Hi I need info and examples to do with , Transition Metal chemistry. I need info on , 1. properties, trends, and oxidation states of the first row of transition elements.

2. Uses in volumetric analysis.

3. coordination compounds, coordination number, chelates, isomers, optical activity, substitution reactions.

4. complex ions in medicine: cisplatin, chelation therapy.

5. bonding in complex ions, d-orbitals, colour in complex ions.

6. transition metals in qualitative and quantitive analysis.

7. ellingham diagrams , free energy of oxide formation and carbon as a reducing agent, the thermite reaction.

I also need info on:

1. Thermodynamics: Gibbs free energy and spontaneous change.

2. Redox reactions and electrochemical cells.

3. Info on the non-metallic elements:

i.e properties, trends, oxidation states of the non-metals

i.e the nature o covalent bonding

(ec)try clicking on some of the blue words. and get a good textbook, i recommend atkins, inorganic chemistry. Xcomradex 08:48, 10 October 2006 (UTC)[reply]


This is a reasonably good site for the basic stuff. Also try some of the periodic tables available from www.download.com, some of which have details on each element. For a detailed answer on the reference desk, you will need to be more specific. BenC7 08:35, 10 October 2006 (UTC)[reply]

Question about Kinetic Theory of Gases and the Physical Meaning of Temperature

I have some (perhaps very stupid) confusions about kinetic theory. From the theory we can show that the internal energy of the system is related to the temperature:

So it encourages me to interpret the real meaning of temperature as internal energy of gases. But what will be happen if we evacuate all the gases from the system (i.e. make a vacuum closed black box)? Can we say anything about the temperature of a system with no gas at all? (does a vacuum box has a temperature?)

What is the physical nature of temperature? particle or something? -- 131.111.164.229 14:39, 10 October 2006 (UTC)[reply]

Hm, I have not thought of this lately. My guess is that if you had a pure vacuum with nothing in it, then you'd in principle reach absolute zero. However, in our world, this vacuum is inside a vessel of some sort, and because the vessel has a vapour pressure, you'll have residual gas atoms inside, no matter how hard you try to pump the vacuum. The internal energy is not as simple as that, for example, there are also contributions from rotational, vibrational, and eletronic transitions. Wait for another Wikipedian to chime in. --HappyCamper 15:07, 10 October 2006 (UTC)[reply]
Anyway, because of vacuum fluctuations, it a "box with nothing in it" is a non-physical concept. Batmanand | Talk 15:41, 10 October 2006 (UTC)[reply]
I guess that would be the quantum explanation. --HappyCamper 16:13, 10 October 2006 (UTC)[reply]
Your original conclusion hits the nail. You cannot ascribe a temperature to a vacuum. Maybe I add a few pints to make things clearer. (i) Heat is defined as the internal energy of matter associated with its temperature. The difference between heat and all other kinds of energy is that heat can never be fully used again. See the second law of thermodynamics for details. An intuitive explanation is this: If you have a lot of particles of a gas travelling in the same direction, you have wind. By meand of a windmill you can make use of the kinetic energy of this movement. But if all molecules speed in different directions, so that their movement cancels on average, it is heat. You cannot use the energy easily -- actually, not at all, unless you have another piece of matter with less temperature. (ii) As Happy Camper points out, there can also be energy be stored in internal degrees of freedom. See here for the textbook example. A gas with diatomic molecules as depicted there has twice as much internal energy as a single-atom gas (for which your formula is valid only) at the same temperature and hence it has twice as much heat capacity, with heat capacity being defined as the the internal energy that the matter stores per degree of temperature. Vacuum cannot store any heat energy, and hence has heat capacity 0. As just stated, temperature is internal energy devided hy heat capacity, and 0/0 is undefined, so it does not make any sense to assign a temperature to ideal vacuum. Less mathemematically: A body is called warmer than a reference body at temperature $T$ if it lets heat flow to the reference body, and colder, if it sucks heat out of it. Now, putting vacuum "next to" the reference body does neither, hence it has the same temperature as the reference body, no matter what this body's temperature is. You see, this does not make sense, vacuum does not have a temperature. Simon A. 17:01, 10 October 2006 (UTC)[reply]
A vacuum most definitely does have a temperature. An evacuated space would still contain a photon "gas" which, at equilibrium, is at the temperature of the walls of the container. So the container also has a heat capacity. If you put a hot object in a vacuum at a colder temperature, it will "suck heat" out of it. For an excellent explanation, see "The Heat Capacity of a Vacuum" and surrounding pages at [4] . And then ask any followup questions. --GangofOne 04:28, 11 October 2006 (UTC)[reply]
Ok, I should have expected that somebody brings that up, but I had already written a long answer. Now, there are some subtleties. First, we did not mention blackbody radiation yet. In my example, the two bodies exchanged heat because they touch. But every body also continuously emits and absorbs electromagnetic radiation, in the case of room temperature mainly infrared radiation. If a cool body is standing next to a warm one, even with an evacuated gap in between, they will exchage heat because the hotter one send more radiation to the cooler one than it absorbs from the cooler one. Now imagine a Hohlraum, i.e. a hollow body of matter. The inner walls exchange heat by the means of radiation, and the hollow, evacuated interior is filled with a bath of photons with a frequency distribution corresponding to the walls' temperature (as explained in the notes cites by GangofOne). In this sense, this so-called photon bath has indeed a temperature. But in a way, it is just the temperature of the walls. To see why this is a problem, imagine one wall is continuously heated and the opposite one cooled. In the Hohlraum, we get a mixture of the Planck spectra correspoding to the two temperatures. If the vacuum were replaced by some material filling, we would get a temperature gradient, i.e. the temperature would change smoothly from hot to cold. This is because matter is able to equilibrate: If the atoms vibrate strongly at one place and weakly at the adjacent place, they will exchange energy and end up both vibrating equally strong. Photons cannot equilibrate because they do not "feel" each other. All equilibration can only happen via the material walls. This is why one can ascribe a temperature to en evacuated space if it is surrounded by walls in equilibrium (i.e. all walls at the same temperature) but not in other cases. Another point is: Let's hold a thermometer into outer space. The photon bath there is in equilibrium, more or less and despite what I just said, namely at the famous 3 Kelvin of the microwave background radiation. How long would it take for the matter of the thermometer to cool down to these 3 K. Within the solar system, it would never get as cold, because the sun keeps heating it up. You would have to go really far out to see the 3 K as actual temperature. Simon A. 07:13, 11 October 2006 (UTC)[reply]
Some minor clarifications: for many materials, the heat capacity is not a constant and ; what is true is that . Also, it's often the case that certain degrees of freedom are "frozen out" — even their first excited state is all but unpopulated — and do not count for internal energy calculations; in particular, at normal temperatures diatomic gasses get 5/3 the heat capacity of monatomic, not twice it. --Tardis 15:44, 11 October 2006 (UTC)[reply]

more

Thanks everyone, here is more refined question. Suppose the system is just inside the container (do not regard the container as a system), and contains only one (ideal gas) atom. Then the internal energy of the system is equivalent to the total energy of that atom (correct me if I'm wrong). What is the temperature of the system? Can I calculate the temperature from this formula?

Does it make sense to say about the temperature of an atom? Or would it be better to think about the temperature as macroscopic quantity, and try not to relate it to the microscopic world? -- 131.111.164.110 13:25, 11 October 2006 (UTC)[reply]

The fundamental issue is that a temperature is only ascribable to a system in equilibrium; this is why the interior of Simon's hohlraum has no temperature. (When we speak of temperature gradients, we are saying that "locally" there is equilibrium, or perhaps only a small deviation from it, with a different equilibrium at every point.) As far as a single atom goes, it's not useful to call it an ideal gas because it isn't a collection of particles that interact with each other and their surroundings in any particular fashion (for an ideal gas, this fashion is "not at all" and "only by collision with walls or so"). You can give the atom a temperature with that formula, but remember that in the atom's frame of reference it's not moving at all. The problem here is that a single particle (or even a collection of very few) doesn't have a well-defined average velocity which we can identify to separate a bulk material velocity from a collection of random velocities (whose corresponding kinetic energies count as heat). The problem of average velocity is exascerbated by interactions with the walls; only for a large number of particles will those interactions average to (very near) 0 momentum exchange over a reasonable time period. What you can say without complication or inaccuracy is that there is a particular temperature T at which this particular particle's velocity (in this particular reference frame) would be most typical of a system (e.g., an ideal gas of particles like the one in question): the characteristic temperature associated with that (hypothetical) system and velocity. When applying that equation, be sure to keep in mind the distinction between (even random) kinetic energy and internal energy, which also includes the non-translational degrees of freedom to which Simon referred. Does that help? --Tardis 15:44, 11 October 2006 (UTC)[reply]
That's really help. Thanks very much! -- 131.111.164.228 08:28, 12 October 2006 (UTC)[reply]

Symmetry in technical drawing

I'm not sure whether this should go in Science or Miscellaneous, so feel free to move it if you want. (If there was a Technology section, I would have put it there.)

How does one properly indicate circular symmetry in a technical drawing? I have seen an example here at Wikipedia, but I don't know if that is the best way to do it. Actually, in my case it's a hexagon rather than a circle, so it's not truly circular symmetric. Should I draw it as such anyway or is there another way for my case? —Bromskloss 17:10, 10 October 2006 (UTC)[reply]

To indicate an equilateral hexagon, draw a small line (dash)crossing the mid point of each side (not quite perpendicular) and a small semi circle indicating the inside angle of each corner. The effect you are looking for is to indicate that each side and angle are the same. If you had a hexagon where there were 2 different lengths to the sides and 2 different angles to the corners, you would mark one set of sides with one dash and the other with two dashes, one set of angles with one semicircle and the other with two. I unfortunately don't have an example I could link, in this case a picture would be worth about 100 words ;) Vespine 06:00, 16 October 2006 (UTC)[reply]

WEIGHT OF HUMAN BEING IN THE MOON

WHAT WILL BE THE WEIGHT OF A MAN IN MOON, IF HE IS WEIGHING 60KGS ON EARTH.

~9.9¼ kg B00P 17:30, 10 October 2006 (UTC)[reply]
Why are you using only capital letters? —Bromskloss 17:34, 10 October 2006 (UTC)[reply]
Well, the Kilogram is not a measure of weight, but mass. So the mass on the moon would be 60Kg. The weight would be about 22lb.
This is the relevant calculation (Newtonian-style), resulting in a measurement of 97.62 N. On a kilogram scale calibrated for Earth's gravity, the person would seem to have a mass of 9.96 kg, though that measurement isn't correct (see Mass vs. Weight). -- Consumed Crustacean (talk) 19:05, 10 October 2006 (UTC)[reply]
Lbs also measure Mass, a unit called pounds-weight (equilavent to the weight in Lbs time 9.8) is used for weight. however if you are using metric i.e. kg, then you should measure weight in N --Englishnerd 19:50, 10 October 2006 (UTC)[reply]
No, you shouldn't. Kilograms are the proper units for body weight, in the normal and proper usage in the medical sciences and in sports. Furthermore, the pounds we use for those purposes are also the normal pounds, units of mass equal to exactly 0.45359237 kg by definition, and not pounds-force.
If you weigh 60 kg (not only not KGS, but not Kg either, and with a space between the number and the unit) on Earth, you would weigh 60 kg on the moon. If you weigh 130 lb on Earth, you would weigh 130 lb on the moon (except, of course, in the usage of some science teachers, who just ignore the conventional and correct usage). Gene Nygaard 01:14, 15 October 2006 (UTC)[reply]
Yes, you should, if you're dealing with physics. If you're measuring something in kilograms, it's no longer strictly "weight", but mass. It's common language to treat weight and mass as the same thing, which is fine, but as soon as you get into physics you have to define things more carefully. Neither definition is any more correct, it just depends on context. -- Consumed Crustacean (talk) 04:14, 16 October 2006 (UTC)[reply]
There's no "treating" about it. That is the proper, legitimate meaning of the word weight, well justified in history, in linguistics, and in the law—the very thing physicists like to call mass in their jargon. It is improper for you to pretend otherwise; fortunately, nobody ever gave any physics teacher any say-so as to what the word weight means in the net weight of a can of beans or the troy weight of a bar of platinum or the weight recorded on your chart in a hospital. Of course, mass like weight is an ambiguous word with several different meanings; when we are talking about our weight, we wouldn't want it confused with the muscle mass as it is used by bodybuilders, for example. Gene Nygaard 14:37, 17 October 2006 (UTC)[reply]
If he's in the centre of the moon, the weight would be 0N.
If he is weighing 60kg on Earth (notice the grammar) then obviously his weight on the Moon is 0 because he isn't there. The reason I don't give a proper answer to follow this up is that I don't like being shouted at. DirkvdM 09:55, 11 October 2006 (UTC)[reply]
Actually, even if the person is at the surface of the Earth, the moon still applies a "weight" on the person. Using universal gravitation, plug in the mass of the Moon, the 60 kg, and the orbital radius of the Moon, and you'll get the lunar weight of the person on Earth. Titoxd(?!?) 04:22, 15 October 2006 (UTC)[reply]
I like slinches better (1 slinch = 1 lbf·s²/in = 386 lb = 175 kg). Gene Nygaard 04:03, 16 October 2006 (UTC) Or, of course, 3 hundredweight 3 stone 8 pounds, for those who think hundred is written in digits as "112". Gene Nygaard 04:06, 16 October 2006 (UTC)[reply]

having laser eye surgery

im thinking about having laser eye surgery so that i don't need to wear glasses or contacts. what is the minimum age for this? what is the average cost? and are there any risks? thanks

I had this last year - it was great. In the US, the costs run from $500 to $2500 per eye ($500 is dangerously inexpensive -- doctors who charge this typically do it by cutting corners that SHOULD NOT be cut). It can't be done until after puberty - a candidate in his early-to-mid-20s is ideal. There are a number of risks, but most of them are low (IIRC, the chance of infection is 1 in 500; the chance of the microkeritome malfunctioning is 1 in 500, 'etc). Raul654 17:33, 10 October 2006 (UTC)[reply]
Also - it's not for everyone. If your vision is too good (say 20/80 or such) they doctors won't risk it; if it's too bad (say, index of -12 or worse) they won't do it and will recommend something else (like an implantable contact lense).
It's also important to recognize that as you get older, your lenses lose their ability to accommodate (change focal length). Eventually, you end up stuck at one focal length. So you can get surgery to change that focal length, but you'll still need glasses for any distance outside of your depth of field. Me, I'm happy being stuck at nearsightedness.
Atlant 17:54, 10 October 2006 (UTC)[reply]
Cuba offers free eye surgery, and they're pretty good at it (among the best in the world I believe). Don't know what the requirements are to get it. You'll probably have to live in a country in South or Central America or Africa, where they have a programme. This appears to be called operation miracle, which is now active in 25 countries and aims to do 600,000 operations per year. You probably have to be poor too, to get it for free. But if you pay, it might still be cheaper than elsewhere and the quality of the work seems to be high. Whether they do this specific kind of operation, I don't know. DirkvdM 10:11, 11 October 2006 (UTC)[reply]
The best person to talk to about this is your eye doctor - your eyes need to be 'stable' (ie not becoming weaker, so your glasses' prescription should not be changing). Also, the doctors need to examine your cornea, some people have a cornea that's too thin for them to remove anything so the operation can't be safely done. — QuantumEleven 15:18, 11 October 2006 (UTC)[reply]

Cell Biology

who is Singer Nicholson and what was his contribution to the cell discovery

You're probably thinking of S. J. Singer and Garth L. Nicolson, who proposed "The Fluid Mosaic Model of the Structure of Cell Membranes" in 1972. Melchoir 20:45, 10 October 2006 (UTC)[reply]

Why are Magnets RED?

I would like to know why MAGNETS are usually painted RED please ?

When i've seen painted magnets, they're usually pained red and blue to distinguish between the poles. Benbread 20:16, 10 October 2006 (UTC)[reply]
Its a nice color? 8-)--Light current 21:32, 10 October 2006 (UTC)[reply]
It's probably just something people did to magnets early on that stuck and became universally recognizable as the visual appearance of a magnet. I mean, if you see a piece of wood painted the same as a magnet, you'd probably assume it's a magnet until you pick it up. -Obli (Talk)? 21:36, 10 October 2006 (UTC)[reply]
I blame Hollywood! Nearly all magnets I've ever seen were just black, and not shaped like a horse shoe either. - Dammit 21:37, 10 October 2006 (UTC)[reply]
I'd blame cartoons, actually. Although a horseshoe magnet does have quite a powerful magnetic field near its poles (pity there's no article on them, apparently). Confusing Manifestation 08:34, 11 October 2006 (UTC)[reply]
Maybe I'm stating the obvious but no one has mentioned it yet, the red wire is always positive, so the positive pole of a magnet is painted red for the same reason. Vespine 03:34, 16 October 2006 (UTC)[reply]

magnitude of electrical force

I have been trying to help my son with his homework and we are stumped, please help. How does the magnitude of electrical force between a pair of charged particles change when the particles are moved twice as far apart? Three times as far apart?

You want Coulomb's law. Melchoir 20:46, 10 October 2006 (UTC)[reply]
In fact, the force is given by
and what you want to do, is to look at how F (the force) varies with d (the distance). You could say, find the ratio of the forces, something like
Do you see why finding the ratios of the forces is helpful? Lots of stuff cancels out, and you're only left with what is important. Plugging in some numbers might help. Let's say, you have two charges at d1 apart. And you move them apart 10 times far. That means, the new distance d2 is now d2 = 10 d1...do you see what to do next? Does this help? --HappyCamper 20:52, 10 October 2006 (UTC)[reply]
For extra points: now determine how electrical force varies with distance for spheres, long cylinders (or wires) and large planar surfaces.Edison 14:25, 11 October 2006 (UTC)[reply]

Electricity

Sometimes you hear someone say that a particle appliance " uses up" electricity. What is it that the appliance actually uses up and what becomes of it? Thanks!!!!!!!!!!!

What is a particle appliance? --HappyCamper 20:53, 10 October 2006 (UTC)[reply]
I believe they meant "particular". Dismas|(talk) 21:37, 10 October 2006 (UTC)[reply]
Energy, in joules, or more commonly known to the average person in Kilowatt hours, because thats what you pay for. And the energy is converted into another type, often kinetic heat or light around the home. Philc TECI 20:55, 10 October 2006 (UTC)[reply]
Way back at the power station some form of energy, often heat from burning coal, oil or gas, sometimes heat from atomic fission, sometimes energy from falling water or sunlight, is used to spin a generator. This produces alternating current which is transformed and transmitted to you through the Electric power transmission system. It then provides heat, light, or mechanical motion for your use. What is used up is obviously fossil fuel or nuclear reactor fuel, or water stored behond a hydroelectric dam. In addition, the transformers, substation equipment, and transmission lines, distribution lines, and distribution transformers are "used up" because they are very expensive to install and have a limited life, so even solar, wind, and hydro power cost something to produce and deliver. Incidentally, no one from a utility comany ever said nuclear power would be "Too cheap to meter." That famous quote was from Lewis Strauss of the Atomic Energy Commission, in 1954. What becomes of it is it turn into heat for the most part, excepting for the moment any which is transformed to another form of energy, such as charging a battery or lifting a load to a height. It will be turned to heat later, when that stored energy is used. Edison 22:04, 10 October 2006 (UTC)[reply]
Sunlight is usually not used to spin a generator, but used to produce electricity via a solar cell. Don't forget about wind either as Zeizmic sais later on.
Plus it turned out that nuclear power was vastly more expensive than he had thought, as you have to develop, and refine the technology, build and decommission plants, mine the fuel, and bhide the waste, not just run the plant, which is all he had accounted for. Philc TECI 22:32, 10 October 2006 (UTC)[reply]

Hey, don't diss nuclear! We're tired of the windmills eating up all the birdies, so we're going to warm up some more atoms! --Zeizmic 00:01, 11 October 2006 (UTC)[reply]

Nothing uses up electricity because electrons are indestructable!!! 202.168.50.40 00:01, 11 October 2006 (UTC)[reply]

Are not!!! Clarityfiend 03:30, 11 October 2006 (UTC)[reply]
Are too!!!Bromskloss 12:58, 11 October 2006 (UTC)[reply]
Having said that, electrical appliances uses up (some of) the energy carried by the electrons. What is used up is energy , not electrons.
What is "used up" is electrical energy
It's too much effort for lazy humans to say "electrical energy" all the time. So they just kept saying "electricity" instead.

dissolving

does iron dissolve in copper sulphate

Is this a homework question? And did you check the articles? bibliomaniac15 23:39, 10 October 2006 (UTC)[reply]
The first article you should check should probably be dissolve. Also, note that copper sulphate decomposes at 650°, and Fe doesn't melt until 1538°.Tuckerekcut 02:54, 11 October 2006 (UTC)[reply]
The melting point of iron is a bit of a red herring. Salt (NaCl) only melts at 801 °C, but dissolves quite well in water at room temperatures.  --LambiamTalk 06:11, 11 October 2006 (UTC)[reply]

This is a (rather badly written) question about metal displacement reactions. And the answer is yes. --G N Frykman 06:47, 11 October 2006 (UTC)[reply]

  • Last time I checked nothing could dissolve into something that wasn't a fluid. - 131.211.210.14 10:51, 11 October 2006 (UTC)[reply]
From the article, "Copper(II) sulfate decomposes before melting." Therefore iron cannot dissolve in copper sulphate, because there is no such thing as a fluid phase of copper sulphate. I also added that iron does not melt 'till 1538° because it would not even be possible to do the inverse: one could not use molten Fe as a solvent for copper sulphate, because the latter would decompose before dissolving. No red herrings, no displacements, it's just not possible by any stretch of the imagination without changing the question. Please remember to be extra cautious when correcting another user because it can lead to even more confusion for the question asker. Tuckerekcut 13:30, 11 October 2006 (UTC)[reply]
If you're not part of the solution, you're part of the problem. Edison 14:27, 11 October 2006 (UTC)[reply]
I always thought that was "If you're not part of the solution, you're part of the precipitate."
Atlant 15:50, 11 October 2006 (UTC)[reply]


The original question probably should have been: does Iron dissolve in a solution of aqueous Copper Sulfate? I think the answer to that would be "no," the iron would not displace the copper... but it has been a while since I did metal displacement chemistry... the place to look would be a table of solubilities. Find out whether Iron Sulfate is more or less soluble than Copper Sulfate... I'm sure this table is in your textbook, since this is probably a homework question. Nimur 18:34, 11 October 2006 (UTC)[reply]

If we're really talking "just iron" here, a piece of metal instead of ions, you have a redox situation, not a solubility equilibrium. One could use the electromotive series as a rule-of-thumb about what metal will dissolve in a solution of what other metal-ion. DMacks 20:38, 11 October 2006 (UTC)[reply]

October 11

Tryptophan versus melatonin

What would be the fundamental difference in ingesting a dietary supplement dose of tryptophan versus one of melatonin? If tryptophan eventually becomes melatonin, would they have a similar effect on the body? However, since tryptophan also becomes serotonin, is it safe to say that it boosts the level of the neurotransmitter to also help with depression? Finally, what would a doctor consider between the two before recommending one to a patient for sleep aid? Sybil Gray 03:15, 11 October 2006 (UTC)[reply]

Tryptophan is a precursor for many important molecules used in the body, and cannot be synthesized in humans, thus it is considered an essential nutrient. As you have mentioned, melatonin can be made using tryptophan. In general, all you need to take (in this simplified model) is tryptophan. As long as there is enough tryptophan the body will make the appropriate amount of downstream chemicals, such as serotonin and melatonin. If, however, there exists an error in one of the pathways for these downstream chemicals, such as mutation or downregulation of an enzyme in the pathway, one may end up with a pool of downstream ligands that is less effective than normal. Depression is thought to be due to a paucity of serotonin in the synaptic cleft, as you intimated. Therefore it is theoretically possible for depression to be caused by low tryptophan. However, a drop in serotonin probably would not be an immediate effect of tryptophan shortage, and certainly would not be the first clinically relevent change, as many many important chemicals (and even other amino acids) are made from tryptophan. Depression is more likely to be caused by a change in a protein that controls the formation, release, binding, or reuptake of serotonin than a decrease in building blocks. So with that primer, to answer your questions in order:
What would be the fundamental difference in ingesting a dietary supplement dose of tryptophan versus one of melatonin? --Tryptophan would give your body the choice to make lots of different chemicals, melatonin would not. If tryptophan eventually becomes melatonin, would they have a similar effect on the body? --No since tryptophan also becomes serotonin, is it safe to say that it boosts the level of the neurotransmitter to also help with depression? --No what would a doctor consider between the two before recommending one to a patient for sleep aid? --Present complaint, Past medical, surgical, social, and family medical history, Allergies, genetic predispositions, presence of phenylketonuria or alkaptonuria.... maybe you should ask one.
Ask your doctor if she thinks it is necessary to take these or any supplements/medications for your problem.Tuckerekcut 04:01, 11 October 2006 (UTC)[reply]
  • Thanks a bunch for your thorough answer. Sybil Gray 20:06, 11 October 2006 (UTC)[reply]

Methane for CO2 manufacturing?

Are there any process available for produce CO2 for the beverage industry using Methane gas? I'm interested in a design of an anaerobic digester which produce Methane and CO2 for some wastes....

So want to know about any processes involved in converting Methane to CO2..!!!

Is Burning of Methane to produce CO2 is effective? Can we collect CO2 which release from burning Methane in a easy procedure??

A Detailed answer will be very useful...Thanks...!!!

Sithara from Sri Lanka

You certainly can collect carbon dioxide from methane combustion, or directly from fermentation. However, to produce sufficiently pure carbon dioxide for use in food production, it is apparently necessary to apply various treatments to reduce impurities in the collected stream. I'm no chemical engineer, but it appears to be a reasonably complicated, multi-stage project to do so.
A google search on "carbon dioxide production", turned up Witteman, which sells complete CO2 production/recovery systems for the beverage industry, and Johnson Mathey Catalysts, which sells catalysts which are used as part of the purification process.
I hope these links serve as a useful starting point. --Robert Merkel 05:01, 11 October 2006 (UTC)[reply]

belly

ahhhh! why is one side of my belly bigger than the other. And do i have to do excercises such as running to get a six-pack, can't i just do sit ups.

The front always sticks out more than the back.... --Zeizmic 12:35, 11 October 2006 (UTC)[reply]
Think of it this way: you already have a six pack, it's just covered by a keg of fat. Doing sit-ups will make the six-pack more pronounced, but it will not be obvious unless you lose the fat covering it. To lose this fat, you must do excersizes that help you lose body fat. The body doesn't really care where the fat it uses for energy comes from, so people tend to lose fat slowly from all over their body rather than from a specific place (even if the excersize is for muscles in a specific place). Since aerobics will burn more fat in the long term, they are your best bet for revealing your six pack, and getting healthier at the same time. And please sign your posts, click the sour squiglies below the text creation box where it says "Sign your name". If one side of your belly is noticably bigger than the other, you should get checked for these conditions, and possibly more, and probably in this order: having to poop, having just eaten, scoliosis, and hepatomegaly (if right is bigger).Tuckerekcut 13:39, 11 October 2006 (UTC)[reply]
If requesting medical or legal advice, please consider asking a doctor or lawyer instead. A doctor could rule out tumors or rupture. That said, a combination of exercise (to build up the six-pack muscles) and sensible eating (to get down to a healthy weight) is required. Edison 14:32, 11 October 2006 (UTC)[reply]
Haha, "legal"?! Aren't we getting overly paranoid now? Must…not…get…sued! :-) —Bromskloss 19:10, 11 October 2006 (UTC)[reply]
See a doctor. I don't know whether or not it's physically possible for you to have my experience, but one side of my belly used to be bigger than the other, and it turned out that I had an ovarian cyst and the lymph nodes in my groin on the same side were swollen. Anchoress 19:13, 11 October 2006 (UTC)[reply]
Who says it's a guy? — X [Mac Davis] (SUPERDESK|Help me improve)
I certainly didn't, and a quick scan of the other replies indicates that no-one else did either. Anchoress 04:31, 12 October 2006 (UTC)[reply]
  • In my experience, it's mostly men obsessing about their sixpack. Women just want to be slim, not neccesarily have a sixpack. - Mgm|(talk) 09:59, 12 October 2006 (UTC)[reply]

Because your body is not absolutely symmetrical. You won't obsess over things like that once you hit, say, 24. -THB 10:34, 16 October 2006 (UTC)[reply]

rethinking science!

hi, has anyone heard or read the book THE FINAL THEORY ?because this book denies what we've known till today including newtons and einsteins laws!please do tell me if you have read the book!

I just read some comments. It appears to be quasi-religious in that people who oppose it, can't read through the whole book, and people who read the whole book just want to discuss it with their own tribe (because partial-readers couldn't possibly understand the magnificence of it all). Reminds me of other such books in the past. --Zeizmic 13:47, 11 October 2006 (UTC)[reply]
It's lauded by some non-scientists. That's about it. People without knowledge of topics are often very opinionated about them, but that doesn't make their viewpoints any more correct. This seems to be quite a problem in science, unfortunately. -- Consumed Crustacean (talk) 21:18, 11 October 2006 (UTC)[reply]
Final Theory makes me roll on the ground laughing. Just look at the example below:
Q: But don't we know all about the gravity of Black Holes and how even light can't escape?
A: No. This often-repeated error is based on a simple oversight. Black Holes are said to form when a star expends its nuclear energy and collapses. But starlight only shines from intact, functioning stars, of course. There is no more reason to expect light to shine from Black Holes than from a burnt-out, smashed light bulb. This is a commonly repeated error in plain view that is intended to showcase and dramatize our scientists' deep understanding of Black Holes and gravity, but which actually exposes how little they truly understand about either.
  • Conventional Physics sez "No light comes from black hole because its gravity is so strong that even light cannot escape."
  • Final Theory sez "A black hole is a star that produces no light because it is simply a burnt-out corpse."

This book The Final Theory: Rethinking Our Scientific Legacy (Paperback) by Mark McCutcheon is a genuine work of art. It's the funniest book, I have ever read in my life. It's the ultimate achievement of human stupidity in the field of natural philosophy.

Holy crap, that did come from the book's website. Let me retract my previous politeness. This is clearly insanity. From what I can see (from the above question, the sample chapter, the sane reviews, etc.), the book ignores empirical evidence and spews out good sounding nonsense regarding physics which on its face sounds reasonable to people who have absolutely no knowledge of the topics at hand. It also glosses right over the mathmatical backgrounds of the existing theories. :/ -- Consumed Crustacean (talk) 17:14, 12 October 2006 (UTC)[reply]
Yup, it's perfect for the clue-immune echo chamber that is its target audience. At least I hope that's all who read and believe, rather than being swayed to believe any of this ridiculous crap. DMacks 18:38, 12 October 2006 (UTC)[reply]

Quantum field theory

we know that schrodinger equation is associated with a non relativistic single particle,moving onto relativistic description of the single particle, we get klein gordon equation and then modified dirac equation.Now in QFT, when we quantise the field we see that both Klein gordon n dirac pictures get modified to many particle picture and also a single particle state is described by the field.so my question is do we discard the wave associated with the particle here as given by schrodinger equation (as here we r associating a field with single partice).if not, then also how do we get to know that is it a wave or field assocaited with the single particle.if the answer is that the particle is relativistic then it is field associated with it n if it is non relativistic then a wave is associated with it. Then the confusion lies in the fact that we know that beta decay is explained by QFT, then is it so that the neutron there is relatistic which goes to proton and other new particle, but that also can not be the case as in other case of Z0 getting produced at rest then it decays to other particle states which are again explained by QFT and here the particle is not relativistic so how can we associate a field with it.can u plzz tel where i m missing the concept.Its very important for me to understand this!!!!!!

This scores a 'Rudeness Scale of 1' (insert standard response here). You didn't sign, and it gives me a headache to try to get through the 'txt'. Perhaps there are more brilliant (and patient) people out there... --Zeizmic 13:51, 11 October 2006 (UTC)[reply]
Seems not rude at all to ask a detailed question and say "plzz" in asking for help. Wish I had a clue about the answer.Edison 14:35, 11 October 2006 (UTC)[reply]
I think the "rude" aspect comes from the fact taht this person does not seem to deem us worthy of proper formatting or spelling, and instead has peppered what would otherwise be a perfectly fine question with the lingo of an teenage instant message conversation. It makes it pretty hard to take seriously, and it is quite unpleasant to try and read through. If they don't have enough time to type out a clear question, how can they expect us to take the time to type out a clear answer? --Fastfission 00:41, 12 October 2006 (UTC)[reply]
I'm fairly certain it's not a sensible question, either. I'm getting a strong impression the questioner is quite a bit out of his/her depth here; You can't carry over Schrödinger equation solutions into the Dirac equation. --BluePlatypus 05:12, 12 October 2006 (UTC)[reply]
I agree that not using punctuation is slightly rude, but the "plzz" compensates. Now, let's go to business and try answering the question, ok?

So, I think, there is just a confusion over the terms wave and field. An electromagnetic field, e.g., is the property of space of having electric and magnetic forces of varying strength at varying places. Now, if these oscillate, i.e., change in magntidue and sign periodically, we call it a wave. Now, quantum mechnichs has always been lax about this distinction. First, a wave packet, i.e. a quantum-mechanical particle, which is half-way localized does not oscillate in the magnitude of its field, only the phase rotates periodically. (Hope you know enough about Schrödinger's equation to understand what I mean.) NOw, what does quantum field theory (QFT) add to this: As long as their is only one particle, and energies are so low that we ca neglect particle--antiparticle pair creation, everything stays the same. Only we call it a field instead of a wave. If we have two particles, we have to take care of quatum statistics ad correlations: We should not associate oe wave function with each particle, but rather a joint wave function (amplitude that particle I is at point x1 and particle II at point II).See Slater determinant for details. To get a grip on the algebraic difficulties this involves, one introduces "field operators" (see second quantization). And if we then add a Lagrangian density to describe how our fundamental particle interact, and do a series expansion to take virtual particles into account and maybe even care about renormalization, the you we have mastered quantum field theory. But if you want to get an idea of this without having to study physics to a Master's degree, you might like Richard Feynman's classic little book QED -- The strange theory of light, which explains the essence of quantum field theory for the layman. Simon A. 06:41, 12 October 2006 (UTC)[reply]

Colours

What colour is a mirror? Englishnerd 15:57, 11 October 2006 (UTC)[reply]

Silver? Melchoir 17:36, 11 October 2006 (UTC)[reply]
In the technical sense, most mirrors have no colour as they reflect all frequencies within the visible spectrum about equally. Certain mirrors which use coloured metals as the reflective agent, use coloured glass or plastic as the transparent substrate, or dichroic mirrors do appear coloured, of course.
Atlant 18:11, 11 October 2006 (UTC)[reply]
Supposing there's an unequal reflectance - how much would it take before the human eye detected a tint to the mirror? What units would be used, "intensity vs. wavelength"? "Lumens vs. wavelength"? How sensitive is the human eye to differences in intensity? Or, to differences in wavelength, for that matter? Nimur 18:38, 11 October 2006 (UTC)[reply]
It depends a lot on whether you are making a direct comparison (with the two colors next to each other) or trying to make a comparison that depends on memory (even if only briefly). With two colours right next to each other, your eye is surprisingly good; watch MPEG-compressed video of a dark sky sometime and you'll almost certainly see the banding between slight colour and (especially) lightness differences. But if the colours being compared aren't right next to each other, your sensitivity to color and lightness variations goes way down.
Atlant 17:26, 12 October 2006 (UTC)[reply]
I would say a typical mirror is white or nearly so. When a white surface is polished enough to be reflective, it appears silver or mirrored. And of course, white reflects all colors. Mirrors can also be tinted different colors, of course. --Ginkgo100 talk · e@ 20:29, 11 October 2006 (UTC)[reply]
Most mirrors are silver - not the color silver, the element silver. You coat one side of glass with silver and put a protective coat over the silver. So, when you look at a mirror, you are looking through clear glass to a plane of silver (the element). Therefore, the question is, "What color is silver (the element)?" --Kainaw (talk) 20:37, 11 October 2006 (UTC)[reply]
Why, white, of course. :) Actually, silver says its color is silver. --Ginkgo100 talk · e@ 23:46, 11 October 2006 (UTC)[reply]
Actually a mirror is colored Aluminum. http://en.wikipedia.org/wiki/Mirror
Damn. I need to keep up on my mirror technology. So, since they use aluminum instead of silver now, the question is: What color is aluminum? --Kainaw (talk) 02:45, 12 October 2006 (UTC)[reply]
Silver. =D Hyenaste (tell) 02:52, 12 October 2006 (UTC)[reply]
Its all to do with coherence of the reflected light. If you had an incoherent mirror, it would look white.--Light current 10:38, 16 October 2006 (UTC)[reply]

REPRODUCTION

REPRODUCTION

No, it's Reproduction!

lol sex -Obli (Talk)? 16:54, 11 October 2006 (UTC)[reply]
Could someone suitly emphazi this for me? Hyenaste (tell) 01:10, 12 October 2006 (UTC)[reply]
Haha, this joke is still alive? ☢ Ҡiff 02:56, 12 October 2006 (UTC)[reply]
Only in our memories... it seems to have been eradicated from the collective intelligence within WP... --Jmeden2000 17:49, 12 October 2006 (UTC)[reply]

vortex ring simplification

" the shell discharges , accelerating the air in the shell cylinder and channeling it into a ring, and ring of accelerated air hits the target and knocks it down"

is this a good laymans description of this weapon http://en.wikipedia.org/wiki/Vortex_ring_gun it is just air accelerated by the discharge of the black shell channled into a hills vortex right?

Robin

October 12

sodium hydroxide

How is sodium hydroxide manufactured?

Sodium hydroxide#Manufacture. Hyenaste (tell) 01:06, 12 October 2006 (UTC)[reply]
Have you looked at our article on sodium hydroxide? (Use the 'search' box in the navigation box on the left side of your screen.) TenOfAllTrades(talk) 01:07, 12 October 2006 (UTC)[reply]

Mould

Hi! I'm having trouble finding explanation for the word "Saprophyte" (i know its something about mould, but what!?)74.12.96.221 01:00, 12 October 2006 (UTC)[reply]

Have you looked at our article on saprophytes? (Use the 'search' box in the navigation box on the left side of your screen.) TenOfAllTrades(talk) 01:07, 12 October 2006 (UTC)[reply]

I did but still it didnt give the clear explanation! =( 74.12.96.221 01:14, 12 October 2006 (UTC)[reply]

Did you really? The answer is quite clear in the first sentence. Here is a computer generated list of bite-size definitions of "saphrophyte". At least one should make sense. Hyenaste (tell) 01:22, 12 October 2006 (UTC)[reply]

Thank you very much! The first line gave me the clearest explanation! TY again! =D 74.12.96.221 01:25, 12 October 2006 (UTC)[reply]

I like the third one the best, but no problem; glad to help! Hyenaste (tell) 01:27, 12 October 2006 (UTC)[reply]
I liked the Wikipedia article the best. This person obviously did not try very hard to understand that fist sentance. It looked too hard? :) — X [Mac Davis] (SUPERDESK|Help me improve)04:10, 15 October 2006 (UTC)[reply]

How many "frames" per second can our eye process?

How many times per second does the human eye snap (for lack of a better word) what it sees and process it? In other words -- what is the FPS of the human eye? Pesapluvo 02:36, 12 October 2006 (UTC)[reply]

It is different per human, but it is below 60FPS. Why? If you could see beyond 60FPS, you'd see the film in a regular 35mm projector flicker. I believe it is also below 50FPS because you don't see flicker on PAL unless it is interlaced. Of course, the interlacing is on every other frame - creating a 25FPS flicker. That 25FPS is very easy for anyone to see. Therefore, I believe it is just below 50FPS. This does bring up an experiment I've wanted to do but haven't had the time. Ever notice spokes on a tire stop and start moving backwards at high speed? Does everyone see the spokes freeze at the same RPM? I doubt it. I'd like to experiment with the range of speeds that the tire spins to get people to see it as standing still. Then, does the change in speed correlate to anything: age, gender, race, IQ... --Kainaw (talk) 02:41, 12 October 2006 (UTC)[reply]
For the original question, see our article Flicker fusion threshold. For Kainaw's unperformed experiment, see Wagon-wheel effect.  --LambiamTalk 04:34, 12 October 2006 (UTC)[reply]
Some relevant articles are Flicker fusion threshold, Frame rate, and Persistence of vision. In general, the ability to perceive flicker dictates a higher frame rate than the need to have a difference between the frames for motion to seem continuous. Silent movies at 16 frames per second seemed like continuous movement, as did sound movies at 24 fps or TV in the USA at 30 fps, with a scan 60 times per second, interlaced. Some projectors flashed each frame more than once to avoid flicker. One article relevant to this is Eadweard Muybridge. Persistence of vision has an animation at 12 fps which looks reasonable continuous. Cheaply done cartoons may use 8 frames per second or less, but they are repeated for 3- or 24 frames per second to avoid flicker. This also relates to Apparent motion and Phi phenomenon. Some electric utilities in bygone years provided 25 cycle alternating current, which caused annoying flicker for some customers. Most of the world provides 50 Hertz ac, and the US provides 60 cycle ac, which generally do not cause noticeable flicker in electric lights.Edison 04:48, 12 October 2006 (UTC)[reply]
A minor nit: 50 Hz power delivers 100 half-sine-waves of power each second, so (most) lights on 50 Hz power flicker at 100 Hz. Likewise, 60 Hz power -> 120 Hz flicker.
True for an incandescent bulb, but some types of light flash only on one polarity, like neon. Fluorescent bulbs are supposed to flash on each half cycle, but may rectify at the end of the bulb when near the end of its life cycle.Edison 18:24, 12 October 2006 (UTC)[reply]
Also, be sure to check out the "talk" pages on either Flicker fusion threshold, Frame rate, or both. Last I knew, there's more detail on the talk pages that hasn't yet been incorporated into the articles themselves.
Atlant 17:33, 12 October 2006 (UTC)[reply]
See this archived RD question, wherein the same articles as linked here are used to explain this question (to me) pretty directly. --Tardis 17:16, 12 October 2006 (UTC)[reply]

Radiocarbon dating

In Wikipedia's radiocarbon dating article, as well as other sources ([5] and [6] for instance), the half life of C14 is given as 5730 \pm 40. Does the plus/minus 40 refer to inaccuracies in estimation of the half life (Someone else suggested it may be the variance or standard dev. of a decay waiting time)? I assume there is a better estimate for it today; is there a reference to a recent estimation which drops this error measurement? Thanks, --TeaDrinker 03:35, 12 October 2006 (UTC)[reply]

The National Nuclear Data Center says 5700 years plus or minus 30 years. That's the uncertainty in the measured value. All radioactive decays follow the same law, described by only one parameter; "the variance or standard dev. of a decay waiting time" sounds like nonsense to me. —Keenan Pepper 04:41, 12 October 2006 (UTC)[reply]
To explain why the uncertainty seems so bad: How do you do an experiment to measure the half life of 14C? You get a very pure sample of 14 (any 12C will contribute to the mass, but not beta decay, which throws off the measurement), completely cover it with semiconductor detectors (any solid angle not covered allows beta particles to escape uncounted), and then let it sit for a long, long time. There are lots of things that can introduce uncertainty. Also, beta decay is unpredictable because two particles escape: the beta particle (an electron) and a neutrino, which can carry away any fraction of the energy, from almost none, up to the whole amount less the rest energy of the electron (511 keV). If the neutrino carries away too much energy, the electron doesn't have enough to be detected, which adds more uncertainty. Taking all that into consideration, 5700 plus or minus 30 is quite a good value, the result of many careful experiments because 14C is such an important nuclide. For comparison, the half life of 93Mo is only known to be 4000 plus or minus 800 years. —Keenan Pepper 05:06, 12 October 2006 (UTC)[reply]

Thanks for the replies! --TeaDrinker 19:29, 12 October 2006 (UTC)[reply]

Critical molar volume of a van der Waals gas

According to Van der Waals equation#Reduced form, the critical molar volume of a van der Waals gas is exactly three times the parameter b which describes the volume of the particles themselves. The linear relationship is intuitively clear to me, but not the specific factor 3. What is special about a configuration in which the particles take up 1/3 of the available space? —Keenan Pepper 05:29, 12 October 2006 (UTC)[reply]

Solve: for a constant T. That's the critical state, and the critical v is 3*b. --BluePlatypus 05:51, 12 October 2006 (UTC)[reply]

I was hoping for something more intuitive... —Keenan Pepper 05:13, 13 October 2006 (UTC)[reply]

PROPOSED CHANGES TO THE REFERENCE DESK

If you haven't been paying attention to Wikipedia talk:Reference desk, you may not know that a few users are close to finishing a proposal (with a bot, now in testing and very close to completion) which, if approved by consensus, will be a major change for the Reference Desk.

Please read the preamble here, and I would appreciate if you signed your name after the preamble outlining how you feel about what we are thinking.

This notice has been temporarily announced on all of the current desks.  freshofftheufoΓΛĿЌ  06:57, 12 October 2006 (UTC)[reply]

For convenience, I propose any reactions to this anouncement be limited to Wikipedia:Reference_desk/Miscellaneous#PROPOSED_CHANGES_TO_THE_REFERENCE_DESK. DirkvdM 08:00, 12 October 2006 (UTC)[reply]

Equations for black body radiation

In the article Planck's law of black body radiation it is stated that:

and then that it can be expressed as a function of wavelength with:

Now in the second part, has been replaced by , which I can understand fine, but has been replaced by .

That's what I don't get. Surely but that leaves the equation short by a factor of .

What am I missing here? BigBlueFish 09:07, 12 October 2006 (UTC)[reply]

They aren't really the same u and technically its an abuse of notation to reuse it. The first, , is the spectral density per unit , while the second, , is the spectral density per unit . It follows therefore that they are related by a scale factor which is , which is where the comes from. Dragons flight 09:23, 12 October 2006 (UTC)[reply]
Ah, I think I see, it comes down to my misunderstanding of the actual concept of spectral intensity. What I'm actually trying to achieve is the amount of radiation emitted at a specific wavelength, or rather, it seems, within a narrow spectral band. To do this do I need to integrate the spectral intensity? And is it possible, or reliant on an approximation? BigBlueFish 12:01, 12 October 2006 (UTC)[reply]
The intensity (energy/time/area) per solid angle in a given wavelength range is just ; however, I know of no closed-form for this. If the interval is , of course, we have the standard results given in the article, but you said "narrow". If it's narrow enough (such that and ), you can profitably approximate the integral as , but that's just an extremely simple numerical integration, which is what you want in general. Many mathematics programs (including graphing calculators) can do this automatically. --Tardis 22:55, 12 October 2006 (UTC)[reply]

magnets

When you stroke an iron nail using the north-seeking pole, which ends of the iron nail will be the north-seeking pole?

(from > then stroke.)

Sign,Chan Hor Onn

A little unsure of the exact question. When you say 'north-seeking pole' I think you are talking about the north pole of the magnet, but not sure (the south pole will seek the north pole of another magnet and thus be north-seeking, but in the Earth's magnetic pole the north pole of a magnet seeks magnetic north, which I guess is what you mean; I think this may be an old term).
So, assuming you are talking about stroking with the north pole of a magnet, it should cause the domains in the nail to align in the direction of the stroke. In other words, if you stroke towards the point of the nail, the point should become the north pole. --jjron 13:23, 12 October 2006 (UTC)[reply]
'North-seeking pole' is a phrase sometimes used in place of the more familiar 'North pole', because that is the pole which will be attracted towards magnetic North. See Magnet#North-south pole designation and the Earth's magnetic field, which explains this, but doesn't actually introduce this term. --ColinFine 23:09, 12 October 2006 (UTC)[reply]
The Earth's North pole, is, of course, a south magnetic pole (or at least close to one). That is why the north pole of a magnet points toward it.Edison 13:31, 13 October 2006 (UTC)[reply]

LAser Eye Surgery

How can the laser used in Eye surgery be selectively destructive?(destroy unwanted eye tissue,neglect rest of tissue)???

I'm not exactly sure what you mean but have you read the LASIK article? Nil Einne 11:43, 12 October 2006 (UTC)[reply]
Think of a lens, which focuses light in one point. In order to burn a piece of paper with a lens and the Sun you have to hold it exacly in the focus area. Laser light has the nice capacity of not scattering (very much). A narrow beam will remain narrow. Let several harmless beams converge on one point and their combined power will have the same effect as on the aforementioned piece of paper. I don't actually know if laser surgery uses this, but I'm pretty sure it does.. DirkvdM 09:32, 13 October 2006 (UTC)[reply]

I'm searching for an inexpensive portable medical X-ray device, or are there any build-your-own kits?

Thursday, 10-12-06; Portland, OR; 2:27am West Coast Pacific Time

Is there an authority in the reference desk who can point me in the direction of an inexpensive portable x-ray device for personal medical use; or would it be more cost effective for me to build my own X-ray device from scratch, or from a mail-order kit - in which I could include radiation-protection safety features in its design?. If I had my own personal X-ray device, then I could save myself the travel time and waiting time for a physician's appointment. I wish to operate my personal medical X-ray device to take images, myself, of the physical condition of any of my own internal anatomy (specifically my bones, vertebrae, ligaments, and spinal discs.).

The webpages below describe various forms of X-ray devices - 1) This webpage describes Flouroscopy -

  http://en.wikipedia.org/wiki/Fluoroscopy

2)This webpage describes X-ray devices used for airport security -

  http://en.wikipedia.org/wiki/X-ray_machine#Security

3) This webpage lists various forms of X-ray methodologies -

  http://en.wikipedia.org/wiki/X-rays#See_also

My reason for listing the above specific webpages is to show the type of X-ray device that I'm searching for: real-time imaging.

--MyPresentCPUisTooSlow 10:36, 12 October 2006 (UTC)MyPresentCPUisTooSlow[reply]

You realize this sounds a little crazy right? Equipment for realtime imaging is likely to be prohibitively expensive. An xray machine of the kind that produces still images is probably less so, but in most jurisdictions radiation emitting machines and/or their operators must be licensed and inspected by the state. I dare say the hassles involved in acquiring, permitting and operating such an instrument will likely offset any hassle that going to a doctor's office brings. Dragons flight 10:55, 12 October 2006 (UTC)[reply]
Of course, you could operate one illegally but you'd probably end up killing yourself. And even if you didn't you'd just be wasting your money and time since you wouldn't actually learn anything from the images Nil Einne 11:41, 12 October 2006 (UTC)[reply]
I again, advise strongly against it. Ionizing radiation hazards, high voltages and currents, lack of medical knowledge. Cost-benefit ratio even if none of those apply is still pretty bad. — X [Mac Davis] (SUPERDESK|Help me improve)14:06, 12 October 2006 (UTC)[reply]
Haha, yes, this sounds crazy, but I think you're cool! Are you by any chance going on an expedition to the south pole? ;-) I mean, it could be a bit tricky to get to a hospital from there, especially if you're isolated over the winter. I hear some unorthodox procedures have taken place there. —Bromskloss 14:12, 12 October 2006 (UTC)[reply]

After you get the self x-ray working, then it's time for the self-surgery. You could start with removing your appendix, it's no good for anything anyway.. --Zeizmic 14:31, 12 October 2006 (UTC)[reply]

In any English-speaking jurisdiction I can think of, it will cost more to get the necessary permits and approvals – devices which generate ionizing radiation, particularly for medical purposes, are highly regulated – than you would save on paying for x-rays at a clinic. How often do you actually need an x-ray? I don't recommend doing them for fun.
Frankly, if you don't know enough about x-ray technology to know that building a home medical x-ray device is a really bad idea, then you don't know enough about x-ray technology to be able to build a home medical x-ray device. TenOfAllTrades(talk) 14:51, 12 October 2006 (UTC)[reply]

X-rays should only be taken rarely, like when you break your leg, and are relatively inexpensive. If you have so many X-rays taken that it would actually be cost effective to have your own X-ray machine, then you are having way too many taken. When you have that many taken, the risk of giving yourself cancer from all the radiation far outweighs the any benefit. Can you explain why you have so many X-rays taken ? StuRat 14:53, 12 October 2006 (UTC)[reply]

One clarification, the shielding on an X-ray machine is designed to shield the OPERATOR from X-ray radiation. There is no way to shield the PATIENT from X-rays and still get an image. This is why it's dangerous to have an excessive number of X-rays taken. StuRat 15:00, 12 October 2006 (UTC)[reply]

Anon, I have a used XRay machine I could part with in exchange for appropriate compensation. The radiation shield is broken but otherwise works. Sold as is, no refunds. Antonrojo 16:40, 12 October 2006 (UTC)[reply]
Coming soon: the sequel to the David Hahn article!
Atlant 17:39, 12 October 2006 (UTC)[reply]
Per the x-ray article, Thomas Edison "dropped X-ray research around 1903 after the death of Clarence Madison Dally, one of his glassblowers. Dally had a habit of testing X-ray tubes on his hands, and acquired a cancer in them so tenacious that both arms were amputated in a futile attempt to save his life." This might be a drawback to hobbyist home x-raying. Many of my generation remember those cool fluoroscope machines shoe stores had in the 1950's. http://www.orau.org/ptp/collection/shoefittingfluor/shoe.htm You could see if your shoes fit by looking down at the x-ray image of the shoes and your toes. It was tough when the government announced they caused cancer and yanked them. Edison 18:33, 12 October 2006 (UTC)[reply]
The weird thing is, why did it take them 50 years after Edison discovered that X-rays cause cancer for the government and shoe stores to figure it out ? StuRat 04:15, 13 October 2006 (UTC)[reply]
The shoe industry as well as the manufacturers made money with the machines. The theory was that the machine would be used only a few seconds at a time, a few times a year. In practice, I would use it every time I went to the store, because it was cool to see the bones in your feet. In the beginning, it was a way to make a few bucks with World War I surplus X-ray machines. It took until the late 1950's to finally get rid of them. Edison 04:22, 13 October 2006 (UTC)[reply]

The sale and operation of X-ray devices is tightly regulated by state law. In short, it would be impossible for you to own or operate an X-ray machine legally, unless you happen to be a doctor or a physicist. If you're curious, you can read Oregon's applicable laws here. —Brim 17:11, 14 October 2006 (UTC)[reply]

Try a friendly dentist who is upgrading his X ray machine, or try to find one of those things they used to use in shoe shops to see if your feet were still there when you put the shoes on. 8-) --Light current 18:17, 14 October 2006 (UTC)[reply]

Negative refractive index

I have heard that scientists have built substances that have negative refractive co-efficient.So why cant we build fibre optic cables out of them (total internal refraction from rarer medium instead of usual denser medium)?

From a quick look at Refractive index which links to Metamaterial, I would say it's probably because they're still largely theoretical with some limited demonstrations. They do appear to be of great interest for a variety of reasons and may be used for fibre optic cables in the future I guess if we can master them and produce them cheaply enough to be worth it but until then... Nil Einne 11:35, 12 October 2006 (UTC)[reply]
Actually I just thought of something. Will it actually work? Won't the light just never come out of your metamaterial at the other end? Nil Einne 11:39, 12 October 2006 (UTC) Ignore this, I wasn't thinking properly Nil Einne 11:20, 13 October 2006 (UTC)[reply]
It has been experimentally worked with before. Links: [7][8][9][10][11][12][13][14]X [Mac Davis] (SUPERDESK|Help me improve)14:03, 12 October 2006 (UTC)[reply]

Loop DNA

What are the functions of loop DNA?

Er what exactly do you mean loop DNA? Nil Einne 11:32, 12 October 2006 (UTC)[reply]
Are you maybe talking about DNA hairpin loops? Simon A. 12:15, 12 October 2006 (UTC)[reply]
Or maybe plasmids? Laïka 13:04, 12 October 2006 (UTC)[reply]
Or is it do your own homework? -- Plutor talk 23:24, 12 October 2006 (UTC)[reply]
Tell that to my friend, she said to me ask this question to Wikipedia because I don't know how to do it, so spare me the lecture
Plutor m:don't be a dick, what gave you the inclination that this was homework. Seems like a perfectly honest question to me. Philc TECI 17:59, 13 October 2006 (UTC)[reply]

Types of rock or stone?

What are the most common types of stone used for altars or megaliths in Europe?--Sonjaaa 14:09, 12 October 2006 (UTC)[reply]

There is such a wide variety of quarry stone all through Europe, that I doubt we can come up with anything. With all stonework, builders usually take what is close, because of transportation difficulties. That said, you might find Italian marble as a slight favourite, at least for thin facing. --Zeizmic 14:35, 12 October 2006 (UTC)[reply]
I'm not sure about "most common", but also see Sarsen. Avebury and Stonehenge are built (primarily) of this calcified silicified sandstone.
Atlant 17:45, 12 October 2006 (UTC)[reply]
I'm sure Atlant meant silicified!

It is a dense, hard rock created from sand bound by a silica cement, making it a kind of silicified sandstone.

But the answer has to be it will be the best local stone available because of transportation difficulties.--Light current 10:44, 16 October 2006 (UTC)[reply]

My short term memory apparently went "boink!" between the moments of reading the Sarsen article and typing this answer; thanks for the correction!
Atlant 14:43, 16 October 2006 (UTC)[reply]

Earth's moon: why it always shows the same face

How is it that our moon always shows the same face. How does the earth cause the rotation of the moon to stabilise in this way. Not too much mathematics, please!

Thank you

Tim

See Synchronous rotation, and the last sentence before see also will link you to the cause. Absolutely zero mathematics! Hyenaste (tell) 19:04, 12 October 2006 (UTC)[reply]
Or, you can just go directly to the tidal locking article. -- Plutor talk 23:16, 12 October 2006 (UTC)[reply]
Note that it doesn't always show exactly the same face. It wobbles somewhat and over a large enough period of time as much as 95% (I believe it was) of its surface can be seen from Earth. Thanks to this, sir Patrick Moore the presenter of The Sky at Night managed to see the mare orientale before the USSR could photograph it. DirkvdM 09:38, 13 October 2006 (UTC)[reply]
I believe the actual figure is 59%, due to Libration. --Jmeden2000 15:42, 13 October 2006 (UTC)[reply]
Oops, now there's a silly mistake. Right figures, wrong order. I already thought 95% was a bit too much. But I certainly thought it would be more than 59%, so maybe that caused the mistake. DirkvdM 09:05, 14 October 2006 (UTC)[reply]

Cooling Towers

Could someone explain why it is necessary to condense the steam at a power station, before heating it back up to steam for turning of the turbine? Why not just direct the steam straight back into the chamber in which it was heated in the first place and use less fuel? --Username132 (talk) 19:14, 12 October 2006 (UTC)[reply]

The phase transition is important. The gas takes up more space. The force of the expanding gas is what drives the turbine, see Steam engine. --JWSchmidt 19:37, 12 October 2006 (UTC)[reply]

The cooling isn't strictly necessary. You could vent the steam directly into the atmosphere, and this is the emergency option when the turbines suddenly have to be shut down. However, this is valuable demineralized water that you are throwing away and difficult to replace after a while. The concept of a condenser recycles this water and the excess heat is carried off with either hot air or hot lake/river water. --Zeizmic 20:15, 12 October 2006 (UTC)[reply]

Most of the heat that is absorbed by the water/steam goes into boiling the water. Raising the temperature of existing steam wouldn't absorb nearly as much heat energy, so much more total water would be needed. Being in steam form, this would take up a huge volume, versus the rather small volume of water needed in the current system. StuRat 23:55, 12 October 2006 (UTC)[reply]

Lower discharge pressure from a turbine or other steam engine means greater delta P which means more work per stroke. Work = force times distance, so zero pressure means greater work per stroke of a piston at a given boiler pressure than atmospheric would give. Edison 04:24, 13 October 2006 (UTC)[reply]

I assume the question is why waste the energy stored in the steam by condensing it. The answer is given by JWSchmidt, though maybe not explicitly enough. In order to get the expansion again you need to 're-shrink' the water first by letting it condense. DirkvdM 09:57, 13 October 2006 (UTC)[reply]

Thanks, I think I understand now. If you didn't have the condenser, then you wouldn't have a region of lower pressure for the steam to move to and therby drive the turbine. I just don't like that 60% or more fuel is wasted in this way. I just checked out the article on Combined heat and power and found this boiler thing which generates electricity as it boils your water. But it only runs on gas and not coal or nuclear fuel. --Username132 (talk) 14:44, 13 October 2006 (UTC)[reply]

And most of our energy production still uses this ancient technique. Even nuclear plants do. Finding a more efficient way to transfer heat to electricity (or maybe hydrogen or other energy carriers) would be a major step forward for mankind. Is there maybe some physical reason one can't achieve a high efficiency? Are maybe the forms of energy too different? DirkvdM 09:09, 14 October 2006 (UTC)[reply]
In practice, the main alternative for turning heat into electricity - the thermocouple - is way, way, way less efficient. They use them in radioisotope thermal generators but there are proposals to replace them there with Sterling engines because they are just so wasteful.
Also, Rankine cycle steam turbines have gotten a lot more efficient over the past few decades. You used to get about 33-34% thermal efficiency with conventional turbines. These days, the very best supercritical turbines in the latest coal-fired power stations get about 50% efficiency. But even that's not the best you can do. If you gasify the coal and run it through a combined cycle gas turbine, you can now get over 60% thermal efficiency. Even fuel cells find it hard to beat that number in practice. --Robert Merkel 00:30, 15 October 2006 (UTC)[reply]
That's still not quite impressive. I suppose teh reaason fuel cells are also that inefficient in practise is that only a fraction of the research effort has gone into developing them, partly because they are a much more recent invention. The article doesn't say what efficiency they might theoretically reach, but if they already compete with the traditional system, there is good potential. DirkvdM 08:13, 15 October 2006 (UTC)[reply]

Ballistics

It requires some energy to discharge a spent casing from a semi-automatic or automatic gun/rifle. This energy HAS TO come from the hot gases that propel the bullet. Can anyone tell me about how much velocity is "lost", for a given bullet, that would otherwise NOT be lost if same bullet was fired from a single-shot rifle? Loss might be slight, but there certainly HAS to be SOME loss. I thank you ahead of time if you can help. 205.188.116.74 19:35, 12 October 2006 (UTC)[reply]

It depends entirely on the type of round and the gun it is fired from. For example, an M16 has a spring buffer that is pushed back by a gas tube. Gas fills that tube as the bullet leaves the barrel - so the bullet is beyond being affected by the gun. The spring buffer retracts after the gas tube fills, popping out the expended round, and loading the next round. --Kainaw (talk) 19:46, 12 October 2006 (UTC)[reply]

Even with an old Lee-Enfield single shot, a lot of energy goes into mashing your shoulder. --Zeizmic 21:01, 12 October 2006 (UTC)[reply]

I believe the gases are in the chamber as the bullet is in the barrel. so you do get some loss of gas that would be otherwise be pushing the bullet, but only a small portion is used this way. weapons like the Steyr AUG (i imagine its pretty standard on other weapons too) have an adjustable gas port so you can use more gas if the mechanism is gunked up. Xcomradex 21:12, 12 October 2006 (UTC)[reply]
Note that the revolutionary thing about the AK47 was that it used the power of the fired bullet to drive a very simple mechanism to not only discharge the spent casing but also load the next bullet. I think. Heard somthing like that once, but I don't know much about guns, so some details may be wrong. DirkvdM 10:02, 13 October 2006 (UTC)[reply]
Well that's what gas-operated reloading is. Don't think the Kalishnikov was a pioneer though. — X [Mac Davis] (SUPERDESK|Help me improve)04:16, 15 October 2006 (UTC)[reply]
It wasn't? That's new. DirkvdM 08:18, 15 October 2006 (UTC)[reply]
Definately not new in the Ak-47. The german WWII era StG-44 has it for example, as did the M1 Garand. Xcomradex 10:21, 15 October 2006 (UTC)[reply]
Ah, that aspect of it. I thought he meant the AK47 as a whole. That would be a bit of an odd thing to say about a mechanical military weapon that is still extremely popular after 60 years. DirkvdM 06:53, 16 October 2006 (UTC)[reply]

Ants

I just ordered one of those NASA-gel Antworks ant farms , and have a few questions about the ants themselves. I think it would be very nice if the colony of ants were able to reproduce so that I could follow their progress over the years. However, I'm not sure if that is possible with the kind of ants one receives from the order form (p 18-19), which is for 25 all-female ants.

Will this group of ants be able to reproduce, and, if not, where could I order ants that could? I would assume both a queen and a male are necessary, and perhaps a bigger environment than a regular ant farm could provide. --JianLi 05:00, 13 October 2006 (UTC)[reply]

No, they won't be able to reproduce. Considering how easy it is to collect ants, do you really need to breed them in captivity ? StuRat 20:11, 14 October 2006 (UTC)[reply]
Well, I dislike having to collect ants not because it would be hard (though, since winter is coming, it actually might be), but because I feel that this provides an incomplete picture of their world. Apart from studying ant reproduction, it would be nice to have a feeling of continuity; in a few years, I could the satisfaction of having a group of ants whose ancestors I knew, or perhaps I could pass my ants onto my grandchildren, for example.
Unfortunately, it seems, it is illegal to mail live queen ants in the US. [15] [16] --JianLi 22:46, 14 October 2006 (UTC)[reply]
Wait until summer, then excavate an ant hill and take the queen along with all the rest. StuRat 21:01, 15 October 2006 (UTC)[reply]

Goldfish

On a related note, when I had goldfish, I was not able to get them to reproduce (though I didn't really try: there were only two of them, and I didn't even ascertain if they were of opposite gender). Are there any specific conditions one has to maintain for goldfish reproduction? I merely keep them in a bowl without any fancy electric filtering, etc, and I empty the water regularly. Wouldn't the eggs be thrown out with the water if I did this?

Thanks, --JianLi 21:58, 12 October 2006 (UTC)[reply]

Asking why they don't reproduce is a bit like asking why you and some random person of the opposite sex don't reproduce when left alone in a room. In short, most species use some type of mate selection process and the chances of any two random individuals meeting each other's criteria is slim. Even then, a very specific environment is needed for them to reproduce. It's really amazing that any animals reproduce in captivity. StuRat 23:45, 12 October 2006 (UTC)[reply]
Goldfish eggs (well, at least "Comet" eggs) are small spheres about 1mm in diameter. They are adhered (by the fish) to various suitable surfaces in the habitat. We have a school of comets that move to a small outdoor pond during the summer. Last year, our comets were "frisky" while outdoors and we used to find a lot of eggs adhered onto the filter/pump and occasionally on the water plants. We carefully separated any eggs we found and put the surviving eggs into a small fishbowl. About 1/3 of the collected eggs hatched out and we eventually raised about a half-dozen new fish "from egg". (A few fish were lost to cannibalism!) The hatchlings are very interesting, looking like nothing more than tiny sticks with eyes. (We used a jeweler's loupe to observe them, both in egg and afterwards.) Later, upon moving the rest of the adult school back indoors, we found two more fry that had hatched and survived outdoors without our "help".
The fish seemed to be friskier when the pond was relatively green with algae. Better privacy? Concern they were going to die? Who knows! But this year, for whatever reason, we don't think the comets were breeding. We never saw any eggs, and no particularly small fish came back in this fall. Or maybe the neighborhood cat was umm, err...
Atlant 01:16, 13 October 2006 (UTC)[reply]
I suspect the problem was your goldfish weren't homosexual so they didn't like each other Nil Einne 11:16, 13 October 2006 (UTC)[reply]
That's a nice fish story :). I guess I should at least get more fish and a bigger bowl to increase the chances. --JianLi 22:51, 14 October 2006 (UTC)[reply]
You may want to check on fish-breeding tips concerning your species.[17] I think I read somewhere on here about some species rarely mating when in captivity. Maybe someplace else, maybe not true. — X [Mac Davis] (SUPERDESK|Help me improve)04:20, 15 October 2006 (UTC)[reply]

brain

why do you want to use the right side of your brain more in art class than your left side ??

One side is artistic, and the other side is logical. StuRat 23:50, 12 October 2006 (UTC)[reply]
But, of course, in some people, neither side is logical 8-)--Light current 00:42, 13 October 2006 (UTC)[reply]
The known lateralizations are trends and do not apply to every person in every case. --JWSchmidt 02:29, 13 October 2006 (UTC)[reply]
And, you knwo, so far I have never actively taken care to switch on the correct side of my brain when trying to do something. Simon A. 09:08, 13 October 2006 (UTC)[reply]
I wouldn't know how to control that either. My brain probably does it all by it self. It seems to have a mind of its own. DirkvdM 10:06, 13 October 2006 (UTC)[reply]
Both sides of your brain refuse to work equally. :-) StuRat 22:11, 13 October 2006 (UTC)[reply]
Of course they do. What, did you think I had a communist brain? DirkvdM 09:13, 14 October 2006 (UTC)[reply]

October 13

Chromosomes

Chromosomes are not generally observed in nondividing cells. Does this mean that they are newly formed for each cell division?

Nope. It's not that they are not present in nondividing cells, but rather that they cannot be observed. When not replicating, they are not neatly arrayed and separated from each other, so all one sees is a non-distinct nuclear blob of "all the chromosomes together". There's an overview explaining it in the intro to the chromosome article. DMacks 00:40, 13 October 2006 (UTC)[reply]

Threat to human race

What do editors think is the biggest currently percieved threat to the survival of the human race and why?--Light current 00:41, 13 October 2006 (UTC)[reply]

Bad spelling? Clarityfiend 00:42, 13 October 2006 (UTC)[reply]

Yes I know I cant spell!--Light current 00:45, 13 October 2006 (UTC)[reply]

Without trying to be too flippant - ourselves? People always seem to find reasons to want to kill other people. One day, it'll probably all go too far and someone will start something *very* nasty that no-one can stop. --Kurt Shaped Box 00:52, 13 October 2006 (UTC)[reply]
I vote for global worming. A big enough temperature change could destabilize the ecology and trigger an unstoppable chain reaction of species extinction. (Damn those hot giant underground worms.) Clarityfiend 01:01, 13 October 2006 (UTC)[reply]

Bullets kill, but it takes basically a trigger pull per death. Retail killing, little better than bare hands assault. Nukes kill faster, but the effect is geographically and temporally limited. Wholesale killing. For real extermination of the human race, think of biological weapons, the Gift that keeps on killing. Self-replicating killer nanomachines are also a plausible method for the demise of the human race, as is nudging a large asteroid to a collision course with Earth. The best case would be the replacement of the human race by the next phase of hominid evolution, just as humans theoretically replaced neanderthals. Edison 04:31, 13 October 2006 (UTC)[reply]

Climate change is by far the biggest clear and present danger to mankind. But not to its survival. Over the next century or so, millions if not billions of people will die, even if we stop producing greenhouse gasse now (I'm not kidding). But some people will always survive somewhere. It would take something even bigger than that. Forget about nuclear weapons, they're nasty but not big enough. An asteroid impact (caused by man or not) would have to be pretty big to wipe us all out (we're pretty resourcefull). 'Something nano' is probably the most likely cause of our extinction, if that is to happen in the near future. Like a very infectious lethal disease with a very long incubation period. But it would have to be lethal to everyone, which is rare. Genetic engineering could make something more alien our bodies don't have an answer to (made intentionally or by accident - both are an equally serious threat). But even that is biological and some might survive. Self-replicating adaptive nano killing machines (a hardware version of intelligent computer viruses) might be harder to find an answer to and are something we could probably build within a century (less even). Nano engineering isn't as serious a threat as genetic engineering because it is not as (commercially) developed yet. But that time will come. My own thoughts are starting to scare me now. DirkvdM 10:21, 13 October 2006 (UTC)[reply]
About the nuclear boms, let me rephrase that. The total arsenal would be big enough if it was employed with that single goal, but in reality it would be used by competing forces targeting only military targets, leaving enough room to hide. For some.... DirkvdM 11:47, 13 October 2006 (UTC)[reply]
You are assuming of course the humans that do survive will not be killed off by the nuclear winters and the radioactivity and the like after the bombing stops. It also depends on your viewpoint of what leaders will do. It is my opinion anumber of leaders if they feel their country is facing eminent destruction will say to hell with it and not just target military targets but try to destroy as many people as possible. I'm not saying I think this is likely to happen but it could Nil Einne 12:20, 13 October 2006 (UTC)[reply]
I don't think that an atomic war would be capable of killing off *all* humans. Most of us, certainly - but there will be a select few who saw it coming (or at least figured that 'something bad is going to happen one day') and made preparations. I do get a feeling that one day, those 'crazy survivalists' that documentary makers like to sneer at will end up having the last laugh. I don't fancy the chances of the world leaders after they emerge from the shelters when the whole thing is over - coming up for air and being faced by mobs of *very, very* angry and most likley heavily-armed (post-apocalyptic man will be a hunter/gatherer/farmer and he *will* have a gun) people... --Kurt Shaped Box 00:14, 15 October 2006 (UTC)[reply]
I think the biggest threat is to the survival of humans is humans. A quick look at this page should prove that to you. I'm actually quite surprised we made it this far... Nil Einne 11:14, 13 October 2006 (UTC)[reply]
There have been sci-fi scenarios, such as H.G. Wells The Time Machine (and movies adapted from it) in which nuclear war drives humans underground to escape the radiation. They have an underground civilization, perhaps with hydroponic agriculture and nuclear power, and survive indefinitely without blue skies and sunshine, perhaps mutating into Morlocks. The human race theoretically consisted of a few thousand people at the smallest, before modern humans expanded their range and replaced the neanderthals. With some radiation exposure and a small population, rapid evolution would be likely.Edison 13:40, 13 October 2006 (UTC)[reply]
I think humans are the biggest danger to themselves. Philc TECI 17:47, 13 October 2006 (UTC)[reply]

Well my personal opinion is that its something beyond human control that will wipe everything out instantaneously. I can think of only one thing that would do it 8-)--Light current 02:33, 15 October 2006 (UTC)[reply]

The gulls? ;) --Kurt Shaped Box 02:35, 15 October 2006 (UTC)[reply]

I really can't think of one at all. And agreeing with Dirk (hehe), humans are damn good weeds and we won't go out easily or quietly. — X [Mac Davis] (SUPERDESK|Help me improve)04:23, 15 October 2006 (UTC)[reply]

A professor from Stanford mulls them all over here.[18]X [Mac Davis] (SUPERDESK|Help me improve)07:42, 17 October 2006 (UTC)[reply]

Looking for medical term...

What is the actual medical term for the situation where someone bursts an artery in their brain whilst pushing too hard on the toilet when constipated? Anyone know? C'mon - doctors must've given this one an offical name... --Kurt Shaped Box 01:05, 13 October 2006 (UTC)[reply]

Drain Brain Strain. Anchoress 01:07, 13 October 2006 (UTC)[reply]
A stroke of bad luck? Or maybe just hard shit! 8-)--Light current 01:14, 13 October 2006 (UTC)[reply]
"You had a stroke while sitting on the can? Tough shit!" DMacks 01:28, 13 October 2006 (UTC)[reply]
Is there an echo box under the Ref desk? I could have sworn I just said that! 8-|--Light current 13:02, 13 October 2006 (UTC)[reply]
An aneurysm? Not necesarily the brain and not necesarily on teh tolit either, but does meet both criteria at times. Hyenaste (tell) 01:28, 13 October 2006 (UTC)[reply]
This cause of death was featured in the X-Files episode War of the Coprophages. --JWSchmidt 02:39, 13 October 2006 (UTC)[reply]
The Elvis effect.Edison 04:33, 13 October 2006 (UTC)[reply]

There is no specific name that I know of. If you find a name, it is likely to be an attempt at humor and not truly in medical usage. InvictaHOG 09:36, 13 October 2006 (UTC)[reply]

The lesson is of course to eat lots of fibre and cut out the sat fats. You then reduce the chances of stroke etc. 8-)--Light current 12:59, 13 October 2006 (UTC)[reply]
Could be called a minor brain scanning event 8-)--Light current 17:38, 13 October 2006 (UTC)[reply]
See Valsalva maneuver.--Mark Bornfeld DDS 21:30, 13 October 2006 (UTC)[reply]
It is a cerebral aneurysm you are thinking of. Unspecific to constipation though. — X [Mac Davis] (SUPERDESK|Help me improve)04:28, 15 October 2006 (UTC)[reply]

Oxygen

What happens when Oxygen is burnt(chemically)? Pure oxygen on it's own (O2 i assume...) is very flammable (like in Oxygen tank for medical or scuba use). When It is burned what does it form? I read the Oxygen article and it didn't give me much help. Can anyone else tell me or point me in the right direction? --Agester 01:20, 13 October 2006 (UTC)[reply]

Pure oxygen is not flammable in the usual sense of the word. Rather, it makes it very easy for other things to burn--burning as we usually experience it is a reaction in which the thing that's burning is combining with oxygen. DMacks 01:23, 13 October 2006 (UTC)[reply]
Im not a chemist, but I would think that oxygen has to oxidise something. So its the 'something' that is burnt , not the oxygen. 8-|--Light current 01:24, 13 October 2006 (UTC)[reply]
Oxygen by itself does not burn. Instead, it is a potent oxidizing agent that will readily combine with other materials; that chemical reaction is what we usually refer to as fire when it's fast, but as oxidation or even good old rusting when it's slow.
Atlant 01:26, 13 October 2006 (UTC)[reply]

For oxygen to burn, it would have to combine rapidly with oxygen. Well, it is already oxygen, so it has no urgent need to combine with itself. Thus oxygen doesn't burn, but flammablke substances will indeed burn rapidly in the presence of oxygen. Edison 04:35, 13 October 2006 (UTC)[reply]

The most common things we 'burn' would be carbon and hydrogen in various forms, carbohydrates(like wood and paper) and many various hydrocarbons. When these substances burn in oxygen (which is very different from "air"), the most common by-products formed are ordinary water "dihydrogen monoxide ;)", carbon dioxide, carbon monoxide.
This is not so much a faq but a frequently given answer (fga): fire requires three things: fuel, oxygen and heat. Having said that, ozone is formed by oxygen reacting with itself. I don't think that can be called 'burning', though. DirkvdM 11:56, 13 October 2006 (UTC)[reply]
The pedant in me notes that the requirement is an oxidizer, not necessarily oxygen, though oxygen is by far the most common one in most people's everyday experience. I wonder if O2+2F2→2OF2 could be reacted directly and to give a visible effect enough to be considered "burning" and not just "oxidation of oxygen". DMacks 12:34, 13 October 2006 (UTC)[reply]

If I recall correctly Scientific American once (decades ago) had an amateur scientist column on building apparatus where "inverse" flames burn, in the sense that a small glass tip was used to introduce oxygen into a container of flammable gas, where the oxygen appeared to burn like a gas jet does in air.(Don't try this at home, since explosions would result if too much of the mixture combined unburned before ignition). The point is, that in that demonstration, it did appear that the oxygen was "burning" in the sense that heat and light were given off at a steady rate when it was introduced into the gas and a spark or ignitor lit it. Edison 13:46, 13 October 2006 (UTC)[reply]

You recall correctly.
Atlant 14:13, 13 October 2006 (UTC)[reply]

(sorry if i'm incorrect somewhere i'm still a student!) So oxygen we do know is highly flammable when it is with other substances(not chemically combined yet!)! But what i'm having trouble understanding is: oxygen on it's own isn't flammable?? --Agester 19:25, 13 October 2006 (UTC)[reply]

Let's put it this way: steak and fish are highly edible substances. They won't be consumed spontaneously, but combining them with your cat may result in the steak or fish being consumed. This does not mean that your cat is highly edible, but you'd better be careful with edible substances that you want to keep in the presence of your cat (and the other way around – it's really the combination that is riskly). Now read steak = cotton, fish = paper, edible = flammable, your cat = oxygen.  --LambiamTalk 20:33, 13 October 2006 (UTC)[reply]
Oxygen is flammable in a reducing atmosphere (say, methane). The oxygen and the methane combine to produce the flame; there's no obvious reason for preferring to say it's the methane burning, rather than the oxygen. But no, oxygen on its own is not flammable (the 3O2→2O3 reaction is endothermic, I think), unless you're talking about nuclear combustion (fusion), which requires extremely high temperature and pressure. --Trovatore 21:01, 13 October 2006 (UTC)[reply]

haha trovatore that's why i specifically mentioned chemically in my posts 8) but thanks for the feedback lambiam that certain helps a bunch! thanks to everyone for their input!--Agester 00:58, 14 October 2006 (UTC)[reply]

Ah, that answers my question. But can I reverse that? Is 2O3 → 3O2 therefore exothermic and can it therefore be called 'burning'? After all, ozone is a form of oxygen, right? DirkvdM 09:21, 14 October 2006 (UTC)[reply]
Yes, it's exothermic. Under normal conditions, ozone decomposes slowly to O2. But I just googled for an MSDS for ozone, an there, it says "At elevated temperatures and in the presence of certain catalysts such as hydrogen, iron, copper and chromium, this decomposition may be explosive." Simon A. 16:52, 14 October 2006 (UTC)[reply]
Wow, oxygen can be explosive. Don't tell any terrorists. :) DirkvdM 08:21, 15 October 2006 (UTC)[reply]
It can be toxic too... if i read correctly --Agester 12:31, 17 October 2006 (UTC)[reply]

Alpha level and power for a hypothesis

Hi:

I am sitting on my desk, trying to figure out the following statistics problem: if the alpha level increases power also does. Now, I wonder whether power can be less than the alpha level for a hypothesis test. I think it cannot be less as they are somewhat related and depend on each other. But I am not 100% sure. Does anyone know anything that would illuminate my mind and refresh my brain cells a little? I would be thrilled. Thanks much.Hersheysextra 02:00, 13 October 2006 (UTC)[reply]

Think of f(x)=mx+b. x is the alpha level. — X [Mac Davis] (SUPERDESK|Help me improve)04:33, 15 October 2006 (UTC)[reply]

Stored energy in tempered glass pellets

I threw a squiggly piece of metal at an oven today. The tempered glass shattered on impact, but when I went closer for inspection, I heard and saw the fragments continuing to break apart for minutes after the glass pane initially broke. The glass pellets were actually breaking apart with enough force to send them several cms in opposite directions. How does tempered glass manage to hold that much energy for so long? Hyenaste (tell) 03:01, 13 October 2006 (UTC)[reply]

The article has a pretty good description, although a diagram would be nice. The outside of the glass solidifies first, leaving too much room for the inside, and then the inside solidifies and contracts, pulling the outside inward. Think of it as a rigid box with a bunch of springs inside, under tension. The springs can't contract because they're attached to the rigid box. Then when you break the box, the springs contract and release their stored energy. See also Prince Rupert's Drop. —Keenan Pepper 05:34, 13 October 2006 (UTC)[reply]
Mystery writer Dick Francis wrote "Shattered" (2000) ISBN 0-399-14660-1 the plot of which focuses on the dangers of improper annealing of masses of glass.Edison 13:54, 13 October 2006 (UTC)[reply]

One day I shut the door on my old van, and the rear glass window completely shattered. For a second I thought there was a shooter! Apparently this was happening with all the vans of the same make and vintage. --Zeizmic 14:17, 13 October 2006 (UTC)[reply]

I was building something with a glass plate in it. The glass was on two hinges to act as a door. After I screwed everything in and such I tapped the glass with my screwdriver and it exploded into a million tiny little pieces all over me. To my astonishment some of the million little pieces just kept jumping. I guess the hinges were too close together!! There was so much compressive stress that my little tap sent it over the edge. — X [Mac Davis] (SUPERDESK|Help me improve)04:39, 15 October 2006 (UTC)[reply]

Microbiology math?

Would a course in pre-calculus be important for a microbiologist? 129.15.131.247 03:38, 13 October 2006 (UTC)Razma Dreizehn[reply]

Most post-secondary institutions require it, or some other math. It can't hurt to have some basic mathematical knowledge. -- Consumed Crustacean (talk) 03:51, 13 October 2006 (UTC)[reply]
Yes, definitely. Calculus is indispensable for all research in natural sciences. Whether it is of use to already aquire some knowledge before starting your university courses, or whether you may depend on them to lead you gently enough into the subject in the introductory courses, depends on the school, of course. But most freshmen in science and engineering subjects notice that in the first year, the math courses are the hardest and show the steepest increase in difficulty as compared to high school, and hence, being prepared may make your live much easier. (Note: I am German, but I assume this advice holds for most countries.) Simon A. 09:20, 13 October 2006 (UTC)[reply]
For microbiology, I'd recommend studying some statistics; stats is probably the most useful field of maths in Biology, as although you can never work out mathematically exactly what an organism will do, perfoming something like a Chi-square test or the Pearson product-moment correlation coefficient can allow you to find the likelyhood that something is related (such as the death of bacteria when exposed to different temperatures), rather than just a random occurance, very accurately. Laïka 18:25, 13 October 2006 (UTC)[reply]
Don't forget that pre-calculus and calculus cover things like difference equations and differential equations which can be used to model, for example, organism populations. Confusing Manifestation 02:40, 14 October 2006 (UTC)[reply]

Education

I am a doing my MA Education in a pakistani international university.The teacher has assigne me to write on the topic of "Mental heath of the teacher",for which i search many web site but found nothing. Now i requesting you to send me on the tpic of "Mental Health of the Teacher". Thanks in anticipation and best regards. (Muhammad Hussain)

Generally speaking, when doing research for postgraduate courses (actually even normal Bachelor's level courses) your expect to rely on primary sources i.e. journals and the like. Perhaps some good books etc as well from the library. Websites are usually a poor source. For a specialised topic like the one you mentioned, it will definitely be the case that you should be relying on journals and books not websites. BTW, I personally suspect the mental health of your teacher is rather poor if he/she has to put up with students doing Masters who think they can rely on websites for their essays/assignments Nil Einne 11:12, 13 October 2006 (UTC)[reply]
I suggest that you start with this question: what is special about the mental health of university professors that they would be singled out as a topic for study? You've found what I'd suspect, that there is little if any research focusing specifically on professors. One good place to start would be Occupational Psychology and studies such as these and this will allow you to evaluate instructors mental health relative to other professions. For example, you might ask: 'is the work of university teaching more or less stressful than other occupation?' and 'what positive mental health benefits does teaching provide?'. If you're really feeling ambitious, you could design or locate mental health questionnaires and ask professors to complete them, possibly comparing them to another occupational group. Learning how to conduct your own research is an essential skill in most graduate programs. Antonrojo 12:51, 13 October 2006 (UTC)[reply]
I'm not sure but I think he mean's school teachers not university professors but your post mostly still applies. BTW I would assume there must be at least some studies on the mental health of school teachers Nil Einne 13:08, 13 October 2006 (UTC)[reply]
I certainly had several who were nuts, and I probably did my share of harm to the sanity of others. One area of teacher sanity to look at would be the wierdness of teachers in their 20 or 20's who throw away their career to have sexual escapades with young teenage students. The teachers' mental health or lack thereof is often presented as an excuse at the trial.Edison 14:21, 13 October 2006 (UTC)[reply]

a short brief essay

Hi, I am a student and for one of my major essay projects-I chose 'is HPPA useful ,has it changed things much....a link to an article from a medical journal or any other reliable source talking about HIPPA's shortcomings or merits and demerits would be of immense help...I'd appreciate any help, thanks

So the purporse of this essay project was for you to learn how to get other people to find references for you? What course is this? "Users 101"? I'll tell you what I'll write your whole essay for you, see below:
I'm very lazy and can't be bothered doing the work that is expected of me. Please give me an F
That's it. The essay will be enough for an A+ I guarantee it. Nil Einne 11:06, 13 October 2006 (UTC)[reply]

This might be one starting point and oddly typos seem to help in this case. Antonrojo 12:54, 13 October 2006 (UTC)[reply]

Don't forget this too... Nil Einne 13:06, 13 October 2006 (UTC)[reply]

Double-slit experiment aiming

I find it incredible that single electrons on their own have the statistical probabilities of waves (weirdly phrased, but bear with me), but in the entire of Double-slit experiment I can't find out, why isn't it possible to perform this kind of experiment with anything bigger than a proton or an electron? 213.161.190.228 08:42, 13 October 2006 (UTC)[reply]

I think it might be, but the wavelength of bigger things is smaller so the interference pattern gets so compressed (to a small area) that you can't really see it's there anymore… I think. —Bromskloss 09:54, 13 October 2006 (UTC)[reply]
I don't see it in the article, but I remember one of my physics professors in college saying they'd managed to measure the interference pattern with helium. Or maybe it was hydrogen. But it was definitely an atom. --Allen 12:12, 13 October 2006 (UTC)[reply]
Here ya go: Carnal, O. (1991). "Young's double-slit experiment with atoms: A simple atom interferometer". Phys. Rev. Lett. 66 (21): 2689–2692. {{cite journal}}: Unknown parameter |coauthors= ignored (|author= suggested) (help) DMacks 15:12, 13 October 2006 (UTC)[reply]
OH, now I found it! The largest theoretically possible is just under the size/weight of a large bacterium, but should be very difficult to find out. Just scan the related Wave-particle duality for "bacterium", and it'll jump to it. Should've searched before asking. :) Thanks! (this is the original poster) 81.93.102.3 15:59, 13 October 2006 (UTC)[reply]
this[19] probably interests you too, it has been achieved with buckyballs. Xcomradex 23:17, 14 October 2006 (UTC)[reply]

How does the transferrin and ferritin be cleared in human body, respectively?

== How does the transferrin and ferritin be cleared in human body, respectively? Thank you. ==

196.200.9.253 14:05, 16 October 2006 (UTC)== Chemical Engr/Production and application of CaCO.3 filled PVC ==[reply]

Hi,I'm a student of chemical Engineering & ve been working on the production and application of CaCO.3 filled PVC.I've done home work and got some knowledge,that inorganic CaCO.3 filler and particulate filled polymer are blended together;

  • because CaCO.3 reinforce polymer(PVC)
  • CaCO.3 has a toughening effect in the PVC/CaCO.3 binary composite

And they most applicable in,

  • Coating/car underbody painting
  • Plastics
  • Rubber.

So,please i don't know if can get some help on the following;

  • what is the method/procedure of production of CaCO.3 filled PVC
  • What other application do we ve apart from that mentioned above.
  • What are the effect of CaCO.3 blended with PVC

Please can i get a name of a reference material/textbook to me in my study for this topic

  • What are the reason and advantages for such blend of CaCO.3 filled PVC.

Please I need your help,and will be waiting for your reply,till then thanks for your kind gesture and assistance.196.200.9.253 14:39, 16 October 2006 (UTC)[reply]

I fixed your post to make it more readable. Also please Wikipedia:Sign your posts with 4 tidles (~~~~) Nil Einne 11:22, 13 October 2006 (UTC)[reply]

Flu shot

I wonder if there is data concerning the time between getting a flu shot and being immuned (partly, as I understand) by it.

I assume you mean an influenza vaccine? I don't know specifically but it would be within a few days (to reach maximum immunity) I would suspect. Do you know how vaccines work? Nil Einne 12:14, 13 October 2006 (UTC)[reply]

definition of the word "plaxing"

I am trying to help a friend out with her child's vocabulary lesson at school. The teacher has raised the question as to the meaning of the word "plaxing" in relation to computers. I have not found any clear definitions or usages on any site. Does anyone have a definition and/or usage for this word?

Given the Google results, I'm doubtful whether plaxing is a word. Are you sure you got the word right? Nil Einne 13:04, 13 October 2006 (UTC)[reply]
Plaxing is, of course, the process of fulmagrating the tardyons in the central time and space dilator module of the Tardis. I thought everyone knew that. 8-)--Light current 13:16, 13 October 2006 (UTC)[reply]
Plaxing has something to with gambling. Google found plaxing poker, plaxing blackjack etc - Wikicheng 13:36, 13 October 2006 (UTC)[reply]

Multiplexing--Light current 15:17, 13 October 2006 (UTC)[reply]

I belive the spelling is correct as the teacher relayed it. In order to confirm, I will have to wait until next week's class. I do really appreciate all of responses at this point.

Urban dict. sez "The usher didn't notice the kids plexing, because he was looking at an attractive girl while they snuck ... If you plexing wit me you aint havin good luck". More senses on any search eng. -- DLL .. T 19:41, 13 October 2006 (UTC)[reply]
All examples I see of "plaxing" using Google search are typos for "playing" or "placing". My conclusion is that this is an unknown word, in relation to computers as well as otherwise.  --LambiamTalk 22:18, 13 October 2006 (UTC)[reply]

Black Hole

In wikipedia, under the paragraph "Recent Discoveries" we find a black hole discovered which is about 12.7 billion light years away which means it "really" existed 12.7 billion years ago.But the alleged Big Bang started about 15 billion years ago which shortens the life of the star to only 2.3 billion years.But we know a star should be highly massive and should be atleast 10 billion years to form a black hole which means the is an error with the dating of the big bang.Please give me an explainaton.

Highly massive stars actually have shorter lifespans than smaller stars. (Their greater mass and consequently stronger gravity cause them to compress and burn fuel much more rapidly.) Very massive stars (tens of solar masses and up) will exhaust their fuel in millions of years, rather than billions. Hope that clears things up. TenOfAllTrades(talk) 14:33, 13 October 2006 (UTC)[reply]
You expect a bit much of us Wikipedians. The black hole article says: In June 2004 astronomers found a super-massive black hole, Q0906+6930, at the centre of a distant galaxy about 12.7 billion light years away. This observation indicated rapid creation of super-massive black holes in the early universe. [6] And in the article of reference [6], they say A team of astronomers have found a colossal black hole so ancient, they're not sure how it had enough time to grow to its current size, about 10 billion times the mass of the Sun., asking the same question as you did. The discovery is recent and surprising, and the obviuos answer to your qeustion is: We don't know yet. That's why the discovery is so interesting. However, the hypothesis that some black holes have not formed from a star but already during the big bang is discussed a lot and we even have an article on this: primordial black hole. Simon A. 14:47, 13 October 2006 (UTC)[reply]
The article Age of the universe lists the best current estimate as 13.7 ± 0.2 billion years, making the universe even considerably younger at the supposed moment of black hole formation.  --LambiamTalk 18:19, 13 October 2006 (UTC)[reply]
Yeh but the point was that black holes dont just appear, a star has to die first, and woould a star have had enough time to live and die in the time given, considering its huge mass. I dont know the answer myself, but you may (or may not) have missed the questioners point. Philc TECI 19:18, 13 October 2006 (UTC)[reply]
Supermassive black holes can, according to present theory, also result from a large gas cloud collapsing into a star of supersized mass, like wow. The star is unstable and may collapse directly into a black hole without going through the usual process ending with a supernova.  --LambiamTalk 23:15, 13 October 2006 (UTC)[reply]

Food pyramid

you know you have to eat 6 servings or more of carbohydrate foods such as bread a day, up to 3-5 servings of vegetables a day and 2 recomended servings of meat and alternatives each day. But how can you tell how much is each serving? for bread i know a slice is a serving, but what about a bowl of rice? there's different sized bowls for carrying rice.I asking this because i think i'm overeating. And also when it says two recomended servings does that mean i should only eat two servings of meat per day?

A serving of rice is 3/4 cup prepared. Most foods you buy at the grocery store have nutritional guides on the side somewhere. But isn't the food pyramid currently considered a poor guide to nutrition? If you're considering losing weight/keeping weight off, you might want to cut down on the carbs (especially processed flours and sugars) and incorporate exercise into your routine. See Mypyramid for more info. Ƶ§œš¹ [aɪm ˈfɻɛ̃ⁿdˡi] 20:59, 13 October 2006 (UTC)[reply]
It's supposed to be 5-10 servings of veggies and fruit, and a serving is 1/2 cup packed, 2/3 of a cup of juice, or 1 cup of salad. And contrary to the above post, reducing carbs is not a guaranteed way of losing weight, and not a proven positive choice for health (although reducing or even eliminating sugar and highly processed foods is definitely good). There are good and bad carbs and lots of people aren't sensitive to them. The only guaranteed way of losing weight is to take in fewer calories than you burn. As for protein, serving size depends on the type of protein. IIRC the meat serving size is approximately the size of a deck of cards, fish and soy is somewhat larger (6 oz), 2 eggs = 1 serving, and about a cup of beans = a serving. Anchoress 21:08, 13 October 2006 (UTC)[reply]

As for whether the recommended number of servings is a maximum or minimum, use it as a min for good foods, like fruits and vegetables, and a max for bad foods, like meat. StuRat 21:57, 13 October 2006 (UTC)[reply]

I think the food pyramid servings would seem incorrect with it's carb intake because some-most people aren't as active as they were in the past when this pyramid was suggested to them! --Agester 01:02, 14 October 2006 (UTC)[reply]
I wasn't guaranteeing weight loss (I said might). Burning more calories than you consume often takes more than just reduced input since your input can affect your metabolism. Ƶ§œš¹ [aɪm ˈfɻɛ̃ⁿdˡi] 07:38, 14 October 2006 (UTC)[reply]
It's certainly true that drastically reducing calorie intake can reduce set point, but I've only heard rumours about foods and food groups that actually affect metabolism. Were you thinking of any food in particular? Anchoress 07:49, 14 October 2006 (UTC)[reply]
No, I was more thinking about basal metabolism and how meal frequency and size can affect it, but apparantly this has been debunked. I vaguely recall a conversation with a health-conscious friend about carbohydrates but it was a while ago and he also believes that his car ("Laura") talks to him, that he speaks French well, and that his name isn't short for Francis. Ƶ§œš¹ [aɪm ˈfɻɛ̃ⁿdˡi] 23:18, 14 October 2006 (UTC)[reply]
Above all, don't stop listening to your body and don't over eat, if you've had your 5 servings of carbs by 7pm and you are not hungry, don't stuff another slice of bread in just because your pyramid says to.. ;) Vespine 05:03, 16 October 2006 (UTC)[reply]

Calculating wavelength of radio waves at a given frequency

I know it says not to give homework problems, so i'll just change the numbers. I'm having a problem where I can't seem to calculate the wavelength of a particular wave in megahertz, and I suspect i'm just not getting the concept, because I can't get any of the other problems which have anything to do with calculating wavelengths, de Broilegh or otherwise. For example, if there was a bunch of radio waves moving at, say, 100 MHz, I start by changing that to hertz to make it 1,000,000 hertz, then divide 2.998x10^8 by that value, and that should be my wavelength in meters, yet somehow i'm still wrong. (And it doesn't appear to be signifigant digit errors either) Can anyone tell me what i'm doing wrong? Homestarmy 21:53, 13 October 2006 (UTC)[reply]

First off, 100 MHz is 100,000,000 Hz, not 1,000,000. StuRat 22:00, 13 October 2006 (UTC)[reply]
So is the mega 10^6 and not 10^4? :/ Homestarmy 22:01, 13 October 2006 (UTC)[reply]
Yes. StuRat 19:31, 14 October 2006 (UTC)[reply]
SI prefix has a table. -- Consumed Crustacean (talk) 22:04, 13 October 2006 (UTC)[reply]
Or you can just look at mega or even mega-. In fact, also megahertz and MHz lead to this information.  --LambiamTalk 23:24, 13 October 2006 (UTC)[reply]

Nuclear fission (in nuclear weapons)

It seems to me that the amount of released neutrons in an atom-bomb grows exponential. How does it slow down? Why won't the neutrons just crash into other atoms and split them? (Henningklevjer 22:28, 13 October 2006 (UTC))[reply]

Why do you think it does slow down? In a "perfect" bomb, every single fissionable nucleus is fissioned; in practice the bomb is violently dismantling itself as it works, so a proportion of the fuel is cast off before it can be fissioned. One of the design goals for any practical fission device is to maximise the yield for a given mass of fuel, and minimise the waste. -- Finlay McWalter | Talk 22:44, 13 October 2006 (UTC)[reply]
Thanks. I guess I just thought every neutron hit another atom. (Henningklevjer 22:54, 13 October 2006 (UTC))[reply]
Ideally that's just what happens, but of course the bomb has only so many fissile atoms and (just like most other exponentially growing things in the real world) it soon runs out of new atoms to recruit into this subatomic Ponzi scheme. -- Finlay McWalter | Talk 22:56, 13 October 2006 (UTC)[reply]

That's why it is so difficult to make a really good atomic bomb that can fit in a missle, and why the North Korea 'bomb' may have been a dud (or never attempted). --Zeizmic 00:12, 14 October 2006 (UTC)[reply]

Decompression

If a person suffers sudden decompression, as in an airplane at high altitudes, does one pass out? If so, how long might such a condition persist? I imagine it isn't around the same time for everyone, even if they all suffer the same decompression. What is the cause of passing out in this manner? Thank you. --Demonesque 23:14, 13 October 2006 (UTC)[reply]

Uncontrolled decompression may cause air embolism, which can be fatal. See also Decompression sickness.  --LambiamTalk 23:32, 13 October 2006 (UTC)[reply]
Hypoxia (insufficient partial pressure of oxygen) is the cause of one passing out. And the condition persists until you reach a lower altitude, obtain an alternative source of oxygen, or die.
Atlant 00:05, 14 October 2006 (UTC)[reply]
Someone told me she hit that problem when a window broke. It was not too awful, only everyone had their ears bleeding ... the eardrums were better some days later. -- DLL .. T 18:40, 14 October 2006 (UTC)[reply]
In aviation the term is Time of Useful Consciousness. See the article, note that altitudes quoted in flight levels (eg FL180) are quoted in hundreds of feet, so FL180 is 18,000 feet. Over 30,000' it's measured in seconds. 203.22.236.14 10:23, 18 October 2006 (UTC)[reply]

October 14

Space Shuttle: burn-up.....

Why doesn`t the Space Shuttle burn-up during it`s ascent?....The protective tiles aren`t in the line of "flight"? Also,,,ty for "answers" to Ballistics. 152.163.100.74 02:09, 14 October 2006 (UTC)[reply]

First off, it's not going very fast on the way up, at least where the atmosphere is dense. Also, it's oriented to minimize air resistance, unlike its descent, when it's aligned to maximize it in order to lose speed. Clarityfiend 02:40, 14 October 2006 (UTC)[reply]
If the same thrust was applied for decelaration on the way down as is needed for acceleration the way up, the shuttle would have comparable speeds at the same air densities and not get very hot. However, that would require carrying an awful lot of extra fuel – doubling the amount and then more because the extra fuel needed for later slow down must also first be brought up. So that is why air braking is used. See also our article Space Shuttle, in particular the section on Landing.  --LambiamTalk 08:25, 14 October 2006 (UTC)[reply]
The above has pretty much covered it, recently I read that the shuttle DOES in fact get quite hot on the ascent. The shuttle starts at a stand still and accelerates through an atmosphere that gets thinner as it gets higher so it doesn't get as hot as when it starts fast and falls into an atmosphere that gets thicker as it descends. But it's definitely included in the design. Vespine 04:44, 16 October 2006 (UTC)[reply]

Planetary rings

In our own star system, rings are only found around gas giants; is this just a coincidence, or are gas giants more prone and/or rocky planets less prone to having rings? Furthermore, could a habitable planet have a ring system? CameoAppearance orate 02:23, 14 October 2006 (UTC)[reply]

The gas giants have rings because they're more massive. Rings are created when space debris is ripped apart by the planet's gravity. Also, I see no reason why a habitable planet can't have a ring system (or why a planet with rings can't be habitable). --Bowlhover 02:48, 14 October 2006 (UTC)[reply]
I believe being torn between two gravitational sources, like the planet and another moon, is what typically breaks up a moon into a ring (or prevents a ring from forming a moon). StuRat 14:50, 14 October 2006 (UTC)[reply]
The solar wind is probably a major factor. According to the article, it's believed to be strong enough to have stripped Mars of a third of its atmosphere. Something as fragile as a ring wouldn't have been able to survive too close to a star. Clarityfiend 04:33, 14 October 2006 (UTC)[reply]
I would say the rings are less easy to blow away than the atmosphere, being composed of much larger particles and rocks. Therefore, the solar wind should have little effect on rings. StuRat 14:43, 14 October 2006 (UTC)[reply]
Hmmm...after further consideration, I declare myself a solar windbag. Clarityfiend 16:27, 14 October 2006 (UTC)[reply]
Two quotes from our article Planetary ring: "Pluto is not known to have any ring systems. However, some astronomers think that the New Horizons probe might find a ring system when it visits in 2015." "It is also felt that Phobos, a moon of Mars, will break up and form into a planetary ring in about 50 million years due to its low orbit." So astronomers even consider dwarf planets to be potential ring bearers.  --LambiamTalk 08:10, 14 October 2006 (UTC)[reply]
One theory about the formation of Earth's Moon (and the most popular one, I believe) is that it formed out of the debris left over from a collision, which first formed a ring and then the rocks in that ring pulled togeether to form the Moon. Of course, that ring was temporary, but what isn't? As a sidenote, the biggest gas giant with a ring in our solar system is the Sun itself, with the ring being the asteroid belt. Or is that essentially differnt? DirkvdM 09:33, 14 October 2006 (UTC)[reply]
Earth's Moon formed out of the debris left over from a huge collision between earth and another very large object. Also, you could say that all the suns orbiting bodies including the planets form a big ring. --Light current 11:35, 14 October 2006 (UTC)[reply]
The "huge collision" that you guys refer to is known as the Giant impact hypothesis. Since the Sun is essentially an immense gas giant, it follows the current "only gas giants have rings" trend. -- Plutor talk 15:36, 14 October 2006 (UTC)[reply]

stadium construction

What will be the approximate cost of construction of International Cricket stadium in terms of no. of spectators

It will depend on factors like location and size of the cricket pitch. Location would be extremely complicated since it will affect the cost of labour, local laws and legal requirements, cost of land etc Nil Einne 14:06, 14 October 2006 (UTC)[reply]
The cost in terms of number of specatators? What, is it supposed to collapse or something? DirkvdM 08:23, 15 October 2006 (UTC)[reply]

A cold

What is the best way to get yourself feeling fit enough to go out when you have a chest cold? No time for doctors-- I need to go out tonite!--Light current 10:53, 14 October 2006 (UTC)[reply]

Lots and lots of fluids (water, not soda), warm chicken soup, tea with honey, cough pastils in case you are feeling it in your throat. Refresh the air in the rooms you are in by opening the windows wide for a small amount of time, which is the way which works. Cup of hot chocolate, Calvin and Hobbes and a sofa, blanket, pillow and a scarf for safety does it for me. "Does it for me" doesn't mean it clears out the cold, it means that it makes me feel superb. 81.93.102.3 11:23, 14 October 2006 (UTC)[reply]
I take it "cough pastils" is Britspeak for cough drops ? If so, I recommend eucalyptus. Beware, though, that too many make your jaw fall asleep. StuRat 14:02, 14 October 2006 (UTC)[reply]

What about sleep?--Light current 11:35, 14 October 2006 (UTC)[reply]

You said you had no time for sleep! Over the years, with myself and the kiddies, I found that the standard approach works best: lots of sleep, fluids, and ibuprofen. The 'other' approach of ignoring the cold, taking symptom-reducers, and partying all night, has consequences. With a chest cold, the most likely effect is that it develops into pneumonia, which is a bitch. --Zeizmic 11:44, 14 October 2006 (UTC)[reply]
No, usually sleep has no time for me. Insomnia. However, when feeling completly shattered with a cold I can sometimes sleep as I have been doing the past 15? hours on and off. Feeling better now. I will be venturing out after food. THe cold has now mainly progressed to the head and is not as painful. Any advice on dealing with that one? 8-? --Light current 18:10, 14 October 2006 (UTC)[reply]
Take a large heat-resistant bowl or dish, fill it with 10cm (a couple of inches) of boiling water, add 1 to 5mL (a teaspoon) of eucalyptus oil, place your face over the steaming water, place a towel over head and bowl to trap the steam, breathe deeply in through your gurgling nose and out through your mouth. Expect to cough, expect to feel very refreshed afterwards. You can get eucalyptus oil where you are can't you? Sorry I'm a bit late for the tonight that passed a few days ago, but perhaps it's of use to someone somewhen. 203.22.236.14 10:36, 18 October 2006 (UTC)[reply]
Not too late. Ive still got a bunged up head. Im wondering if inhilation of Olbas oil in hot water would work. I have some of that. 8-(--Light current 13:54, 18 October 2006 (UTC)[reply]

Boiler Efficiency

I have read before that boilers produce waste steam as it functions as a HEAT SINK. So can we install economisers to extract heat from the steam in the heat sink and return that heat to main boilers,such that the temp of condensed water in the heat sink is slightly higher than surrouding areas, resulting in higher efficiency??

You should sign your name, so we can see if boiler questions exceed seagull-time. Boilers are really old and perfected technology. Any further attempt to squeeze more out of them has consequences. Also, boilers are under a raft of laws, ever since they used to blow up a lot in the early 1900's. --Zeizmic 11:49, 14 October 2006 (UTC)[reply]

They may be old but they are used everywhere( nuke power plants, submarines,ships)

do-it-yourself X-ray Crystallography?

What's the best way to build a homemade rig for getting Crystallographic data? Made out of household materials?172.149.83.128 13:28, 14 October 2006 (UTC)[reply]

Divine intervention? N.B. It depends on your crystal quality and what data quality you wished to obtain as well as which method you plan to use to solve the phase problem. However your best bet would be to use a synchrotron and therefore you'd need several kilometres of tunnelling for starters. With very good quality crystals 2 Ångströms resolution should be possible! Nil Einne 13:38, 14 October 2006 (UTC)[reply]
A basic x-ray crystallography machine sits on a table top and certainly doesn't require kilometers of tunnels. Rmhermen 21:38, 14 October 2006 (UTC)[reply]
No but that crap isn't going to give you the resolutions and quality you need to get published in Nature or Science :-P Plus what do you do if you have a small or weakly diffracting protein? Nil Einne 14:59, 15 October 2006 (UTC)[reply]
Your project sounds quite ambition but I could imagine that Debye-Scherer scattering may be in the reach of a skilled amateur. Maybe you want to read up on that subject. However, even for this, you need an X-ray tube, and if you manage to obtain one, remember that X rays arehazardous and potentially fatal. And they kill silently. Make sure you know enough about radiation safety to avoid endangering you or others before you even think about fooling around with such dangerous stuff. Simon A. 15:37, 14 October 2006 (UTC)[reply]
Isnt there some hazard to be aware of regarding X Ray reflection from certain surfaces?--Light current 18:13, 14 October 2006 (UTC)[reply]

homemade NMR?

What's the most effective way to build your own NMR probe out of old household electronics?172.149.83.128 13:29, 14 October 2006 (UTC)[reply]

The first thing you need is a very powerful electromagnet. To test if your electromagnet is powerful enough, I suggest you obtain a tank full of toxic gas, say hydrogen cyanide gas. Bring this to within say 6 metres of your electromagnet and turn it on. If you are still alive, you need a more powerful magnet. Hope this helps! Nil Einne 13:43, 14 October 2006 (UTC)[reply]
What he is trying to say is that the magnet needed for magnetic resonance must be huge; the smallest MRI scanner uses 0.3 Tesla magnets, which is about a thousand times the magnetic flux density of a powerful horseshoe magnet. Plus, in order to make it even slightly practical, you'd need to cool it with a liquid gas (nitrogen is most commonly used, I think). I'm not sure about making a probe with MRI. The magnet might be a bit less powerful, but it would still be absolutely massive, and probably therefore a superconducting magnet, and in order to make it shrinkable to probe size, would need a phenomenal cooling system. Laïka 14:10, 14 October 2006 (UTC)[reply]
Yes. Bringing a metal object anywhere near to a magnet used for MRI or NMR when it's on will likely result in the object being drawn to the magnet at fairly considerable force. Although I'm not sure whether the tank would actually break, possibly not AFAIK most oxygen tank incidents the tank doesn't break (but has killed by force) although with hydrogen cynide all you would need would be a minor leak and... It probably would have been better to recommend the guy wear several chains. Or perhaps put something put several knives on a table and then stand between the table and the magnet. BTW this link may be of interest to some [20] Nil Einne 14:38, 14 October 2006 (UTC)[reply]

What would you need to build a Mass spectrometer from junk sitting around the house?172.149.83.128 13:31, 14 October 2006 (UTC)[reply]

Again divine intervention might be helpful. Failing that, I don't suppose you happen to have a million dollars in cash lying around? This junk should give you a decent spectrometer. Nil Einne 13:46, 14 October 2006 (UTC)[reply]
But Nil, come on, don't discourage our young optimistic DIY enthusiast-troll unnecessaily. I'm sure you can get a simple second-hand beginner's synchrotron for less than $100,000, plus shipping, handling and installation. Simon A. 15:43, 14 October 2006 (UTC)[reply]
Mass specs are nowhere near that expensive. You can pick a basic one used for less than $10,000 and it doesn't require any supercooling or superconducting wires. Rmhermen 21:35, 14 October 2006 (UTC)[reply]
Oh sorry, I though we were still talking about synchrotrons. Actually, Froogle in fact lists suppliers for mass spectrometers, starting at US$500. Simon A. 08:59, 15 October 2006 (UTC)[reply]
And before Rmhermen corrects me on that, too: I know that one can build synchrotrons without superconducting magnets, but it's simply less cool to do so. Simon A. 09:00, 15 October 2006 (UTC)[reply]
Well it depends on what kind of mass spectrometry you want. I would suggest you start off with a fourier transform ion cyclotron mass spectrometer. You'd probably want a few different types but really for bio research nowadays you shouldn't leave home without a FTIC-MS (except that you're doing it at home so I guess you just should leave home and when you do leave home you probably shouldn't take it with you). BTW $100k for a synchrotron, are you joking? Try reading the synchrotron article... The smallest synchrotron I know (I don't know many) is Singapore's 0.7 GeV and I don't know how much it cost but I'm pretty sure it would have been in the millions [21]. But a protein crystallographer is going to want something like the 3.0 GeV Australian synchrotron [22] ([23]). To be fair, you're not going to need so many beamlines if you only want to do X-ray crystalography but if you've got the synchrotron why waste it? Nil Einne 15:30, 15 October 2006 (UTC)[reply]

RadioShack?

Does radioshack sell coils of super conducting wire?172.149.83.128 13:35, 14 October 2006 (UTC)[reply]

Very good news... Yes they do! Unfortunately they don't happen to sell the equipment you need for the near absolute zero temperatures for the wires to show superconductivity Nil Einne 13:49, 14 October 2006 (UTC)[reply]
Before you go and sink hundreds into gold-plated solidified-hydroxyl acid audio cables or whatever, you should read High-end audio cables. Basically, the quality of the cabling has no bearing on the sound quality, as the frequency of a wave being transmitted down the cable will be unaffected by any impurities found. You might hear a slight increase in volume from the cables, due a reduced resistance, but the effect is apparantly mainly synesthetic. Laïka 15:33, 14 October 2006 (UTC)[reply]
I'm afraid that you missed that the OP is going to build a synchrotron, mass spectrometer, or NMR apparatus in his backyard. Given that for all three of them, an extraordinary powerful magnet is the crucial part, having superconduvting wires is indeed the way to go. Now, there is the funny paradox that the best normal conductors such as gold do not get superconducting at all or only at ridiculously low temperature and even lower magnetic fields, while mediocre conductors get superconducting above liquid helium temperature. Hence, the classic is lead, and a niobium-tin-alloy is the state of the art. However, you cannot exactly buy this brittle stuff as wire. But we are all waiting for the obvious next qestion: Where do I get a cheap supply of liquid helium? Simon A. 15:58, 14 October 2006 (UTC)[reply]
Lead isn't brittle, it's relatively soft. StuRat 18:57, 14 October 2006 (UTC)[reply]
I know, but I was taling about the NbSn alloy. Simon A. 09:02, 15 October 2006 (UTC)[reply]
Isn't gold never a superconductor (as silver and I assume iron and steel since their ferromagnetic)? At least this is what the article says. Normal copper is not a good superconductor either because of impurities. Aluminium and tin are supposed to be superconductors however and from Google you can buy aluminium wire from radioshack. The tin or aluminium shield in coax wire should do too I guess... Alternatively how about solder? Sounds like it should be a superconductor (although don't know what temperature). In any case, I'm fairly sure you should be able to obtain a superconductor from radioshack as I mentioned (of course obtaining the equipement for the temperatures required is a different matter) Nil Einne 14:56, 15 October 2006 (UTC)[reply]
Isn't gold never a superconductor is right! Aluminium and tin? What? Since when did this happen?Template:Susbt:user:Mac Davis/Sig07:44, 17 October 2006 (UTC)

synchrotron?

1) Can you buy a synchrotron on ebay? 2) Where would you keep it?--172.149.83.128 13:36, 14 October 2006 (UTC)[reply]

I happen to have a spare synchrotron lying around! I would list it on ebay except I think their system might crash with the success fee. In any case, given the extremely large success fee, how about we just negotiate and out of auction trade? I might be willing to part with mine for say NZ$500 million? It'll be best if you pick up (and yes I do accept cash on pickup although we'll have to visit the bank first). However if this is not possible for you, unfortunately it's a bit large to be couriered but I might be able to deliver it myself. Just add another say NZ$500 million and we have a deal. Of course, I do expect your payment before I arrange delivery. BTW, I do assume you already have the necessary space? As mentioned above you will need a lot of underground tunnelling. Just make sure you have say 50km and we're sweet! P.S. No PayPal or Western Union or escrow please. Again I don't want to crash their systems. P.P.S. The above IP address looks up to AOL. Explains some things I guess... Nil Einne 13:56, 14 October 2006 (UTC)[reply]
Maybe this has something to do with eBay's sponsored links on Google; among the amazing things eBay apparently sells are Photons and Plutonium, while Amazon is offering Angular Momentum. No sign of an eBay sponsored link for Synchrotrons, but there is an Amazon one [24]. Laïka 14:18, 14 October 2006 (UTC)[reply]
I wouldn't mind selling photons. I wonder what the starting bid should be? Maybe I could sell angular momentum as well but not so sure where I could get plutonium from... Nil Einne 14:43, 14 October 2006 (UTC)[reply]
I don't think these things would ship well... --Zeizmic 15:01, 14 October 2006 (UTC)[reply]
I can't help thinking the guy who talks about "An alternative to the $16 Photons" got a bit ripped off; the sun provides about 1021 of photons per sqaure metre a second for free. Paying $16 for each would bankrupt the entire planet in something like 1 E-24 s! Laïka 15:16, 14 October 2006 (UTC)[reply]
Well for the photos I just ship the maker. I mean if you were buying a er, motherboard, which one will you prefer. One motherboard or a device which produces motherboards? I'll go with the production device any day. Same thing with the photons... Alternatively, I could make the photos pick-up only. That way I meet them somewhere and give them the photons (finding a way to collect them is their responsibility) Nil Einne 14:48, 15 October 2006 (UTC)[reply]

New Dynamo Not Working Right

The groves on my dynamo wore away and the rubber that they were made from fell to peices so I replaced it with another. Originally, two wires went into the connections marked with an earth symbol (ignoring the two connection holes marked with bulb signals) and both the front and rear lights functioned. After swapping the dynamo for one with only two connection holes, the rear light no longer functions but the front light does. How could this be? --Username132 (talk) 14:00, 14 October 2006 (UTC)[reply]

It is not clear from your description how you have things wired up now.  --LambiamTalk 17:25, 14 October 2006 (UTC)[reply]
You have to tell first if the rear bulb is OK in itself : test it against some battery. -- DLL .. T 18:27, 14 October 2006 (UTC)[reply]
Electrical connections have a nasty habit of corroding over time. Perhaps when you put the wires back, the exact points of contact for the rear light were not clean metal but rather oxidized, high-resistance points? We're not talking about enough voltage to punch through any significant blockage. Or maybe it is a change in the required wiring from old dynamo to new.
Atlant 13:43, 16 October 2006 (UTC)[reply]
I figure out that my problem was that both wires should go to the same connection. I never really understood how the lighting worked but it seems that it uses the frame of the bike as one of the connections. I had the back light wired to the frame of the bike instead of the other connection provided. Thanks anyway! --Username132 (talk) 14:03, 16 October 2006 (UTC)[reply]

Speed of digestion

How long does it take for food to be digested by human beings, i.e. to go in one end and out the other? Clarityfiend 15:26, 14 October 2006 (UTC)[reply]

Now this sounds the perfect question for a science-at-home experiment. Have fun. But I'd say it depends a lot on what yiu eat, wouldn't you agree? Simon A. 16:02, 14 October 2006 (UTC)[reply]
Someone did experiments on this in the '60s?. They found that the transit time amongs African tribes was about 12 hours, whereas in the Western world it was more like 24 hrs. Hence the recommendation to eat more fibre like the African tribes did to avoid Western diseases. 8-)--Light current 17:35, 14 October 2006 (UTC)[reply]
Oh, I thought it was to avoid a pale skin.  --LambiamTalk 17:40, 14 October 2006 (UTC)[reply]
No: I dont think lots of fibre turns you black. However, excessive amounts may make you act like a horse.--Light current 18:37, 14 October 2006 (UTC)[reply]
This time, for a change, let me be the one to advise you to go see a doctor. DirkvdM 08:30, 15 October 2006 (UTC)[reply]
I Africans have such a healthy diet that we should copy it, then what are they complainig about? :) DirkvdM 08:30, 15 October 2006 (UTC)[reply]
I didnt know they were complaining about the diet, maybe just the quantity?--Light current 10:35, 15 October 2006 (UTC)[reply]

Anyone who claims their digestive process takes over a week is full of crap. :-) StuRat 18:41, 14 October 2006 (UTC)[reply]

Well maybe half full. THe bottom half we hope 8-(--Light current 18:45, 14 October 2006 (UTC)[reply]
I knew a guy who claimed that his digestion only took about half an hour. --WhiteDragon 17:59, 16 October 2006 (UTC)[reply]
Well, some things, like apple cider, can go right through you that quickly. StuRat 19:20, 16 October 2006 (UTC)[reply]
Yeah what this guy meant was that half an hour after eating something, he had to have a crap. Cause and effect? Yes and no, but what goes in does take time to travel the 33ft of intestines. Half an hour would be 1 ft/min or 1" every 5 sec! Plainly ludicrous unless you have dysentry--Light current 08:18, 17 October 2006 (UTC)[reply]
Right, that's just the new stuff pushing the old stuff out. The same thing happens with electricity in a wire. You put some electrons in one end and some fly out the other end. For those specific electrons to make it through the wire takes much longer, however. To prove the case to your friend, have him eat a can of corn and then "watch for the results". :-) StuRat 15:25, 17 October 2006 (UTC)[reply]

rainbow

does the rainbow look the same to everyone

Probably not; some people are colour blind. Or do you mean whether it depends on where you stand? As long as there are enough water drops around and you have the sun in your back, you should see a rainbow as an arc, centred around the point where the shadow of your head is and with an angular radius of about 42°. That is the same for every observer. As you move, the rainbow moves with you, just like your shadow. See further our article Rainbow. Finally, perhaps you mean the old conundrum whether you see the colour red the same as the next (not colour-vision challenged) person. Do you have the same subjective experience? That question is intrinsically unanswerable, and it can even be argued that it is meaningless. See further the article on the philosophical concept of "Qualia".  --LambiamTalk 17:36, 14 October 2006 (UTC)[reply]

No. From above it looks like a complete circle. See rainbow Hence you can never get to its end and find the pot of gold 8-(--Light current 17:38, 14 October 2006 (UTC)[reply]

Clotrimazole 1% Cream

Would it be safe to use this once or twice as impromptu treatment for oral thrush?

No, clotrimazole is available in lozenges for this purpose, ask your doctor about them. Note that oral preparations of clotrimazole need a prescription in part because they can have profound effects on the metabolism of other drugs. This drug is not normally absorbed through the skin, so it is safe to use, for example, on the feet without a prescription; but systemic (i.e. oral) use should be done under the supervision of a physician.Tuckerekcut 18:31, 14 October 2006 (UTC)[reply]

Contaminated drinking water

I have been involved with a situation this week, where a water sample taken from taps used for drinking was found to have "Rotaser" in the sample and the Water Supplier demanded that the water supply be turned off immediately. Can someone tell me what "Rotaser" is please. Thanks.

Perhaps you misheard the word protozoa. Are you from Boston?Tuckerekcut 18:34, 14 October 2006 (UTC)[reply]
How about rotifer ? StuRat 18:35, 14 October 2006 (UTC)[reply]
OK I'll have it well done with salad. 8-)--Light current 18:43, 14 October 2006 (UTC)[reply]

I received a phone call from the Water Supplier's Scientist who spelt the word out for me. I am from Kent in the UK, but I fancy Boston.

I suspect that the "scientist" is either incompetent or not really a scientist, and was just reading a misspelled version of "rotifer" to you. StuRat 15:22, 17 October 2006 (UTC) StuRat 15:17, 17 October 2006 (UTC)[reply]
Safe to say we don't know? Google brings up nothing.[http://www.google.com/search?num=20&hl=en&lr=&safe=off&q=Rotaser&btnG=SearchX [Mac Davis] (SUPERDESK|Help me improve)07:50, 17 October 2006 (UTC)~[reply]

Why does my stomach growl?

Little growls are common, but when I haven't eaten for about 12 hours (which is rare), my stomach makes a reeeallly long growl - like 5 seconds straight. I was just curious to know what exactly is going on down there... - R_Lee_E (talk, contribs) 19:25, 14 October 2006 (UTC)[reply]

Your new word for today is borborygmus. alteripse 19:32, 14 October 2006 (UTC)[reply]

Since the article doesn't actually answer the question, I'll make an effort: The sounds you hear in your belly are essentially sounds created from air bubbles pushing in the direction opposite the chyme (food) in your belly. Your intestines are like a long cylindrical baloon with both mushy food/poop and gas. The muscles lining the intestines are regularly squeezing in a rhythmic fashion (see Peristalsis), parallel to the lumen of the intestine. When the muscle squeezes the gas and chyme down toards the terminal end, the pressure builds up and two things can happen: either the chyme/gas can push further down the alimentary tract, or the gas can squeeze through the contracting muscles to relieve the pressure. The first case can happen until the chyme (poop) pushes against your anus (specifically the voluntary sphincter, the first area with voluntary control), but the latter case happens often too, and this causes bowel sounds. These sounds are more easily heard when they occur near the distal end of the alimentary tract, but they occur throughout, and can be auscultated with a stethoscope.Tuckerekcut 04:39, 15 October 2006 (UTC)[reply]
Could you also explain why the sound is much louder when the digestive tract is rather empty? Simon A. 09:04, 15 October 2006 (UTC)[reply]
I don't know but maybe because there's more gas? Nil Einne 14:44, 15 October 2006 (UTC)[reply]
Probably it is louder when there is less material in the tract for a few reasons. First, if there is less chyme, then the gas bubbles are large (whereas lots of chyme causes lots of small bubbles distributed through the intestines), so the individual noises will be louder, though longer in between. Also, the gas spaces themselves, being larger, will cause the sounds to seem louder through resonance. The latter case is not dissimilar to a sink emptying: when the sink is mostly full, the drain sounds are muffled by the rest of the water, but there is a loud "sink emptying sound" when the last bit of water swirls down the drain. Don't take this as fact, though, I'm really just guessing on this one.Tuckerekcut 18:44, 15 October 2006 (UTC)[reply]
Good idea, the comparison with the sink. That sounds convincing. Simon A. 20:25, 16 October 2006 (UTC)[reply]

The article we have on this beastie states, "The stinger of the Asian giant hornet is about a quarter-inch (6 mm) in length, and injects an especially potent venom that contains an enzyme so strong that it can dissolve human tissue" and goes on to say "The venom is optimized to kill bees". Am I the only person to feel that there is an inconsistency with this? Why would it produce such potent venom when it would seem that a single well-placed sting with a venomless stinger of that length would likely incapacitate, if not outright kill, a bee? Or am I underestimating the hardiness of the insects? Vitriol 20:03, 14 October 2006 (UTC)[reply]

Interesting question. I'm no expert either, but I can well imagine that chitin is so hard that it is easier to dissolve it with a corrosive liquid than to pierce it. Note especially that in order to pierce a hard fabric, you need a substantial force. (Have you ever tried to push a needle through leather?) A hornet, even if it is a giant hornet, might be simply to light to apply this force without pushing itself away rather than piercing the victim's shell. And as far as well-placed is concerned: if the hornet and the bee engage in a dog fight they might not have the liberty to carefully place their stings. Simon A. 20:15, 14 October 2006 (UTC)[reply]
The word "inject" suggests that the exoskeleton is first pierced. I assume the venom has to act quickly, since a bee sting might still be fatal to the hornet.  --LambiamTalk 21:02, 14 October 2006 (UTC)[reply]
Bee stings don't really affect the hornet, in fact, only the native bees have found a way to kill it (see the bee ball in the article), the European honeybees that beekeepers initially used were just slaughtered by the giant hornet. The dog fight stuff is wrong too, the hornet raids the beehives. As for the asker's question, I don't know the answer. The above information comes from a program on Discovery I saw about them. However, I *think* saw the giant hornets kill small birds. And i believe some professor talked about being attacked as a child so I think the statement could in fact be correct. - Dammit 21:21, 14 October 2006 (UTC)[reply]

Building Audit

How do I conduct a building audit ? 219.95.213.170 22:05, 14 October 2006 (UTC)[reply]

Do you mean a structural inspection, a financial audit of the company that owns the building, or what ? StuRat 00:28, 15 October 2006 (UTC)[reply]

What kind of insect is this?

Hi, what kind of insect is this: http://static.flickr.com/100/269517767_fdf7e2009b_b.jpg http://static.flickr.com/96/269517764_b83e0f27e0_b.jpg Here someone says assasin bug (sometimes called a conenose) http://forums.dpreview.com/forums/read.asp?forum=1036&message=20460456 But ... I don't think that is correct. /Roland

It's not correct. Vitriol 22:59, 14 October 2006 (UTC)[reply]
I don't know and I doubt I can help but I suspect others could potentially be more likely to help if you could give details of where this photo was taken etc Nil Einne 14:41, 15 October 2006 (UTC)[reply]
It certainly doesn't look like an assassin bug, but it is definitely some sort of bug; you can tell by the ventrally retracted mouthparts. BenC7 01:11, 16 October 2006 (UTC)[reply]
Having said that, though, a number of insects are known to change colour in the early stages of their life. BenC7 01:15, 16 October 2006 (UTC)[reply]

Two questions about chemical reactions

1. The atoms in HCl and NaCl all have full outer electron shells in these constellations, so why does HCl react with, say, skin, and NaCl does not?

2. H2 and O2 have full outer electron shells, so why does 2H2 and O2 react to become water? Jack Daw 23:15, 14 October 2006 (UTC)[reply]

Do your own homework, please. Vitriol 01:06, 15 October 2006 (UTC)[reply]
this doesn't have a homeowrk ring to me, so here goes. 1.what you want is the page acid. HCl is an acid, because it is a source of "H+". this is the species that causes damaage to your skin. 2. full electron shells don't necessarily denote stability. things can be stable for a number of (rather complex) reasons, but here is a simplification. in the case of molecules like O2 the molecule can react to form molecules of lower energy, like the reaction with H2 to give water. so given the choice between the full shells of O in O2, and the full shells in H2O, the oxygen would much rather be in H2O. Xcomradex 01:23, 15 October 2006 (UTC)[reply]
1. What is it about hydrogen that makes it react with skin? Why isn't Na "loosened" from its bond with Cl to explode (as it says on the sodium article that it does on contact with water)? 2. They don't? So what's up with what you learn in school that "atoms want to achieve noble gas structure", is that oversimplified BS? Anyway, why, then, will a molecule of lower energy "rather" be formed? Jack Daw 02:40, 15 October 2006 (UTC)[reply]

sodium explodes because of the burning of the hydrogen formed:

2Na0 + H2O -> H2 + 2Na+ + 2OH-

so compounds containing Na+ will not explode in the same way. The acid burns you by changing the pH of your skin, which causes denaturation. the Na+ ion does not change the pH, so doesn't burn you. your school-level simplification is exactly that, a simplification. you can have compounds with full electron shells undergoing reaction (eg. the noble gas xenon forms many compounds, such as xenon difluoride). and you can have molecules with unfilled shells that are quite stable (eg. the stable radical TEMPO). it is not a worthless concept, but it is not a hard and fast rule either. the entire universe is slowly winding down into its lowest energy state, these same rules apply to chemistry, and ultimately control what compounds are formed in chemistry. if you are interested in this sort of thing a few advanced level chemistry courses could keep you entertained for quite some time. Xcomradex 04:28, 15 October 2006 (UTC)[reply]

Thank you. Actually chemistry is my first alternative to med school. Anyway, I can once again conclude that pre-college education is simplified to a very frustrating level. Jack Daw 16:29, 15 October 2006 (UTC)[reply]

if those are your interests you should consider scrapping medicine, and doing medicinal chemistry. they're the guys who design and build new drugs. Xcomradex 21:27, 15 October 2006 (UTC)[reply]

October 15

How big must it BE?........

I hope you weren`t thinking "dirty" here, but, here`s the question... How big, in aperture, would a ground-based optical telescope have to be to able to 'just' glimpse a man-sized, let`s say, object, on the Moon. Let`s also say that the Moon is at its mean distance, assume 'perfect' optics, 'perfect' observing conditions, object is "lying down" on the Moon`s equator, etc.....I`m guessing around 300 feet of aperture. I think Dawe`s Limit might prove helpful. I don`t need to know HOW to make the calculations,,,as this is certainly NOT homework,,,,just curious. I hope you have fun with this,,,and Thank You. 152.163.100.74 00:07, 15 October 2006 (UTC)[reply]

The "how" is at angular resolution. --Robert Merkel 00:39, 15 October 2006 (UTC)[reply]

(:)Boy! Thank you so much,,,,THAT was helpful. I thought this might have been interesting for OTHER readers. Was it easier to answer the "how" rather than give the real answer? Thanks again. 152.163.100.74 00:54, 15 October 2006 (UTC)[reply]

What's with,,,,,,the commas? Vitriol 01:02, 15 October 2006 (UTC)[reply]
What about commas don`t you understand?....pause to think. 8 ). 152.163.100.74 01:08, 15 October 2006 (UTC)[reply]
Edit'd for the destruction of confusion. I mean, why do you sometimes,,,,,put strings of commas in your posts? Vitriol 01:10, 15 October 2006 (UTC)[reply]

(:::) I hope I did this correctly. I`m just a newbie here. I guess my commas are for extended pauses, as one might do qhile simply talking with someone, a nuance, I guess. I offer total and complete apologies for any confusion. I think I need to go to the help desk to better understand how to ask and reply to information. Again, so sorry. Now,,,how big must that telescope be? 8) All jokes aside. Sorry, forgot to sign, but, really, how big must that telescope be? Dave152.163.100.74 02:04, 15 October 2006 (UTC)[reply]

You are looking for an Ellipsis for that effect (...). Sorry, I don't know anything about the question. --liquidGhoul 02:53, 15 October 2006 (UTC)[reply]
For heaven's sake, the calculations are a bit of high-school trigonometry, a bit of arithmetic, and most importantly a bit of effort to dig up the right numbers to plug into the calculations. FWIW, I get a result suggesting you'd need a telescope about 2 kilometres wide, which is not implausible given the numbers quoted in Apollo Moon Landing hoax accusations. .--203.214.55.189 03:32, 15 October 2006 (UTC)[reply]
With a telescope 2 km wide, you'll be able to resolve details as small as 8.6 cm. It would be helpful, but unnecessary. --User:Bowlhover 05:15, 15 October 2006 (UTC)[reply]
That's what I was working on. I was making the assumption our human is standing upright rather than lying there sunning himself...--Robert Merkel 10:50, 15 October 2006 (UTC)[reply]
However, for completeness, you could instead use interferometry to, in effect, synthesise a telescope of that size. --203.214.55.189 03:34, 15 October 2006 (UTC)[reply]

Well, I didn`t want to use interferometry but, the distance between 2 'lenses' would turn-out to be exactly what the diameter of a single lens would be anyway. As for 2 kms...that sounds WAY too big. Are you sure about that? Did you do any calculations? Here`s a start...Dawes Limit, in arc-seconds, is d/4.5..."d" being in inches of aperture of primary. Moon is approximately 30 minutes of arc. A 6 foot object at the equator is a tiny fraction of an arc-second. Do you need a 2 km-wide primary to resolve that? Dave152.163.100.74 03:58, 15 October 2006 (UTC) Small correction, and more help....Dawes Limit d/4.6, Moon is approximately 2000 miles in diameter and subtends approximately 30 arc-minutes. Now divide 6 feet into 2000 'miles-worth' of feet, the quotient being THAT fraction of 30 arc-minutes. Plug that into the Dawes Limit equation, and the diameter of the primary should be the answer to my initial question. Now, as per my initial question, "Did you all have fun with that?" Dave152.163.100.74 04:29, 15 October 2006 (UTC)[reply]


The largest man-made object on the Moon is 5 metres large, which is Atan(5/363 104 000)==7.88*10^-7 degrees as seen from Earth. (363 104 000 is the distance, in metres, to the Moon.)
From the angular resolution article:
Sin(7.88*10^-7 degrees)/1.22 = 1.1*10^-10
 Finally, "an" answer, and very close to my initial guess. Thank you very much! Dave 152.163.100.74 05:20, 15 October 2006 (UTC)[reply] 
380/(1.1*10^-10) = 3.367*10^10 nanometres = 33.7 metres --User:Bowlhover 05:06, 15 October 2006 (UTC)[reply]
A qualitative answer from half-remembered college physics: You make the lens LARGE to increase the light gathering ability and image faint distant stars. You separate the two lenses and combine them interferometrically to enhance resolution. So two or more small lenses at a distance apart could resolve the stated target at a lower cost than building a very large lens to achieve the same objective.Edison 05:50, 15 October 2006 (UTC)[reply]
Yes actually some people are considering these multiple (distant) lens telescopes. I believe it was in the New Scientist some time this year or perhaps last year Nil Einne 15:42, 15 October 2006 (UTC)[reply]
Don't forget to take in to account the earths atmosphere Nil Einne 15:40, 15 October 2006 (UTC)[reply]

Thank you very much for all your replies. Very interesting. Dave 205.188.116.74 15:55, 15 October 2006 (UTC)[reply]

Come nuclear armageddon...

...will the gulls survive? I was just watching them flying around today and musing to myself "you're smart, you're resoruceful, you're self-sufficent, you're adaptable, you've learned to live off us but have not come to rely on us, you still know how to survive, you do what you must in order to live, you just carry on with your life, unconcerned with all the bullshit that's happening below you - as long as you have a full belly, a safe place to sleep and occasional sex, you are content with your life". I started comparing the gulls to us and realized, in a slightly profound moment, that somewhere along the line, we've lost some of the things that they still have - to our detriment. So, when/if the bombs fall - what are the odds that more gulls than humans will make it through to see the daylight again? Sorry if this all sounds a bit off-the-wall, it's quite late here and I sometimes get a bit 'deep' at this time of night. --Kurt Shaped Box 01:30, 15 October 2006 (UTC)[reply]

I think the question here is, "will the Reference Desk gull questions survive?" Vitriol 01:38, 15 October 2006 (UTC)[reply]


Maybe following armageddon, a 'gull world order' will arise to fill the niche we left behind - causing the WP ref. desks (or the gull equivalent thereof) to be deluged by questions from adolescent gulls worried that their adult plumage hadn't grown in yet. I'd imagine that if gulls had evolved enough to use computers, there would be plenty of five-year-old gulls neurotic about still having some brown feathers whilst all their peers were snowy-white. Oh yeah, they'd probably freak out the first time their breeding hormones kicked in too - cock birds worried about the sudden uncontrollable feelings of sexual aggression they were feeling, hen birds worried about unexpectedly laying unfertilized eggs, etc. --Kurt Shaped Box 10:40, 15 October 2006 (UTC)[reply]

I would think birds would be particularly susceptible to fallout, since they fly through it. Meat-eaters would also suffer from an accumulation of radioactivity that gets more severe the higher up the food chain you go. So, I would expect gulls in areas of fallout to be wiped out. Gulls in unaffected parts of the world should survive, assuming there isn't a "nuclear winter". StuRat 03:14, 15 October 2006 (UTC)[reply]

The radiation would cause them to mutate thusly. (actually only gulls that live by the bay, bay gulls --WhiteDragon 18:08, 16 October 2006 (UTC)[reply]
The gulls would survive a nuclear winter. The world would be a feast of carcasses. The larger species would hunt the other scavengers. They would also eat the cockroaches. :) --Kurt Shaped Box 10:31, 15 October 2006 (UTC)[reply]
Please God, No! Ordinary cockroaches are one thing, but cockroaches with wings? --Light current 12:38, 15 October 2006 (UTC)[reply]
Don't most cockroaches have wings? Nil Einne 15:38, 15 October 2006 (UTC)[reply]
Many adult cockroaches have wings, and some – for example the Asian cockroach – can fly quite well.  --LambiamTalk 16:07, 15 October 2006 (UTC)[reply]
But not as high as a kite!--Light current 13:48, 16 October 2006 (UTC)[reply]

What could be a physiological explanation for the Chinese folk medical concept of 熱氣 ("heat")?

I'm not sure if the concept is in the orthodox theories of Traditional Chinese Medicine (it may well be), but I'll just call it a folk medical concept. The concept I'm referring to is 熱氣 ("rè qì" in Pinyin romanization; Literally, 熱="heat" or "hot", 氣="gas"). As best I can describe it, it is a syndrome with (some or all of) these associated symptoms:

  • a general feeling of hotness (the body temperature may be normal or only slightly elevated),
  • sore throat,
  • a feeling of thirst (despite plenty of fluid intake),
  • yellow (and possibly foamy) urine,
  • lips that look a darker shade of more red than normal, and
  • a sensation that the air exhaled through the nostrils is warmer than usual.

The syndrome is often attributed to over-consumption of certain kinds of food (e.g. deep fried foods (especially overcooked or burnt), spicy food), or under-consumption of the antagonistic kinds of food (described as "cooling"). According to (folk?) Chinese medicine, to stay healthy, one should consume food in antagonistic categories in balanced quantities.

Can someone think of one or a small number of body conditions, in physiological terms, that can explain the symptoms associated with 熱氣?

[Edited to add:] 熱氣 is a well-recognized phenomenon in Chinese (folk?) medicine but, as far as I can tell, is not recognized as a syndrome in western medicine. I suspect that the symptoms are not unrelated but manifestations of one (or perhaps a few) underlying conditions, which should be describable/explainable in scientific terms. I'm hoping to find a correct scientific explanation, or at least a testable theory, for the phenomenon. --71.246.5.19 04:51, 15 October 2006 (UTC)[reply]

  • Yes 熱, that would be a normal state, so 熱熱 would mean there is to much (double). Mion 04:55, 15 October 2006 (UTC)[reply]
    • Tricky thing is what is your source ? as all medicine had to be converted to the local farming population, the source can quote a mainstream medical in that time or a local one in that time. ?. Mion 05:21, 15 October 2006 (UTC)[reply]
No, there isn't. At least none that i've heard of. And i don't think there will be - because the concept of heat is a large collection of symtoms that chinese medicine explains with an umbrella concept of "heat energy" in the body. I'd say many of them, if not most, are not actually so closely related. Most of the individual symptons can be explained pretty easily with simple biology. For example, one of the things i often hear is not to eat too much lychees - because lychee is apparently very hot, and eating too much lychees gives you the traditional symptons of heat, most notably...people easily get blood noses from it. This goes for most tropical fruit as well. It's like, chinses medicine would say "eating lychees --> lychees are very heating --> therefore people get symptons like blood noses". Western biology will probably just explain it as that tropical fruit probably contains compounds which in some way affect the permeability of small blood vessels, the blood vessels in the nose which contribute to blood noses are very fragile. More permeability = much easier to rupture = much higher chance of blood noses. This is pretty much the same as people with hay fever get blood noses very easily on hot dry days (hay fever = an allergy to flower pollen. very common in some parts of the world.)
anything that increases your metabolsim will most likely give you a general feeling of hotness and excitment, a feeling of warm exhaled air, and redder lips. For example, exercise increases metabolism - if you go for a run, you will feel very warn, get redder lips, and the air you exhale will feel warmer because your body temperature is heating up. Although the normal body temperature is about 37oC, your body can tolerate a few degrees higher. And when you exercise, you're body temp will normally increase a few degrees before cooling mechanisms kick in. A lot of those foods that cause heat increase metabolism one way or another, although not nearly as much as how your metabolism increases when you go for a run. Spicy foods are described as "hot" in english for good reason - because that's exactly what they do. The "spicy"-ness tastes hot, and makes you feel hot. A chinese person will just tell you it's 'heat'.
Something like the color of urine is also very easily explained. Urine varies in color a lot - lighter color urine means there is more dilute and more water. Go drink 2-3 liters of water in one shot, and take a look at the color of your urine the next time you piss. You should find it is very clear, almost colorless. Where is if you go for a day without drinking, you will find your urine very dark. You'd also find yourself very thirsty. When youre body doesn't have enough water, you feel thirsty, and your urine turns dark. Because your body will try hard to conserve water - and a very good way to conserve water is to have less water get peed out. Coincidentally (or maybe not so), being hot expands a lot of water. Because normally when youre feeling warm/hot, you also happen to be sweating. Which wastes/uses a lot of water. Warm air also tends to be more moist.
It is not very surprising that one of the 'fixes' for heat is to drink water. Which tends to fix most of those symptons, for example...the dark colored urine. A chinese doctor may tell you it's because the water helps balance your body, and fixes the fact that you've eaten too much heat food. I'd just say drinking more water = pissing out more water, and therefore you have a more dilute urine. Foods which are cooling, like pears, happen to be very light (not much digestible stuff to digest) and very juicy. Once again, the water.
Anyhow, i've never actually read anything about chinese heat from a western medicine's point of view. It would be very interesting to see what western medicine does say on it. But those are just the explainations that i would give as a bio student if someone just told me about the symptons (i.e. i ate this and this and noticed i had this and that sympton) based on general human biology.
Thanks for responding. Your suggestion of increased metabolism is a good start, whether or not it turns out to be part of the correct answer. I still think that the symptoms are explainable in terms of some underlying physiological conditions. A few additional comments:
  • The feeling of hotness causes by spicy food persists for only a relatively short time. The feeling of hotness from 熱氣 persists way beyond the disappearance of the immediate effects of consuming spicy food. So, the role of spicy food in 熱氣 is beyond that of a short-term stimulant.
  • Increased blood vessel permeability is one possible explanation for the increase likelihood of nose bleeding, but it may not be the only hypothesis available. Raised blood pressue, and perhaps a number of other mechanisms that I'm not aware of, may be the (more principal) cause of the symptom.
  • The subjective sensation of hotness, both in the body and in exhaled air, does not necessarily correlate to increased body temperature. From experience, I know that the sensations are sometimes felt when the orally measured temperature is normal or even below the nomimal normal body temperature.
  • The color of the urine may correlate to a state of (de-)hydration, but that's probably not the full story. More concentrated urine does not directly explain the foaminess (sudsiness?) of the urine. It seems to me that something in the urine is responsible for the apparent reduction in surface tension. Also, the subjective feeling of thirst is not caused by dehydration alone. It can persist even when one has drink so much water than urination becomes frequent and the urine passed becomes colorless. I should note that there's at least one allegedly very effective herbal remedy for the "thirst" symptom.
--71.246.5.19 14:55, 15 October 2006 (UTC)[reply]
medical Flower? it would explain the increased metabolism, next time i'm going to ask the farmers first about it.. Mion 10:23, 17 October 2006 (UTC) ?[reply]
I asked my friend (he's a farmer}, he said best is once daily after meal, in the beginning its not a problem if you skip a day. reg. Mion 11:17, 17 October 2006 (UTC)[reply]
Western medicine  : Λ
.................................... V

six pack

is there a way to tell if you have a six pack without removing the fat through excercises such as running but just by doing sit ups?

Well of course you have one; everyone has a rectus abdominis muscle. However, any belly fat will obscure it, so it will not be prominent unless you have a low body fat percentage. Since sit-ups don't remove fat as well as aerobic exercises such as running, no, you will probably be unable to achieve a defined six pack without running or doing other supplemental exercise. Hyenaste (tell) 05:52, 15 October 2006 (UTC)[reply]

I would say that a "six pack" means having well defined abdominal muscles which are visible (not obscurred by fat). So, by definition, you don't have a six pack if you can't see it. Your question is something like "How can you tell if an 800 pound woman has a pretty figure under all that fat ?". StuRat 20:38, 15 October 2006 (UTC)[reply]

fine is there a way you can tell that you are ready to start running to show the six pack after lots of sit ups

You don't need a lot of muscle to have a six pack if you also don't have a lot of fat to obscure it, I'm a really lean guy, I never do sit ups, but i have a six pack, just because I have very low body fat. My friend is a gym instructor twice my size, he went on holidays and put on about 10 kg, bam, no six pack, even though his muscles would be twice the size of mine. Sounds to me like you are just procrastinating about losing the weight. If you want to sculpt, you can do both aerobic and weight training, it will take you twice as long to build the muscle and then lose the weight. PLUS if you go full on with weights without losing the fat you can easily end up with stretch marks when you lean down, which is not pretty. Don't forget to sign your posts. ;) Vespine 04:14, 16 October 2006 (UTC)[reply]

Seeing Colors

When I look at the color red do I see the same thing as what other people see when they look at it? Is there a way to know for sure?

You see almost the same thing (assuming you are not color blind). Seeing as everyone has a different genetic structure, it is impossible for everyone’s Cones and Rods to develop EXACTLY the same. However, seeing as every healthy person has similar Cones and Rods, what you are seeing is similar to everyone else’s. The color cannot be exactly the same, but it is VERY close. THL 07:32, 15 October 2006 (UTC)[reply]
You see with your brain not your eyes. Philc TECI 20:09, 15 October 2006 (UTC)[reply]
Seeing is also experiencing. Even if the underlying hardware (the sensors) is the same, the interpretation by the brain, the experience, may be very different. I have thought about this a lot, and this is getting a new impulse now that I am reading up on Alzheimer. Different people can have very different perceptions of the same thing. knowing how they experience something may seem imposible, but magic mushrooms can give a hint. They can make you see colours that you never saw before, even though you are viewing the very same things you normally see. If a brain on mushrooms can see those colours, the what might another person see? No answer, but an indication of how differently minds can percieve the same things (assuming there is an objective reality out there, but let me not go into that). DirkvdM 08:47, 15 October 2006 (UTC)[reply]
If you believe you may be color blind, you can get that checked by a doctor. Dismas|(talk) 10:35, 15 October 2006 (UTC)[reply]
(This is strictly for clarification) Dirk raises a good point. My answer doesn't contradict his, nor does his mine. Shrooms cause some major chemical imbalances in the brain (be they temporary ones); therefore, a person with a brain having differences drastic enough to cause a change in perception would (by our definition of "normal") be unhealthy. THL 10:53, 15 October 2006 (UTC)[reply]
The color you see is physically the same. What you call red is the same wavelength as what everyone else calls red (assuming neither you or they are color blind). But color, like all perception, is very subjective. Have you ever like...looked at something really bright white (like something that was actually *shiny* white), and then looked at a sheet of normal paper and suddenly realized the paper looked really yellow...and not so white at all? Or seen someone who has black hair, but looks slightly brown in the sunlight. But then when you look at it outside of the sun, you swear it is 100% black? --`/aksha 12:42, 15 October 2006 (UTC)[reply]

The latter has to do with the fact that lightbulbs do not emit all light wavelengths, where as the sun does. Under artificial light, the person's hair is black, and out in the sun the person's hair is brown. The hair can't reflect what isn't there. The former is an interesting point. I'm really not sure what to say about that. I don't know much about the brain itself, so I don't know how to address the initial perception. However, seeing as the experience carries over to everyone, that shows that the brains of healty, "normal" people are similar in many ways. THL 14:35, 15 October 2006 (UTC)[reply]

If you mean: "Do you have the same subjective experience?", well, that question is intrinsically unanswerable, and it can even be argued that it is meaningless. See further the article on the philosophical concept of "Qualia".  --LambiamTalk 16:20, 15 October 2006 (UTC)[reply]

You see with your brain not yours eyes, your eye recieves information, your brain sorts this into a visual image which you interpret. Therefore I would say that the chances that you see the same thing are extremely slim, but as long as you can tell the difference between colours, light shades etc. it doesnt really matter. Philc TECI 20:09, 15 October 2006 (UTC)[reply]

There have to be some major similarities because we all view the same colors as being complementary. I also think that if people's interpretations of colors were at much variance we wouldn't have the relatively similar perceptions of art which are made up from mixes of many different colors. (There is a bit in Niven's [[[The Mote in God's Eye]] where the humans come across alien painting and find it incredibly unpleasant to look at since their perceptions of colors are so different.) I imagine if people were too off you'd get more of the "I don't see what you see" or "Why is this weird color placed here?" that you get when trying to talk about colors with someone who is colorblind. Without any compelling reason to think that humans have terribly different perceptions of color, I wouldn't assume it; color vision evolved long before humans were around, and while there are no doubt some slight variations in it, I don't see any reason to assume that amongst people there would be fantastically different views of it. I once took a class ages ago with a psychologist who had done a lifetime's worth of work in color vision who basically said the same thing — there is no indication that humans perceive colors with any great difference from one another (except colorblindness, which is a problem with the physical structure of the eye), so there's no good reason to assume there is much of a difference from person to person. --Fastfission 20:21, 15 October 2006 (UTC)[reply]

Lack of evidence for something is not evidence against something though. Especially since this is something that it is impossible to experiment with, as we will only ever see the colours wee see. You would still be able to distinguish colours and see the same relationships between them, as that is information carried in the wavelength of the colour itself, but perception and understanding of this information is something different. Philc TECI 17:31, 16 October 2006 (UTC)[reply]

How Many calories does a Gram of Human Fat Have?

How many calories (note lower-case c) does a gram of human fat have? THL 07:22, 15 October 2006 (UTC)[reply]

One for every degree it can heat a litre of water. Sorry, that was silly. :) BINAS gives 37 kJ for cod liver oil, which is pure fat. If you want that in calories, you'll have to convert it yourself because I don't do obsolete units. I'm not sure, though, what this means. Is that the amount of energy that can be extracted by a healthy human (with an empty stomach?) for actual use in muscle power or to heat the body? I have wondered about this before. What does the amount of energy given on food packaging mean? And your question seems to be about fat already 'digested into' the body (assuming you're no cannibal), which is a different thing yet. DirkvdM 09:04, 15 October 2006 (UTC)[reply]

I was wondering (no, the following is not a joke. It is a half-baked idea that would never work) if human fat contains enough energy to be an alternative fuel source. I could have people eat until they are very fat, lipo the fat out, process it, and sell it for fuel. Tons of people would love getting paid to eat all day, and the fuel source would never run dry. THL 10:41, 15 October 2006 (UTC)[reply]

The food source might. 81.93.102.3 10:56, 15 October 2006 (UTC)[reply]
In 6 billion years, has food for animals ever run out? Maybe in localized areas, but now that humans have airplanes, that really isn't a problem. THL 11:02, 15 October 2006 (UTC)[reply]
Right, we could "lipo the fat out" of the passengers for fuel in flight!  --LambiamTalk 16:24, 15 October 2006 (UTC)[reply]

<--Besides, if food runs out fuel is the least of our worries. THL 11:03, 15 October 2006 (UTC)[reply]

1 gram of fat is 38 kilojoules of energy. Or 9 Calories (that's BIG CALORIE) or 9000 calories (that's small calories).

That's rendered fat. Unrendered will be fewer. Anchoress 12:00, 15 October 2006 (UTC)[reply]

Human fat is a terrible fuel source because you have to make it by eating other things. Any time you move up a trophic level in the food chain, about 90% of the energy in the source is wasted. Its much more efficient to use the things we eat as fuel rather than processing them through the human body. pschemp | talk 12:30, 15 October 2006 (UTC)[reply]

Well, I did say the idea was half baked and would never work. I was just curious how much energy was in fat so I could compare it to fossil fuel. THL 14:18, 15 October 2006 (UTC)[reply]

Protein and carbohydrates contain 4 calories per gram, and fat contains 9 calories per gram. Edison 20:05, 15 October 2006 (UTC)[reply]

This page may help too: [25] But regarding your original idea, it's pretty much equivalent to meat farming, except in that case we have the luxury of actually killing the animal. And meat farming is notoriously inefficient. The article environmental vegetarianism has some stats regarding how the efficiency (output per input) of meat is much less than of crops; these measure output in terms of protein rather than calories but you get the idea. Biofuel is a very sensible idea, but you want to use the crops more directly. See also comment from pschemp above. Arbitrary username 12:49, 17 October 2006 (UTC)[reply]

Much thanks everybody. I knew the idea would never work, and I wouldn't have the money to finance such an undertaking anyway. It was more of a curiousity than anything else. THL 22:50, 17 October 2006 (UTC)[reply]

Am I secretly gay?

While I am sexually attracted to women, sometimes I have wet dreams about having sex with 'chicks with dicks' (not so much transsexuals but women who have both a penis and a vagina), which usually involve me sucking or playing with cock at some point. I've started looking at dickgirl hentai online and found it sexually arousing. Does this mean that I have a homosexual streak hidden within me somewhere? I'm confused and I'm not sure what to do. --84.65.109.37 10:19, 15 October 2006 (UTC)[reply]

Most people have no sexuality focused on a gender alone, and it is quite possible to be different gradients of bisexual, in favour of either hetero or homo. I seem to remember a quite thorough statistics done on this topic alone, but it has evaded me, and I don't have any satisfying graph to show you. Who you are, very few but yourself hold any answer to, and if you don't, of course no one should claim to. 81.93.102.3 10:54, 15 October 2006 (UTC)[reply]
I would go talk to a psycologist. They may not hold the answers, but they can help you find them for yourself. THL 10:59, 15 October 2006 (UTC)[reply]
It depends on your definition of GAY. If you defined gay as loving people with penises, then you are secretly gay. If you defined gay as loving people who look like men, then you are not secretly gay. So make up your mind, what your definition of gay is and you would know what you are.
This may sound like a stupid suggestion - but have you ever actually stopped and asked yourself whether you are attracted to men? Just having wierd fantasies doesn't nessasarily equivalate to homosexuality. But if it's bothering you so much, a psychologist would be appropriate. --`/aksha 12:36, 15 October 2006 (UTC)[reply]
Many people find certain fantasies arousing that they would not be aroused by – and perhaps even find abhorrent – if occurring in reality. There is a tendency to define neat categories people's sexual orientation can be placed in – straight, gay, bi, fetishist, submissive – but reality is more fuzzy and fluid. The confusion is caused by inadequate categories. I see no reason to be worried unless your dreams start interfering with your social life.  --LambiamTalk 14:53, 15 October 2006 (UTC)[reply]
I wouldn't care much, since if you are actually attracted to women you are by definition non-gay, since homosexuals are those who are attracted (just or "mainly") by their own sex. I think it's quite normal for people to have strange sexual fantasies apart from their main definition of sexuality. So my opinion is that you are not secretly gay.

How is your sex life? If this question is uncomfortable I understand. Remember, you are anonymous, none of us are going to know who you are. THL 17:17, 15 October 2006 (UTC)[reply]

Don't worry about it, we all have a few skeletons in the closet. StuRat 20:32, 15 October 2006 (UTC)[reply]

Are they gay skeletons? Hyenaste (tell) 20:34, 15 October 2006 (UTC)[reply]
Well, all skeletons are rather limp wristed and light in the loafers, aren't they ? :-) StuRat 03:03, 17 October 2006 (UTC)[reply]
I'm guessing you are probably still pretty young, you've gone through puberty and your a big bag of hormones that your body simply hasn't gotten used to yet. I don't think you have anything to worry about, there are things I had wet dreams about I wouldn't even admit anonymously ;) . The worst thing you can do is think you are a weirdo or something, everyone goes through things that seem strange. Don't worry about it, the wet dreams will stop (sometimes I wish they didn't, they can be more fantastic then anything you could do in real life!! :D ) enjoy it while you can. Your mind will settle and it will start making sense before you know it. And whatever happens, there's nothing wrong with being gay. Vespine 03:57, 16 October 2006 (UTC)[reply]
when I was younger I had somewhat similar fantasies. You may be imagining doing things like that to other people because you want that done to yourself. THL 05:22, 16 October 2006 (UTC)[reply]

You are homosexual if you are also a chick with a dick. If you are clearly male or female biologically, then you are having pansexual fantasies. (Not all people are clearly 100% male or female physically).

You can't control your dreams and fantasies or sexual attractions anyway, so you must learn to accept them. They may also change. - THB 08:29, 16 October 2006 (UTC)[reply]

I was told once by someone who is usually knowledgable on such things that interest in transexuals who appear as women with penises is predominantly a fantasy of heterosexual males. Not sure why. Dragons flight 08:50, 16 October 2006 (UTC)[reply]

I talked to a psychologist about it when I was having them; he said that I was having them because that was something I wanted done to myself. It was interesting. THL 13:11, 16 October 2006 (UTC)[reply]

What kind of frog is this?

I found a frog in a creek a few miles inland from the Connecticut shore, and took a picture of it. What kind of frog is it? grendel|khan 11:36, 15 October 2006 (UTC)[reply]

Leopard frog. alteripse 12:50, 15 October 2006 (UTC)[reply]
I don't see the "distinct light-edged dark spots across the back" in the Flickr picture that should make them recognizable according to our article.  --LambiamTalk 14:29, 15 October 2006 (UTC)[reply]
Look at the choices: [26]. There are a couple of species of leopard frogs, and they are one of, if not the most, common green frogs with dark marks in the eastern US. I suspect the light borders are not universal in all lifestages or variations, but do not claim unusual expertise in frog identification. alteripse 14:52, 15 October 2006 (UTC)[reply]
In any case, I still don't spot them, just as I wrote. Maybe the text of our article should be adjusted (by an expert). Or perhaps Grendelkhan discovered a new species :)  --LambiamTalk 14:58, 15 October 2006 (UTC)[reply]
(edit conflict) From this (same as Alteripse, but cam across before your post), you have 10 species to choose from. This frog is missing a few characteristics found in the Leopard Frog, particularly the large spots and dorso lateral line. I would say that it is the Green Frog (Rana clamitans). The green above the lip is a characteristic common in Green Frogs. Thanks. --liquidGhoul 15:00, 15 October 2006 (UTC)[reply]
Can`t be a leopard, I don`t think. Feet appear to be too long for leg length? Just a guess. 152.163.100.74 18:33, 15 October 2006 (UTC)[reply]
I'd go with the (boringly named) Green Frog, based on the pictures, perhaps a young one. The leg stripes and back pattern look pretty similar to this little guy's. --Fastfission 20:12, 15 October 2006 (UTC)[reply]
Yep, it is a Green Frog (Rana clamitans).--Tnarg12345 22:24, 15 October 2006 (UTC)[reply]

"Absolute hot"?

Everybody knows about absolute zero. But I've been wondering, could there be such a thing as "absolute hot", in which all the particles are moving at c? Would this be a temperature that you could measure in Kelvin? --The Lazar 15:22, 15 October 2006 (UTC)[reply]

Negative temperature might help (I don't quite understand this level of physics). I would assume it would be infinite temperature although you can have things hotter then infinite temperature so... Nil Einne 15:35, 15 October 2006 (UTC)[reply]
According to the special theory of relativity, only massless particles (photons, gravitons if they exist) can travel at c, only at c and nothing but c. Particles having invariant mass remain below c. From the formula for (relativistic) kinetic energy you can compute how close to c theory allows you to make them go for any amount of energy that you put in:
.
If the kinetic energy is very large, you can approximate that by
,
which is obviously less than c.  --LambiamTalk 16:58, 15 October 2006 (UTC)[reply]
Not really. No massive particle can actually reach the speed of light. But could they all have the same very high speed v that is only a little bit smaller than c? I'll discuss this questions.
The main point to understand about temperature is that kinetic energy is heat only if the particles (let's talk about a gas for now) all move in random directions. If they all moved in the same direction, it would just be wind. (We discussed this only a few days ago in the Science Reference Desk. You may want to scroll up a bit for that.) Now, if all particles have the same temperature, but different direction, they will all speed outwards, and you have no longer a volume filled with gat but rather a spherical hollow shell of gas that expands. (The picture I have in mind is supernova remnants. Or were you imagining a gas in a box? Let's imagine that all gas particles have the same velocity v. The particle scatter, i.e., they bump into each other an exchange energy, i.e., if one particle bumps from behind into another one, the rear one loses speed and the front one gets even faster. (Go and drive bumper cars to see what I mean.) So, after a short while (the so-called thermalization time) the particle have no longer all the same velocity, but rather, the velocity is distributed after a specific formula, the Maxwell-Boltzmann distribution. Only then does it make sense to say that the particles of the gas have a temperature, and this is the temperature according to the M-B distribution. The temperature can get arbritrarily high, only that after a while, the Maxwell-Boltzmann formula becomes wrong: Even for high energies, the Boltymann formula correctly tells us, what kinetic energy a particle has with which probability, but in order to see how fast it is, we must not use the usual because this formula only holds for velocities v much smaller then c. Rather the formula from special relativity has to be used, that tells us how the kinetic energy can become larger and larger without the particle ever reaching the speed of light, because its mass also gets larger ad larger. (See mass in special relativity).
Hence, there is not maximum temperature. Negative temperatures don't really count IMO because they are a rather artificial concept. I think the article explaind it but if you have questions, ask. Simon A. 17:08, 15 October 2006 (UTC)[reply]

Negative temperatures are not an artificial concept; actual physical systems have been observed at negative temperatures for minutes at a time. To answer the original question, the hottest possible temperature is −0 K; it occurs when a system is in its unique highest energy state. But a gas does not have such a state, and there is no relativity involved. Melchoir 19:41, 15 October 2006 (UTC)[reply]

Ok, I take the bait and explain. Let us first move away from a gas to an idealized spin system, i.e. a solid whose consituents (atoms) store their (thermal) energy not by vibrating (i.e., no phonons in my model) but by being excited. Furthermore, we do not allow for arbitrary excitations, rather, there is a amximal energy, that each atom can store. So, let us say, that an atom can store either no or one quntum of energy. You might say, that this sounds like an idealization far from reality, but there are situation where all only those parts which I did not just "idealize away" are relevant for a certain specific discussion. And only then may we talk about negative temperatures -- and this is why I called it "somewhat artificial". The standard example for such a situation is a laser medium, and one might encouter it also when discussing magnetism. In this situation, the amount of energy stored in an atom (or [quasi-]spin, to use more correct terminology) is one of a discrete, finite set of values E0, E1, ..., En,, and the probability that a given atom has one of these values is proportional to the Boltzmann factor . This means, that if the system is thermalized (i.e. at equilibrium), the probalities are such that they fullfil this formula with T being the ambient temperature. Now, we can engineer the system such that the probilities do not follow this law, for example by making some mechanism that puts most of the quasispins into the state , and as long as we keep this mechanism switched on, it would be incorrect to assign a temperature to it. Now, it is not so easy to devise such a mechanism and it turns out that often, we cannot deviate much from the formula. For example, if we have only two energy levels and shine very bright light of the right frequency onto it, we can pull the atoms to the higher level but they will keep falling down (see Einstein's theory of the photo-electric effect and the James-Cummings model) to the lower one, so that at most half of them are in the upper one. And this situation can be described by the Boltzmann formula by putting T to infinity. There is a more clever thing, called population inversion and being the basis of the laser, where the upper state has higher probability as the lower, but in such a way, that the Boltzmann formula gives the correct ratio if one plugs in a negative T. This is then called a negative temperature.
Now, why is this not a real temperature? Well, by definition, a body A is hotter than B if B gets warmer when brought into contact with A. So, if A is now our quasispin system with the negative temperature, of which we have learned that it is hotter than all positive temperature, should it now not heat up body B? It does in fact, if body B (our thermometer) is the same idealized system of quasispins. But statistical physics stresses that temeperature is athing that can be defined universally: Every object, if leftalone long enough in order to thermalize, will aquire a temperature which can be compared in a definite way with any other body with at thermal equilibrium. (By comparing, I mean, we can watch, whether heat flows from A to B, or vice versa, which tells us which one is hotter.) This is often called the zeroeth law of thermodynamics. Now, as body B is any system, we have to take into account all the degrees of freedom that we "idealized out" before. And if B is sufficiently hot, we will see without doubt that heat flows from B to A, showing that A is not so hot, although it had allegedly a negative (more than infinite) temperature. Simon A. 21:59, 15 October 2006 (UTC)[reply]
This too is wrong. If A has any negative temperature and B has any positive temperature, and the two are brought into thermal contact, heat flows from A to B. If heat were to flow from B to A, then both systems would be decreasing their entropies! Melchoir 22:10, 15 October 2006 (UTC)[reply]
Exactly what I am saying: If A has truely negative temperature, then heat would flow from A to B, reagrdless how high the positive temperature of B. Now I say that empirical evidence supports the following claim: For every body A, a body B can be prepared that is so hot (has so large positive temperature) that it would let heat flow onto A, even if A were in a state which some may call having "negative temperature". Hence, it is untenable to say that A's so-called negative temperature is hotter than any positive temperature. If you disagree, you have to cite an example for an object A that is able to transmit heat onto any object B with positive temperature. There is no such thing, or do you claim otherwise? If so, I'm curious to hear what this might be. Simon A. 22:53, 15 October 2006 (UTC)[reply]
Nuclear and electron spin systems. If you want emprical evidence, go track down the Physical Review references cited by Kittel and Kroemer. Or perhaps you'd like to expand upon your own "empirical evidence supports the following claim"? Melchoir 23:41, 15 October 2006 (UTC)[reply]
Look, habe you read my longer text above? I know, that nuclear and electron spin systems are the standard example. Let's take the example of nuclear spin systems. The levels in nuclear spins correspod to very low energy, typically in the microwave range. So, you have some bulk of matter and put it into an NMR apparatus. Using clever pulse sequences, you achieve population inversion. Taking the Boltymann formula you say, "Population inversion means negative teperature. My stuff is hotter than enything else in the world." To rove, you put another lump of the same kind of cold matter next to it, and, hey presto, the population inversion spreads over, warming up the other piece a little bit. Actually, this last part of the experiment will turn out quite difficult in practice, and most importatly, it will work at all, only if all non-spin degrees of freedom are frozen out or decoupled in both samples. I hope you know what I mean be "frozen out or decoupled" because that is the crucial assumption that physicists imply when they talk about negative temperature. And that is the problem, too: Proper temperature is defined with all degrees of freedom taken into account. So, to prove you wrong, I caome to your lab and est your population inverted sample, but not using another one with the aformentioned property but rather a piece of hot nearly-melted iron, which I put with tongs next to your sample. Now, do you seriously claim that my iron will not simply melt down or destroy your sample, but instead get even hotter?
Please look up your references yourself. I'll see if I find a good exaple later today. But I'm sure you'll notice that they compare the temperature of their samples with other samples of the same kind and not with any arbritrary body as required by by the zeroth law. You'll also notice that they nowhere claim that their sample would heat up any other samples. Typically, in such papers, the language is not careful enough as to mention the reservation about the unusual notion of temperature employed, because this is assumed to be knwon by the reader. Simon A. 07:23, 16 October 2006 (UTC)[reply]
Well, how weakly coupled do two systems have to be to merit separate temperatures in your book? Is an hour's relaxation time between spin and kinetic degrees of freedom not long enough? What if we achieve a day? A year? How unphysical are you willing to make your demands?
When my spins achieve equilibrium with any conventional probe -- your iron, the lattice they live on, whatever -- they will fall from a majority high-energy state to a majority low-energy state. Conserve energy. Melchoir 08:12, 16 October 2006 (UTC)[reply]
I'd agree, an hour is long enough. So after a n hour, the coupling with the lattice (the phonon degrees of freedom (DOFs)) is established, they fell down and gave their enrgy to the lattice DOFs. So, it is justified to say that be spin subsystem was hotter than the lattice sub-system, because energy has flown from the spin DOFs to the phonon DOFs. Agreed. Also, we may argue that as long as we keep within this framework, the spin DOFs had indeed a higher temperature, and a negative one. But this is only correct as long as we stay in a finite state system (as you have pointed out yourself above). If I put the sample into a really hot oven, it melts, i.e., leaves this subspace, and takes on heat. This means that the sample as a whole was cooler than the oven.
But now we're getting to the core of our dispute, finally: I said that "in my book" a temperature is a property of a body in equilibrium, and if I put a very hot second body next to it, the second body will let heat flow to the first, showing that it is cooler. "In your book", you allow to ascribe temperature to subsets of the DOFs within a single sample. You suggest to assign a different temperature to the spin DOFs and the lattice DOFs, because they couple only weakly. Correct, this is what one likes to do in NMR physics and then, negative temperature make sense. But this is a generalization of temperature. To summarize: (i) There is no contradiction in the claims of your last post and my "empirical claim", because you talk about heat flows between different DOFs of the same system and I talk about heat flows between seperate bodies, and asked for an example of that kind. If you in fact claim, that there is a physical example of a body or DOF subsytsem with negative temperature that can let heat flow onto another, conventional body (and not just onto this body's better coupled DOF system), then I ask for an example. Otherwise, (ii) we agree and only use different terminology: While I claim that the usual definition of temperature requires the different DOF systems within one body to thermalize and consider metastable states with thermalization only with DOF subsets as a generalization of temperature, you don't see it necessary to split this off as a generalization. The reason why I see it as a generalization is that we lose the validity of the following axiom: When a hotter body is brought in contact with a cooler one, heat will always flow from the hotter to the cooler (which becomes untrue in the case of your "hot" negative-temperature spin system next to my hot (but only positive temperature) piece of iron). Simon A. 11:06, 16 October 2006 (UTC)[reply]
But you don't lose any axioms. Your experiment, at best, involves three systems with their own different initial temperatures: the iron (warm), the sample's spins (hot), and the sample's other DOFs {cold). At no point is the iron exclusively in contact with the spins, so you are not observing the heat flow between them. You might as well pour 0°C water on a frozen-over lake, watch it freeze, and then complain that the lake floor is only above zero in the sense of an axiom-breaking generalization, because it couldn't heat up your probe. Oh, never mind the layer of ice between them. Melchoir 18:00, 16 October 2006 (UTC)[reply]

physics

tthere r 8 identical balls each having same charge & energy E ,if they r united to form a sphere.wat is the energy of this new sphere/

How do you unite balls to form a sphere? Do you squeeze them? Otherwise you get something like this. What is the nature of the energy they have? Are they spherical charged batteries? Flywheels? If they have the same electrical charge, you may need energy to bring them together. Apart from that, the law of conservation of energy says that the energy of the combined system should equal the sum of the energies of the original individual systems.  --LambiamTalk 17:14, 15 October 2006 (UTC)[reply]
Is this, by any chance, an incorrect analogy for atomic nuclei? ☢ Ҡiff 17:18, 15 October 2006 (UTC)[reply]

Triple point revisited

I read with great interest here 1.11 "Triple Point of Water", and got to thinking/wondering: What if we were to think of the problem using a single molecule of water, or perhaps better 3, single molecules of water...Can each molecule be in a different state simultaniously, and separately? Can one actually "have" a single frozen molecule of water? I would think that no matter what the temperature/pressure, a single molecule of water would ALWAYS be in the gaseous state. Is this true? 205.188.116.74 17:05, 15 October 2006 (UTC)[reply]

The various phases (solid, fluid, gas) are properties of a collection of molecules. It is meaningless to ask what phase an individual molecule is in.  --LambiamTalk 17:16, 15 October 2006 (UTC)[reply]

Decrease in Sexual Interest?

Does anyone know if pornography can cause a decrease in sexual interest, desensitivity to sex, and callousness towards intimacy and sex? I'm a really young guy, and I think porn may have caused some disinterest towards sex and woman when previously I was all about them. — Preceding unsigned comment added by The Truthish (talkcontribs)

If one takes into consideration how violent games can cause procedures to be learned, or arguably desensitivity to the consequences that violence or killing may cause, then there are limited reasons to say the same won't apply for pornography. Still, I only think that massive amounts of SEX will cause desensitivity to sex - and watching many pornographic pictures eventually builds up one's expectations to the standard of the contents of these pictures. Being a virgin myself, I am of course no authority. 81.93.102.3 19:22, 15 October 2006 (UTC)[reply]

I think it might raise your expectations to an unobtainable level so that only women who resemble porn stars would excite you: "Geez, I can't even see your ribs your so fat !". StuRat 19:34, 15 October 2006 (UTC)[reply]

Anyway, if you stop masturbating to the porn for a couple of weeks, you'll be fine. THL 19:36, 15 October 2006 (UTC)[reply]
I haven't heard of any studies that show a correlation. As far as your own personal experience, I think that that probably would've happened even without the porn. You've become more mature, is all. Ƶ§œš¹ [aɪm ˈfɻɛ̃ⁿdˡi] 22:04, 15 October 2006 (UTC)[reply]
Desensitisation is probably temporary.--Light current 00:06, 16 October 2006 (UTC)[reply]
Probably for the same cause and reason that orgasm creates sexual satisfaction. — X [Mac Davis] (SUPERDESK|Help me improve)04:27, 16 October 2006 (UTC)[reply]

Pornography does have the potential to do as you are saying. The article is a bit lacking in quality, but have a look at pornography addiction; also sexual addiction. BenC7 01:36, 16 October 2006 (UTC)[reply]

Well, the article says: "There is considerable dispute about whether "pornography addiction" actually exists, and if so, whether it has harmful effects." Or is it the presence of this statement that makes BenC7 state that the article is a bit lacking in quality? Even if you believe in such an addiction, it has to rise to a level that can be called obsessive, interfering with the rest of your life. The eager consumption of porn by a young person is fairly normal and can hardly be called an addiction.  --LambiamTalk 01:43, 16 October 2006 (UTC)[reply]

Alright, I may seem young and childish, but I'm an old fart and have been around the block a few times. It is temporary, just stop masturbating to the porn for a couple weeks whenever you feel this way, and you'll be fine. THL 05:18, 16 October 2006 (UTC)[reply]

It's not the porn itself, it's your frequent masturbating. Hands off. Or at least this would be my guess; I've never masturbated myself. -THB 08:47, 16 October 2006 (UTC)[reply]

HA HA! 90% of people admit to it. THe other 10% lie 8-)--Light current 13:51, 16 October 2006 (UTC)[reply]

what genes are in a genes pair

what two genes are in a gene pair??

The "genes" in a gene pair aren't genes at all; they're alleles. --David Iberri (talk) 19:47, 15 October 2006 (UTC)[reply]
Well, they may be (and usually are) DNA sequences that code for a gene, and then it is reasonable to call them "genes".  --LambiamTalk 22:39, 15 October 2006 (UTC)[reply]

There could be many Genes found in a pair of jeans (hopefully not all at once), for example, Gene Simmons. :-) StuRat 18:34, 16 October 2006 (UTC)[reply]

Identical alleles are found in what type of gene

Identical alleles are found in what type of gene?? Thanks

I expect you are interested in understanding the difference between hemizygosity, heterozygosity and homozygosity. Rockpocket 20:27, 15 October 2006 (UTC)[reply]

Science Fair

I am doing a science fair project for my school. The title of my project is: What foods attract ants? I need help on finding an If and Then hypothesis and a problem for my project. The project is very simple. I put differnt kinds of food, in different containers. I study which containers have the most ants around it, how long it takes for the ants to get to the food,and how long the ants will stay there. I know that the ants will probably go to the sugary foods. Bu I still need to write a If and Then hypothesis and a problem. If you could give me a suggestion, or any help with the project I would be very thankfuland very glad. -Sarah Baker

Okay, I think "If and Then" means your teacher wants you to include the reasoning behind your hypothesis in the complete statement. You predict the ants will "go to the sugary foods", but why did you make that prediction? What causes the ants to go to the sugary foods? For more information, see Writing Hypotheses and Writing a Hypothesis. —Keenan Pepper 23:26, 15 October 2006 (UTC)[reply]
Why are you using different types of containers? Usually you want to only change one variable at a time (i.e. the food), not two of them. How will you know what effect is due to the containers and what effect is due to the food type? I would make everything equal except for the food type. I would also try and distribute the foods semi-randomly (roll some dice?) to make sure that your bias going into it that the sugary foods will be more preferred does not lead to you to unconsciously make it easier for the ants to get to that food (i.e. by putting it closer to a known set of ants). In fact it would be ideal if you made the containers so that you the experimenter could not see what type of food was in them until after you had made your observations — that way there is little chance that your expected outcome will bias your observation (after all, maybe ants actually prefer turkey over candy — who knows? I've never really given them the option).
She said: "different containers", not: "different types of containers". You don't want to put all the different foods in the same container.  --LambiamTalk 01:13, 16 October 2006 (UTC)[reply]
The easiest "If/Then" statement you can do for this is simply, "If ants are given a choice of many types of food, Then they will prefer foods with high levels of sugar in them." --Fastfission 00:58, 16 October 2006 (UTC)[reply]
I agree that this may be meant by "If and Then hypothesis". But since you can't test all types of food, the generalization is probably not warranted. Perhaps the ants really love canned tuna (some actually do), or lasagna, much yummier than sugar, only it wasn't included in the experiment. Then you jumped to the wrong conclusion.  --LambiamTalk 01:13, 16 October 2006 (UTC)[reply]

I can think of much better ideas than what food attract ants. How about the relative attraction of fatty food versus fruits and vegetables for obese humans. You offer to pay for 1 days of fatty food or X days of fruits and vegetables. Find the variable X where it is in equilibrium with 1 day of fatty food by interviewing lots of obese people. Find the mean and standard deviation.

Why stop at fruit and vegetables? You can build up a hierarchy of food attractiveness for obese people. 202.168.50.40 01:02, 16 October 2006 (UTC)[reply]

If you want to be really scientific, you have to be specific about the species of ant (not all ants species have the same food preferences) and make sure, if your experiment runs over several days, that you put out the food each day at the same time (some species have a fixed daily schedule of what time to do scouting and what time to start fouraging).  --LambiamTalk 01:13, 16 October 2006 (UTC)[reply]

October 16

bananas

I have conducted a psudo-scientific experiment: I have left a bunch of bananas in my kitchen at room temperatue, and i have a single banana i wanted to eat in my bedroom. The skin of the one in my bedroom turned brown at an accellerated rate (and tasted jolly nice too). Why is this? The bananas were regular supermarket fayre and not organic or free range. I have been playing che séra séra on a number of occasions and have not mastrubated recently. Why would this be? --86.133.176.75 00:18, 16 October 2006 (UTC) -[reply]

Maybe they need to be kept in the dark.8-|--Light current 00:32, 16 October 2006 (UTC)[reply]
Well, gaseous emissions are known to affect the speed of fruit ripening and decomposition; is there something you're not telling us about your bedroom? Anchoress 02:03, 16 October 2006 (UTC)[reply]
What is a free range banana, and where do you get such things?  :) --best, kevin [kzollman][talk] 02:12, 16 October 2006 (UTC)[reply]
My guess is that the banana wranglers have to chase them down. I understand they roam the same territory as the cat herds. Clarityfiend 02:36, 16 October 2006 (UTC)[reply]
Doods, you're all wrong! They're called 'free range bananas' because they give them out FREE at the DRIVING RANGE. Anchoress 02:38, 16 October 2006 (UTC)[reply]
Seriously, the wild ones are fast, shifty little beggars. One of their most effective defense mechanisms is to drop a peel, causing their pursuers to slip, and then they just...umm...split. Clarityfiend 02:49, 16 October 2006 (UTC)[reply]
Try this: Put a bunch of bananas in your bedroom and a single one in the kitchen and report what happens. I wonder wether removing a single banana from the bunch accelerates the ripening process. The needs of the many... ;) Maybe bananas don't decompose as quickly while still on the bunch?? Vespine 03:39, 16 October 2006 (UTC)[reply]
I have also heard that splitting a bunch can have an effect on accelerated ripening. (look at the singles in the shop) Also the way they are stored can have an effect. You are supposed to use a banana tree to hang them in your kitchen. Refrigeration does not help and is not recommended. Im sure light has more to do with it.--Light current 10:11, 16 October 2006 (UTC)[reply]
If your room is colder than the kitchen, that could have an effect. (I bought bananas once when the furnace was broken in the winter, and they were turning brown pretty fast.) - Rainwarrior 15:23, 16 October 2006 (UTC)[reply]
The ripening of bananas is affected by ethylene gas, a plant hormone, and by temperature. So you should look at differences between the temperature in the kitchen and the bedroom, and at ventilation, since less ventilation would allow ethylene to become more concentrated. (The best way to take advantage of ethylene, however, is to place the bananas in a paper bag. Ripe bananas can ripen other fruits using this method as well.) --Ginkgo100 talk · e@ 19:27, 16 October 2006 (UTC)[reply]

Critters in the stomach

Greetings. In 1954 this guy Palmer published a study where he looked at biopsies from over 1,000 patients' stomachs searching for organisms which had been reported off-and-on for the last 70 years. After extensive study he reported that he did not find any colonizing bacteria. We know now that he missed helicobacter pylori, but of course he didn't know that. From what I've read people seem to say he missed h. pylori because he didn't use the silver staining technique that apparently reveals h. pylori. Unfortunately, the stuff I'm reading is for specialists, and I'm kinda dumb. Can someone help me here? Why does silver staining reveal h. pylori when other staining techniques don't? (Is this even right?) Even better, can anybody say one way or the other if this was a serious error on Palmer's part, or in the 1950s, could he have reasonably decided this was not required? Thanks in advance. --best, kevin [kzollman][talk] 01:58, 16 October 2006 (UTC)[reply]

Well, staining only works if the thing that you're staining holds on to the chemicals used to stain it. Think about invisible ink squirted on a newly laundered white shirt. It should stain the shirt, right? However, as the ink evaporates, the shirt is left white. Microbiology has a lot of different stains used to look for bacteria. However, some bacteria (such as H. Pylori) do not take up some of the more common stains, just as a white shirt doesn't take up invisible ink. It took looking with the right kind of stain (silver) to actually see H. pylori. It wasn't a mistake - his report stated the stains that he used and so the true way to interpret his results is "there are no bacteria in the stomach which take up the stains tested by Palmer" rather than "there are no bacteria in the stomach." InvictaHOG 10:56, 16 October 2006 (UTC)[reply]
Thanks for the response. Could you be a little more specific about why H. pylori doesn't take up other stains and does take up silver? This is the bit that I don't understand. WRT proper conclusions, I understand that a more strict induction might have been better, but scientists use limited evidence to reach broader conclusions all the time. I'm wondering how out of line Palmer was in comparison to others at his time. In other cases would people looking for bacteria have used the silver stain? Would others have concluded there was no bacteria without using such stain? You see what I'm getting at? Thanks again! --best, kevin [kzollman][talk] 19:44, 16 October 2006 (UTC)[reply]
However, there was indeed a mistake, made by the medical community. Overlooking the fact that Palmer might have not seen all there is, the claim entered most textbooks that research had proven that stomach ulcers and gastritis are not due to bacteria and hence, prescription of antibiotics to treat them is not worth a try. Hence, it took quite a while until somebody looked again, and when they found something, Warren and Marshall had a really hard time convincing their colleagues that the textbook knowledge was wrong. Well, that's at least how I remember the story. Simon A. 14:06, 16 October 2006 (UTC)[reply]
Simon, thanks for the response. You indeed have the story right, and looking back it was indeed a mistake. At the same time, if Palmer had been right and people had followed your recommendation, we might now be looking back wondering why all those scientists wasted their time redoing what we already knew. :) Its a tough balance, and what I'm trying to figure out is if Palmer was out of line compared to other scientists at the time. Thanks again. --best, kevin [kzollman][talk] 19:44, 16 October 2006 (UTC)[reply]

Arm wrestling

I've heard that it's harder for long-armed people to win at arm-wrestling. How exactly are they disadvantaged? I would image it has something to do with torque, but I'm not sure how the muscles in the arm work. --Anon

Yes, it has to do with torque, but also leverage. Imagine holding a sledgehammer from the heavy end, horizontally, so that the handle was parallel with the ground. This would be quite easy, because the heavy part is closer to your body. But if you held it from the handle, with the heavy end far from your body, that would be much more difficult. So you can see that having long arms might make things difficult in a situation like arm wrestling. Also, the human arm is a third class lever. If you look at the diagram on the lever page, you can see that it is easiest to move the load on the far end of the lever when the load, fulcrum, and force are close together. This is analogous to a short-armed person. Gary 02:45, 16 October 2006 (UTC)[reply]

Can you identify this strange bug?

Hi, I live in New England and I have encountered this strange bug a few times. It can fly, and when you squish it smells strongly of apple. Here is picture of the bug, it is not my picture but the bug is exactly like this: http://i50.photobucket.com/albums/f305/Parkram412/bug.jpg If you can identify this bug, I would greatly appreciate it. Thank you.

You will need a clearer picture, I expect. BenC7 05:49, 16 October 2006 (UTC)[reply]

Yeah, it's a little hard to tell from the picture, but I think it might be some sort of kissing bug. In which case be very wary of it. I didn't think they got that far North, though. --Trovatore 06:32, 16 October 2006 (UTC)[reply]

  • What you seem to have there is some species of stinkbug. They're not too uncommon around the NE Unites States (as opposed to the kissing bug, which numbers exactly zero). From the image, however, I can't be much more specific. If it is such an insect, than it's not dangerous, unless you happen to be an apple, of which it is a serious pest species. – ClockworkSoul 15:41, 16 October 2006 (UTC)[reply]
I've seen this bug many times in extreme-southern Canada. From http://www.cirrusimage.com/bugs_stink.htm, is seems to be either a brown stink bug or a shield bug. --Bowlhover 00:42, 18 October 2006 (UTC)[reply]
I was under the impression that stink bugs are shield bugs. Actually I'm too lazy to read that article. – ClockworkSoul 01:58, 18 October 2006 (UTC)[reply]

Why is some skin immune from sunburn?

The palms of the hands and soles of the feet can be exposed to the sun all day at the beach, yet these areas do not show any sign of damage from solar radiation. Why is this so? And do these skin areas produce Vitamin D from sunlight? And in evolutionary terms, why isn't our whole body covered in this apparently better quality skin, wouldn't that save us a lot of trouble? I could do with that type on my bald patch a) for solar damage immunity and b) for those times when I scrape my head, painfully, against the underside of my car tailgate :0) --EdX20 05:33, 16 October 2006 (UTC)[reply]

Just as a small note, I can't answer the entire question. Skin cancers usually occur after someone has reproduced, and there is less evolutionary pressure to change that, for it would probably bring some disadvantage. Also note, that evolution has done something to combat burn, white people lost it when they went to cold regions. It is interesting though, as black people have white feet and hands. --liquidGhoul 06:14, 16 October 2006 (UTC)[reply]

Going out on a limb here, but I really kind of doubt it's true that palms and soles of feet don't burn, if overexposed. I think they just don't get much sun under ordinary circumstances. How often do you sunbathe palms up? But maybe someone who actually knows something about it will comment, so I'll shut up now. --Trovatore 06:59, 16 October 2006 (UTC)[reply]

I don't know if soles and palms get sunburnt, but they sure do suffer sun damage. I read an interview a few months ago with a dude who spent years in the tropics, and when he got some foot damage, I think rats chewed on his soles, he was advised to expose them to the sun for long periods to heal them and he later got skin cancer on the soles of his feet. Anchoress 07:10, 16 October 2006 (UTC)[reply]
Oh, it was John H. Groberg. Anchoress 07:11, 16 October 2006 (UTC)[reply]
They do burn. It's just they're not exposed to much sunlight. Feet are usually in shoes/socks. Even when tanning, feet sometimes end up in sandles, or you end up playing around with the sand with your feet. Plus, your soles lie verticle when you lie down, not horizontal. People rarely tan with their sole facing up directly to the sun. Same goes for palms. Natural position for a human hand is with the fingers slightly curled in. It takes conscience though to straighten your fingers to fully expose your palms, which people don't do when they are trying to get a tan.
If you stood on your head for long enough your bald-patch would probably cornify to some extent thereby giving you what you seek. As other suggest, you would still get sun burned though. From a theoretical point of view, palm and sole skin could be even more sensitive to sunburn. If memory serves, palms and soles of in dark skinned folks are often lighter than other parts of their body. This would mean less melanin to protect from UV and thus a greater risk of "burning". If you are A) Black or Asian, B) brave/stupid enough and C) have access to a UV transilluminator, you could (though really shouldn't) test this hypothesis out for yourself. Rockpocket 06:41, 17 October 2006 (UTC)[reply]

Diesel fuel life

How long does diesel fuel last? Say I want to bury a 50,000 gallon diesel tank and be set for life, will the fuel keep? Are there any hydrocarbon fuels that won't deteriorate over several decades?

We don't know for how long you aim to survive, but assuming it is less than 100 years I'd be more worried about the tank rusting through. And any engines you power with the diesel won't last that long and may be difficult to repair. Have you thought about medical emergencies? It's tough surviving on your own, without (naughty) nurses. Or should the fuel power a whole hospital?  --LambiamTalk 12:55, 16 October 2006 (UTC)[reply]
you liked the naughty nurse, huh ;)
I think he's betting on the idea that the price of fuel will only increase over time, so if he hordes it now he won't have to pay rising gas prices, or something like that. --Fastfission 15:03, 16 October 2006 (UTC)[reply]
Actually, the economics angle is where I'm comming from, for two reasons. First, I've heard that alternative energy (wind, solar and associated batteries and electronics) might cost more than just a big diesel tank for generators and vehicles. And second, the rising fuel costs, as you mention. It's a moot point however if diesel deteriorates too quickly. I would include a simple nitrogen purging system and want the fuel to last for 40 - 100 years.

The problem with your idea is that the tank will cost far more than the fuel, so there goes any savings. On the other hand, if you fill it and then sell it everytime the price goes up or down a dollar a gallon, then you could possibly make money. This is fuel speculation, and you don't have to store it yourself, there are companies that provide that service for you. StuRat 17:53, 16 October 2006 (UTC)[reply]

Appropopos another recent discussion here, see our pseudomonas aeruginosa article for another threat to your (hypothetical) diesel hoard.
Atlant 20:52, 16 October 2006 (UTC)[reply]
50000 gallons of diesel? Are you serious?? For that amount of outlay you could definitely fit yourself out with alternative power. Probably solar is the way to go I reckon, and an electic car, or at least a prius ;) .. I wouldn't even think about fossil fuels, in 40 years time you don't know what they'll invent, diesel could be obsolete, like leaded petrol.. I certinally wouldn't bet on it. Vespine 00:37, 17 October 2006 (UTC)[reply]

Most effective firearm

What is the most effective firearm for self protection? My application is for protecting a razorwire fenced compound from hoards of starving attackers after a nuclear holocaust or similar event. I'm only interested in weapons that are currnetly leagal in the US (I'm not one of thoes wackos that's into illegal weapons).

not at all a wacko... ;-) different strokes for different attacking folks, ain't nothing like a shotgun for close range, and a semi-automatic like the ruger M-14 with telescopic sights should take care of anything else. Xcomradex 06:52, 16 October 2006 (UTC)[reply]
(edit conflict)Is this North Korea thing bringing out the paranoia in everyone? --liquidGhoul 06:54, 16 October 2006 (UTC)[reply]
In stead, you're a wacko that's into legal guns? Oh, but you've got a great excuse. Of course, the starving hoards after a nuclear attack. Who wouldn't want to prepare for that? :) DirkvdM 07:26, 16 October 2006 (UTC)[reply]

If you have to ask this question, you probably are not the type of person who should be owning a weapon yet. Attend a firearms safety class offered by the NRA and ask your instructor about your specific needs.

If you have to ask this question, you probably are not the type of person who should be owning a weapon yet -- Why, what's wrong with asking a question? The US guarentees people the right to own weapons to protect thier liberty (in a well regulated militia). I know that in some civilized circles this is viewed as barbaric and is seen as not needed because civilization has (supposedly) moved past this point. However, a thousand years from now people will still be killing each other and taking their stuff. The most healthy societies will be the ones that still allow people to defend their liberty. I certianly am the type of person who should own a weapon - because I ask questions, have never intentionally harmed another human being, and I'm interested in self defense and liberty, not power. But this is a digression, and you haven't tried to answered the question.
Thats right, you shoot that nuke with your shotgun, that'll sort everyones problems. And your very wrong, the healthiest societies are the ones where people dont need to defend their liberites. Philc TECI 17:45, 16 October 2006 (UTC)[reply]
Thats right, you shoot that nuke with your shotgun, that'll sort everyones problems. -- Huh? what?
the healthiest societies are the ones where people dont need to defend their liberites -- Name one
The closest I can come to are countries like Germany, and scandanavian countries, where crime is so much less of a problem than in countries like america. Philc TECI 17:45, 17 October 2006 (UTC)[reply]
Oh Pleeese, this times a hundred. Germany and scandanavian countries have the exact same problems as we do, that of defending their liberty. And it has nothing to do with crime, I have no idea why you added that into the mix.

If you’re just using legal guns, you'll never survive an attack. Actually, if you’re just using guns period, you'll never survive. Get an electric fence that has enough voltage to electrocute those who touch it, and have a mine field for 50 yards outside of a 20 yard empty space outside of the fence. That is probably your best chance for survival, along with the ruger mentioned earlier for anyone who may get past. THL 13:45, 16 October 2006 (UTC)[reply]

good insight - thanks.
You might do better to just take Dr. Stephen Falken's strategy in WarGames: Move really close to a primary target so you don't have to worry about living in the aftermath. I'm pretty sure it works for me located where I am.
Atlant 16:42, 16 October 2006 (UTC)[reply]
Many people accet this for themselves, would you accept it for your children though, or would you try a little harder?

The better strategy is to build your bomb shelter underground and not tell anybody about it. That way, nobody knows you're there so nobody tries to steal your food. Hopefully, by the time your food is gone and you come back up, law and order has been restored. StuRat 17:48, 16 October 2006 (UTC)[reply]

Yes, I'm planning to use surplus shipping containiers and bury them, still need self defense mesures though.
How would law and order be restored if everyone just hides undergound in their shelters. Philc TECI 17:59, 16 October 2006 (UTC)[reply]
Not everybody would. Those surviving members of the military, for example. StuRat 19:58, 16 October 2006 (UTC)[reply]
Theyd rebuild cities in the hope that people would just emerge from the ground and leech off of them? You expect there would still be an organised military in nuclear holocaust, and that they'd have the charity to help people who do nothing but hide when it is a battle in itself to survive themselves? I must say I think you outlook on this sort of thing is very optimistic. Philc TECI 20:34, 16 October 2006 (UTC)[reply]
You appear to be trolling for a fight, Philc. I never said anything about them "rebuilding cities". I think it's a fair assumption that some members of the military would likely survive (as would some civilians), and would then set about re-establishing their authority, as much for their own protection as for that of everyone else. What do you expect they would do, just run about with their arms flailing wildly ? StuRat 02:57, 17 October 2006 (UTC)[reply]
Dont offend me please. Im not a troll, and I dont want a fight. But in nuclear holocaust, with a countries complete infrastructure destroyed, the military have no duty to help you. Not to mention a complete lack of resources. And the fact that they need to ensure their own survival ahead of yours. Philc TECI 17:43, 17 October 2006 (UTC)[reply]
It sure sounds like trolling for a fight when you claim I said the military would "help you" when all I actually said was that they would reestablish law and order. That would be done to help themselves, not you, but you might very well derive a benefit from it, anyway. StuRat 18:38, 17 October 2006 (UTC)[reply]
When did misenterpretation of someones point become trolling!? I'm sorry you felt it necesarry to bring this up, and interpret a difference in opinion as trolling, I think its really sad people cant just discuss things, accusing someone of trolling is somewhat an inflammatory move in itself, maybe so that you could claim any equally inflammatory action I may have taken as proof that I am a troll. Well I'm not. Its a shame, but I would like to end this discussion before you try further escalate it into an argument. Philc TECI 20:47, 17 October 2006 (UTC)[reply]
Theyd rebuild cities in the hope -- Oh sure, everything would be just Hunky Dory.
Yes, I was being sarcastic, you are infact, agreeing ith me. Philc TECI 17:45, 17 October 2006 (UTC)[reply]
If you are by yourself, I think you need much more in the way of passive defence. No gun on the face of the planet can save you if you can get flanked and over run and no amount of wire fence and barbed wire will stop a truck ploughing through it. I'd recommend an electric fence surrounded by a brick or concrete wall topped with barbed wire surrounded by a decent moat with a bunker on a steep elevated position in the middle.
To go to fantasy land and answer your question directly, I don't know what weapons are 'legal' where you live. In Australia, no automatic weapons are legal at all but I think that's where you should be looking. Also because of Hollywood, most people have a very distorted impression of how easy it is to shoot and kill people. Someone who is not experienced with weapons is not likely to hit anything man sized with a shot from a pistol over 20 meters maybe double that from a rifle, half it again in combat.. Truth is, anything that shoots will kill, I don't think the actual gun is as important as what you do with it. What I would look for is the cheapest, automatic rifle you can find and buy four, something like an AK-47 would be perfect, and MOUNT them in your bunker so their fields of fire overlap the entire 360 degrees around you. Mounting will greatly increase your chance of hitting something, also, make sure the optics are accurate and get red dot sights or laser sight!Vespine 00:24, 17 October 2006 (UTC)[reply]

He brings up a good point, trucks. Get trained in how to use firearms (and take anger management so you can resist any temptations), and take his advice on the accuracy accessories. I still like my minefield surrounding an electric fence, but you should probably have your entire perimeter surrounded by a modern Berlin Wall as well. Pray that you don't come up against a tank; I don't think that you would have a chance of stopping one of those; especially if they have ammunition for the big gun/rocket launcher. THL 04:22, 17 October 2006 (UTC)[reply]

If you upgrade a little you don't have to worry about any of those things, and you can keep your shelter above ground. Station automated Phalanx CIWS systems around your perimeter... but put a few Bushmasters in on top of the Berlin Wall for backup. Then, for yourself, you can use the Pancor Jackhammer automatic shotgun and the FN F2000 assualt rifle. — X [Mac Davis] (SUPERDESK|Help me improve)08:30, 17 October 2006 (UTC)[reply]

Humans driving a ship

If the people on a seaworthy ship would take turns driving the propeller with hometrainers (shifts of 8 hours per day), how fast would it go? I've got this vision of a gym in lieu of an engine room, with the slogan 'get a cheap passage, get a workout and save the environment'. Alternatively, how many people would it take to drive a cruise ship at a reasonable speed? DirkvdM 07:13, 16 October 2006 (UTC)[reply]

Ship diesels are huge and have thousands or even millions of horse powers. Given that you need several humans to replace a horse, you would need literally a whole city of people. On the other hand, galleys were quite fast, when powered by rowers with oars. So, that's an option. The reason why a galley can be propelled by humans and a cruise ship cannot indicates the main problem: you'd have to do without sun deck, swimmin pools, squash rooms and dining halls. Actually, galleys came out of fashion already during the early age of sail. It seems to me that you even have to do without heavy cannons. But the concept of a cruise on a galley has bee explored in an Asterix volume. Simon A. 07:39, 16 October 2006 (UTC)[reply]
And propellers are not very efficient at slow speeds. People-powered ships would get along better with oars or paddle-wheels.--Shantavira 11:34, 16 October 2006 (UTC)[reply]
Are you referring to the speed of the propellers or the vessel? If the former, couldn't you just use gears to speed it up? —Bromskloss 12:38, 16 October 2006 (UTC)[reply]
Then you would need to have even more people to drive it. For example, if you geared it up to 10 times the speed, you would need 10 times as many people, or perhaps more, due to increased drag. StuRat 17:39, 16 October 2006 (UTC)[reply]
Its not going to happen, it takes about an hour to get across a bay in a pedalo, you'd be better off conserving the energy and drifting, that way you might actually live long enough to get across. The power ships use is phenominal, in a series of excercises I think my horsepower was measured between 0.7~1.1 (I cant remeber) and if you need a million of these horsepowers.... Philc TECI 17:43, 16 October 2006 (UTC)[reply]
The US Navy converted an old sidewheeler steam paddle boat into a training aircraft carrier, the Wolverine in Lake Michigan during WWII, with 8,000 horsepower producing 28 mph claimed. Figure 1/8 HP per person, and it would take 64000 passengers, somewhat more than it could carry. The Confederate Civil War sub H. L. Hunley (submarine), powered by 7 men, could go 4 knots on the surface. A galley with 50 oarsmen could reportedly go 9 knots.A racing shell can probably go faster with a smaller crew, but it is hard to decode the abbreviations in Rowing World Records, and you were not discussing rowing a lightweight craft anyway.Edison 17:57, 16 October 2006 (UTC)[reply]
The world record for an 8-man racing shell (the fastest commonly used boat) equates to just over 12 knots over 2000 metres. But the difference being that the combined weight of the crew is massively more than the boat itself. For a crew made up of 90kg oarsmen and 55kg coxswain - a total of 775 kg, not far short of 8 times the weight of the boat. To achieve this with a cruise ship, e.g. the RMS Queen Mary 2 - you'd need nearly 7 million rowers on board (but you'd have far exceeded the power of the QM2's engines (and the QM2's load capacity).Richard B 23:56, 17 October 2006 (UTC)[reply]
A professional cyclist can churn out 300 to 350 watts (1 HP = 750 watts) over a period of several hours and much more over a shorter time frame, but he trains 20,000 miles a year to be able to do that. Your average, beer-bellied, sedentary cruise customer would be lucky to produce 100 watts for more than a few minutes, so I fear this idea is doomed. EdX20 03:31, 17 October 2006 (UTC)[reply]
Yep. Put it this way - you'd need about 7,500 cyclists at to move this 59-foot motor yacht at its top speed. You could probably get away with 1500 or so to move it at its hull speed of about 10 knots, though :)
If you want a boat that you can propel efficiently with cycling - this does the job nicely. --Robert Merkel 06:50, 17 October 2006 (UTC)[reply]
I was indeed thinking of galleys and figured that an oar is a much less efficient way to use a human body's enery than pedals are. But if propellers are less efficient at low speeds (never heard of that), that is a problem. Also, galleys went fast, but only for a short timespan, not quite enough to cross the Mediterranean or even an ocean. And the cruise ship idea was a bit wacko. But what I was thinking about is a lightweight boat that is big enough to be seaworthy and has only basic amenities - just the engine room and sleeping bunks. Of course sails make sense, but they shouldn't require a professional crew. The idea is to make a very cheap passage for budget travellers to places that are expensive to fly to. Robert's cadence link doesn't say how fast it goes, but something similar for 100 people would be much more efficient and therefore faster. There would be enough budget travellers interrested in this, priovided it doesn't take too long. Say one week to cross the Atlantic. West Africa to Brazil is about 3500 km, so that would require a speed of 20 km/h or 10 knots. Given the answers above, that would be pushing it. And more than one week of constant cycling would be too much even for most budget travellers. So the idea is pretty much out the window, I suppose.
A siling ship would still be an option, though. DirkvdM 09:27, 17 October 2006 (UTC)[reply]

Unknown Goat

Unknown Goat

I think this would add value to an article, but I am not sure what type of goat it is. Does anyone know? NauticaShades 12:17, 16 October 2006 (UTC)[reply]

Hehe, reminds me of a perhaps well-known story.

An astronomer, a physicist and a mathematician were holidaying in Scotland. Glancing from a train window, they observed a black sheep in the middle of a field. "How interesting," observed the astronomer, "all Scottish sheep are black!" To which the physicist responded, "No, no! Some Scottish sheep are black!" The mathematician gazed heavenward in supplication, and then intoned, "In Scotland there exists at least one field, containing at least one sheep, at least one side of which is black."

SourceBromskloss 12:31, 16 October 2006 (UTC)[reply]
If I had a coat like that, Id feel a bit sheepish(Or sue the dry cleaners) 8-)--Light current 13:55, 16 October 2006 (UTC)[reply]
It is a beautiful goat. I would say that it is an Angora goat, but it could also be a cashmere goat. I looked up some photos and it most resembles Angoras. We used to have a goat dairy, and had a few Angoras, but I don't know how to differentiate the breeds outside of dairy goats. I can even tell crosses to some degree. --liquidGhoul 14:37, 16 October 2006 (UTC)[reply]

pistons in combustion engines

i would like to know why all the pistons in combustion engines are cylindrical...why cant they be in some other solid shapes like cuboid, prism etc.... Siva sankar 12:45, 16 October 2006 (UTC)siva[reply]

Good question. Three tentative reasons. First, the piston has to fit the bore of a cylinder snugly, and if that's not cylindrical they'd have to come up with a new name for it (a six-cuboid engine?). No that's silly. Start over. First, it's easier (cheaper) to manufacture up to the required precision, especially the bore of the cylinder. Two, a chamber of cylindrical shape is best able to withstand the pressure. Three, if there is accidental torque on the piston – and there's always some – the cylindrical shape is the only one in which this does not result in potentially damaging friction between piston and cylinder. By the way, pistons are also cylindrical in steam engines and other heat engines, or, more generally piston engines.  --LambiamTalk 13:13, 16 October 2006 (UTC)[reply]
May be it is also easier to seal a cylindrical piston rather than the one with angular corners? The gases can escape from the improperly sealed corners... -- Wikicheng 13:17, 16 October 2006 (UTC)[reply]
Yes It would be difficult to make polygonal piston rings--Light current 14:00, 16 October 2006 (UTC)[reply]
I think you'll also find that a circular piston sweeps the largest cylinder volume with the least contact area around the skirt (and rings) of the piston. So from the point of view of minimising sliding friction, the circular piston is the winner. The same also holds true for the heat loss from the combusting gases. The cylindrical "cylinder" has the least amount of wall surface area to drain heat from the combusting gases. This also raises the efficiency of the engine. Heat loss from their narrow, high-surface-area combustion chambers is one of the reasons why Wankel engines aren't very efficient.
Atlant 14:04, 16 October 2006 (UTC)[reply]
But 'maybe' not so good for removing the heat from the hot piston. THis is one of the functions of piston rings apparently. 8-)--Light current 14:11, 16 October 2006 (UTC)[reply]
Engines often have specific oil jets aimed at the bottoms of the piston (in order to remove heat from the pistons). Me, I'm looking forward to ceramic adiabatic engines with no cooling systems. It ought to be very cool to see these glowing when you raise the bonnet.
Atlant 14:19, 16 October 2006 (UTC)[reply]
Oil jets. Sounds cool!--Light current 22:09, 16 October 2006 (UTC)[reply]
Stress is channelled into corners, and experience a lot more of it than smooth parts of the same shape, this is why a lot of high stress thnigs are round, spacecraft, missiles, gun barrels, etc.. Philc TECI 17:38, 16 October 2006 (UTC)[reply]
There have been some reciprocating engine designs that have used something other than a cylindrical cylinder. The Honda NR had an oval-shaped piston (and cylinder bore). However, this bike was designed to meet racing rules which restricted the number of cylinders the bike could have. The oval pistons allowed a larger valve area with this limited number. The Formula One rule makers promptly banned the technique from their competition, so it's never been seen in cars. Absent such regulations, the benefits are so marginal it's not worth doing, apparently. --Robert Merkel 23:48, 16 October 2006 (UTC)[reply]
I suspect it's also an easier shape to manufacture. RJFJR 20:09, 17 October 2006 (UTC)[reply]

Epidermis

What is the layer of the epidermis where the cells are considered protective but nonviable? Tyler Argue

Nonviable? You mean, dead? The stratum corneum. Simon A. 14:12, 16 October 2006 (UTC)[reply]

Fuel consumption

In an effort to improve fuel consumption, I have pumped up the tyres to the max recommented pressure, and am now experimenting with driving in high gears most of the time. I have 5 gears and the 5th is only supposed to be used above 34 mph. Will I damage the engine if I go slower that this in 5th?--Light current 13:57, 16 October 2006 (UTC)[reply]

With regard to tyre pressure: Take care that you don't wear out the center of the tyre tread abnormally early. Excess inflation (for a given weight load in the car) tends to make the center of the tread bulge, leading it to ear out first. For the same reason, over-inflation may reduce the tyre's traction (grip) somewhat. But it's certainly good for mileage.
With regard to engine RPM: Your owner's manual will probably specify a minium RPM for the engine, but that usually means "under load". You can certainly coast (or run very lightly loaded) below that minimum RPM with no risk of engine damage. Usually, cars are pretty good about letting you know when you're trying to load the engine too heavily at too low an RPM: That shuddering and bucking lets you know you're "lugging" the engine. (Is "lugging" an Americanism?)
Atlant 14:09, 16 October 2006 (UTC)[reply]
Seems like it is. Over here we use the term 'labouring' for running the engine in too high a gear. I always thought this practice used to hammer the big ends--Light current 01:55, 17 October 2006 (UTC)[reply]
Also, my handbook (Ford) doesnt seem to say much about RPM: just gives the recommended speed ranges in each gear. I know the engine idles about 900 RPM nad I can get it to go without acceleerating at 1500 RPM in top gear.

I had a friend ruin his transmission by letting it "lug" in 5th gear driving at slow rates of speed. You should be able to hear/feel your vehicle vibrate when you do this. It definately is hard on your tranny. Tyler

You've seen Fuel economy in automobiles and Fuel efficient driving, right?
Atlant 14:38, 16 October 2006 (UTC)[reply]

How you drive can also have a major effect on mileage, particularly in stop and go traffic. Here are some things I do:

  • Slowly accelerate, don't floor it.
  • Rather than braking, try to time traffic lights and let the car decelerate so you don't get to the light until it's green.
  • In heavy traffic, leave enough room in front so you don't have to brake every time the car in front brakes. This might result in people cutting in front of you, so you need to have an easygoing temper to do this.
  • Allow your speed to vary on hills, slowing down as you climb the hill and speeding up and you descend. This may be illegal in many areas with tight minimum and maximum speed requirements, however. Ironically, those maximum speeds put in place to save gas frequently cost gas instead (petrol to you Brits !). StuRat 17:05, 16 October 2006 (UTC)[reply]
Thanks Stu. I did most of that already. However, I have now started driving 'one gear up' on what I used to and get into 5th at about 30 - 35 mph. THis doesnt labour the engine if Im not accelerating. Below 30 I need to go into 4th at highest.--Light current 06:56, 17 October 2006 (UTC)[reply]
That certainly is a first - putting in a dialect note for UK people. Thanks for noting it, but it seems we're all so used to "that's gas for you Americans"'ing we're pretty fluent in the majority of US specific words, shame it's not the other way round, eh? Benbread 18:48, 16 October 2006 (UTC)[reply]


I can write an entire article about the mathematics of automobile fuel comsumption but my time is short. In short, it depends on what type of car you drive. Different techniques for different cars. You must always target the main cause of fuel consumption first. 202.168.50.40 23:21, 16 October 2006 (UTC)[reply]

For the majority of Americans, the main cause of fuel consumption is always MASS.

When you are accelerating your SUV the following occurs

1st Law: F = M * A
Energy: F * Dist = M * A * Dist
Power: F * d(Dist)/dt = M * A * d(Dist)/dt
Power = M * A * V

strictly speaking

Power(t) = M * A(t) * V(t)


In short, the main cause of high rate of fuel consumption is Mass times Acceleration times Speed.

  1. Reduce your mass
  2. Reduce your acceleration
  3. Reduce your maximum speed

Noticed that I did not talk about air resistance, that is because the main usage of engine power (in city driving) is overcoming MAV and not air resistance. Within a city, air resistance is not the main cause of fuel consumption. 202.168.50.40 23:36, 16 October 2006 (UTC)[reply]

Well apart from ripping some body panels out and swapping the engine for a lighter model, Im not sure what can be done about mass. I do carry round a 56 lb speaker cabinet, But I dont think that really makes much difference to the total mass of the car plus driver.--Light current 07:04, 17 October 2006 (UTC)[reply]

Bacterial Infection

What is the name of the bacterial infection that enters the blood and is often found in burn patients? 11:20 A.M. Tyler

Pseudomonas?
Atlant 14:28, 16 October 2006 (UTC)[reply]

Don't think its that, pseudomonas is most often acquired through the fecal-oral route when the infectee is near cows and causes intense GI issues. Do you know anything else about the bacteria you are asking about? pschemp | talk 14:38, 16 October 2006 (UTC)[reply]

No, just that it enters the blood and is often found fatal in burn patients. I don't think it's pseudomonas though, my text book says that occurs in the ear and can be treated with ear drops.

I suggested Pseudomonas because, in the book 365 Days (by Ronald J. Glasser, an American military doctor writing about his tour of duty in Viet Nam), that was the opportunistic infection that seemed to mark the end for many of the doctor's patient soldiers.
Atlant 14:46, 16 October 2006 (UTC)[reply]

Well it could be lots of things, especially the antibiotic resistant variants of bacteria like Staphylococcus aureus which is one of the causes of bacteremia. pschemp | talk 16:44, 16 October 2006 (UTC)[reply]

The correct answer would be Pseudomonas aeruginosa. Sepsis is a problem generally in hospitalized patients, and of course other bacteria, such as Staphylococcus aureus are very important concerns, but the organism associated specifically with burns in hospitalized patients is Pseudomonas. The list of organisms isolated from necrotic tissue in burn patients is, in order of frequency: methacillin resistant Staphylococcus aureus (MRSA), Pseudomonas aeruginosa, Enterococcus, Proteus species, Escherichia coli, Candida albicans and Acinetobacter calcoaceticus anitratus. Part of the reason for the affinity of Pseudomonas for burn patients is that it is often found colonizing accumulations of water (in hydrotherapy pools, ventilators, and other items found in burn units), and that the skin's ability to prevent infection has been effectively neutralized by the burns. Yes, when Pseudomonas infects the external ear canal it is called swimmer's ear and is treated with topical antibiotics, but that doesn't mean it doesn't cause other, serious or fatal, infections. - Nunh-huh 18:54, 16 October 2006 (UTC)[reply]
Yes, that's it, and it smells like rotten fruit. You can smell a pseudomonas infection when you walk in the room. -THB 03:23, 17 October 2006 (UTC)[reply]

Compact Bone

Is compact bone replaced faster than spongy bone? Susan 11:23 Am October 16/06

Hormones

Several hormones control the remodeling of bones. Which two respond to the changing of calcium levels? Tyler 12:07 16 October 2006

Tyler: I answered your question on the epidermis by simply searching for it, and I'm sure, you can do the same yourself. So, please read the top of this page: Do your own homework!. So please look through the articles on bones and hormones, check your textbook, and if you then still cannot figure out your homework question, come back and ask again. Simon A. 16:06, 16 October 2006 (UTC)[reply]

Science Fair Proposal

In my science class we are expected to creat a proposal for a science project with a controll and 2 variables. I already have that but on the paper it asks for Research in the following way which I do not understand.

Research: At least two preliminary sources of information, correct MLA format

WHAT IS MLA? Devol4 15:09, 16 October 2006 (UTC)[reply]

MLA is just the style or format that you write your paper in.

But what style is it?? Devol4 15:13, 16 October 2006 (UTC)[reply]

Have you searched for the term? If you enter MLA into the search box, you'll find the disambiguation page MLA which points you to MLA Style Manual. I suppose the latter article should help. It seems your teacher wants you to list your two sources in the format recommended in MLA Style Manual#Citation. Simon A. 16:07, 16 October 2006 (UTC)[reply]

MLA is just a way to write a citation for a book or newspaper article or magazine article. If I had a book called "A Great Book About Books" which was written by A.J. Funkhouse and published by the New York-based Alimony Press in 1923, my MLA citation would be: Funkhouse, A.J. A Great Book About Books. New York: Alimony Press, 1923. It is just a way of writing out a citation, one way of many. --Fastfission 01:47, 17 October 2006 (UTC)[reply]

Dodgy Bike Chain/Sprockets

My bike has hub gears but the sound that the chain makes as it goes round, sounds like deraillier where the gears aren't calibrated properly and the chain wants to change gear. It also feels "grindy" and occasionally jumps (my foot jerks forward while pedalling). I've taken the chain guard off but I can see what could be causing it. Any ideas? --Username132 (talk) 16:58, 16 October 2006 (UTC)[reply]

The obvious answer is a mis-calibrated deraillier. Check to see if the sound is coming from the metal which actually shifts the gear. If there is not sufficient clearance (it should not be rubbing at all), that is likely you problem. Other than that, one commonly overlooked cause of difficulty is a frozen link in the chain (which in my experience causes jumping, but rarely grinding sounds). If the cogs are new and chain is old (or vice versus, or potentially both old), there may just be slipping due to wear on the cogs or a stretched chain. My experience is this is most common when one of the elements was recently replaced. --TeaDrinker 19:20, 16 October 2006 (UTC)[reply]
Once in a while, you'll also find that the chain has gotten "stiff" and so individual links don't "hinge" smoothly. This produces symptoms that partially mimic a badly adjusted derailler but it's easy to confirm or deny this diagnosis: Just see if the entire chain flexes smoothly.
Atlant 20:18, 16 October 2006 (UTC)[reply]
i fixed a similar probelm a few days ago with this site[27]. click the "Derailleur, rear tuneup" option and it'll show you whats needed. Xcomradex 20:54, 16 October 2006 (UTC)[reply]
The question is about a hub gear, not a derailleur. Not sure if hub gears are user-serviceable, if indeed the problem lies within it. EdX20 04:36, 17 October 2006 (UTC)[reply]

Hey, I just discovered Hub gear! Looking at the diagram, they are impossible to service! You have to replace them. --Zeizmic 12:08, 17 October 2006 (UTC)[reply]

Check this out: Hub assembly video (MP4)
Atlant 23:18, 17 October 2006 (UTC)[reply]

To reiterate, the problem exists on a hub gear bike BUT this problem applies to the FIXED gears that exist between the pedals and the rear wheel. Next time the chain falls off (whenever the bike falls over), I'll check each hinge in the chain. --Username132 (talk) 13:54, 17 October 2006 (UTC)[reply]

frequency / function generator? Buying one?

how much and where?--64.12.116.74 17:47, 16 October 2006 (UTC)[reply]

You can buy new from any number of suppliers on the internet. Price can go as high as you can afford, depending on features and specifications. Ebay.com has lots of test equipment. Search "function generator" and there are over 100. Ones which say 'as is' are about $25. They might or might not work. Ones which claim to be in good working order seem to be $75 and up. Register, and you can check completed sales to see what the actual sale price was for a given model. I would avoid old tube type clunkers and kits unless you want to tinker. Older ones may not go to as high a frequency. You must determine what functions and specifications you might want. Check the seller's feedback ratings to see they have lots of sales of equipment such as you are buying, with close to 100% satisfaction. Check for shipping charges and don't be afraid to ask questions before bidding, and for the lowest price, "snipe" by bidding your maximum price in the last minute before the sale closes. Good luck. Edison 18:11, 16 October 2006 (UTC)[reply]
Be sure you understand what specs are important to you as well. If you're going to be doing testing and repair of audio equipment, you don't need frequencies above (say) 100 KHz, but you probably do want a nice, clean low distortion sine wave and you may very well want a square-wave output as well. If you're going to be doing radio frequency work, you'll want whatever carrier frequencies and modulation schemes you'll be using. With either, you'll also need to decide if you need swept-frequency capability, remote (computer) control, and all the fancier bells-and-whistles.
Once you've settled on what you really need, your choices will be more-clear.
Atlant 20:25, 16 October 2006 (UTC)[reply]
Ever considered building one? THe MAX 038 [28] function generator chip goes to 20MHz, sine, sqaure, trangle etc and you dont need many external components (just some timing Cs a nd a pot or two). A real gem of a chip from MAXIM. (or so it appears from the data sheet). Googling will get you the data sheet plus application notes.

Pickled food and tooth decay

Does pickled food contribute significantly to dental caries? I know some pickles contain sugar, but I am asking only about the acid they contain directly, assuming a low sugar food such as pickled onions without added sugar. According to the article, "Enamel begins to demineralize at a pH of 5.5", while the vinegar article gives the pH of pure vinegar as typically around 2.4. But I don't know about the pickled food after draining. Thanks. Arbitrary username 21:34, 16 October 2006 (UTC)[reply]

That doesn't sound right, 7 is Neutral, 5.5 is only slightly acidic, 2 to 3 is lemon juice and vinegar.. I'm no dentist but I'm guessing here that the "demineralisation" you are talking about isn't actually a problem, it probably only occurs in very very tiny amounts and your saliva would act to neutralise the acidity almost straight away, I'm guessing the effect is negilegable. The REAL thing you have to worry about on your enamel is Dental_plaque which is caused by bacteria and has a constant effect on your teeth if allowed to build up. Vespine 23:03, 16 October 2006 (UTC)[reply]
Finally, a question I can understand! :-P The pH is important because with a pH lower than 5.5 the minerals in enamel are not resistant to the bacterial advancement found in plaque. Further, when you eat something like sugar which result in bacteria producing lactic acid, the mouth maintains an acidic pH for about 30 minutes. During this time, tooth structures like enamel and cementum remain vulnerable. Now, if you increase the time throughout the day in which the teeth remain vulnerable, such as snacking on candy throughout work, your teeth will be more likely to develop dental caries. On a similar topic, sucking on lemon juice often also demineralizes tooth structures because of the low pH, and over a long period of time it can cause the loss of tooth structures. This would be known as erosion. - Dozenist talk 23:57, 16 October 2006 (UTC)[reply]
Thanks for the further info, but I'm afraid I haven't quite followed what conclusion I should draw from it about pickled food. Please could you clarify. Many thanks. Arbitrary username 12:23, 17 October 2006 (UTC)[reply]
Although acid foods are thought to contribute to enamel erosion, the consensus is that they do not play a role in the initiation or propagation of dental caries. Admittedly, this may be counter-intuitive, since the initiation phase of caries involves a demineralization of tooth enamel from the acidic metabolic by-products of bacteria.--Mark Bornfeld DDS 17:56, 17 October 2006 (UTC)[reply]
Thank you very much for the explanation. Would you say that, all things considered, the enamel erosion directly from acid foods is itself a significant enough consideration to make it worth limiting one's consumption, given what you say regarding lack of link with caries? I ask because in other respects unsugared pickled vegetables would appear to be a healthy snack. Thanks. Arbitrary username 12:13, 18 October 2006 (UTC)[reply]
I would say all things considered, saliva would act to neutralise the acidity of pickled food faster then it could effect your teeth significantly. Acidity of those levels is only detrimental when it is allowed to build up under a layer of plaque and affect the enamel for significant periods of time. Interestingly, I also found out that the acidity of a certain female bodily fluid is also below the tooth enamel threshold, the intake of which I would not like to limit my consumption of in fear of tooth erosion. ahem;) ... Vespine 01:50, 19 October 2006 (UTC)[reply]
Actually, saliva is quite limited in its buffering capacity. If it were all that effective in that regard, one would not see dental erosion. In fact, erosion is quite common, and may be due to various acidic foods (citrus juices, acidified soft drinks or vinegar, for example) or refluxed gastric contents-- most visibly in cases of bulimia. Short of outright erosion, acid exposure of cemental root surfaces that have become uncovered due to gum recession can exacerbate dental hypersensitivity. So, although it would be churlish for dentists to ask their patients to forgo all pickled vegetables (giardiniera is one of my favorites), it would not be wrong to place some limits on the frequency of their consumption. Remember-- anything in excess is--- excessive.--Mark Bornfeld DDS 02:55, 19 October 2006 (UTC)[reply]
Oh yeah, I apologize for not answering the question directly about pickled foods, but alas I do not claim any knowledge about pickled foods in particular! :-P And I agree with Bornfeld, saliva is not usually enough to protect teeth all the time from acidic foods. - Dozenist talk 03:01, 19 October 2006 (UTC)[reply]

Water

Does anybody know if water can change directly from a gas to a solid, without first becoming a liquid? I saw a diagram of the changes of states of water in my textbook, and it doesn't show gas to solid, and I'm curious. Thanks. Яussiaп F 22:27, 16 October 2006 (UTC)[reply]

Phase transition calls it deposition. Err, that's all I know, but the article does mention that it happens with snow and whatnot. -- Consumed Crustacean (talk) 22:36, 16 October 2006 (UTC)[reply]
If we assume that the phase diagram here[29] is correct, then at pressures rather lower than we're used to you can find water ice subliming directly into water vapor and vice-versa.
Atlant 22:41, 16 October 2006 (UTC), revised 13:54, 17 October 2006 (UTC)[reply]
He was asking about water vapour -> ice ;). -- Consumed Crustacean (talk) 22:46, 16 October 2006 (UTC)[reply]
The reaction is reversible ;-). Atlant 13:47, 17 October 2006 (UTC)[reply]
Once upon a time refrigerators were not frost-free. Every month or so the owner had to turn it off and scrape off or let melt the buildup of ice in the freezer compartment, which could get an inch thick if the defrosting was neglected. This was an example of water vapor (from the air, from open liquids, veggies, ice cream, meat, etc) freezing direcly from vapor phase (gas) to ice (solid) on the cooling coils, which were kept around -18 °C (0 °F). In a modern frost-free unit, I have seen the opposite happen: I fill the ice trays, go back a month later and there is only a fraction of the thickness of ice, because the automatic defrosting has melted the frost on the cooling coils and drained it away, but the solid ice has sublimated, and become vapor in the arid freezer (thence deposited on the cooling coils) without going through a liquid phase.Edison 23:35, 16 October 2006 (UTC)[reply]
This is an odd question, because I'm just now learning about this in school. Basically, this is how frost and snow forms(frozen dew is not frost). AMP'd 02:46, 17 October 2006 (UTC)[reply]

question about microwaves [EM really] and momentum transfer

I one could create a machine to focuss micro-waves to cause large scale momentum and energy transfer in air molecules, would there be and reaction on the emitter itself, i would think not right? because the momentum transfers after the emisions, this would make the EM drive unworkable as a flight system wouldnt it, have read the section but just checking if this was the problem, even if it hasnt got stuff to do with please answer question anyways curious

There's a reaction on an antenna when it produces EM radiation, and on a mirror or lens when it redirects such waves. (For the antenna, consider the system of the antenna and (eventually) the radiation from its center of gravity; it has 0 momentum to start, so it must have 0 afterwards, but the radiation has some in one direction so the antenna must have some in the opposite direction.) So you would in fact get the drive you want before the interaction with the air, and the air isn't even necessary: you can strap some lasers on the back of your spaceship and literally ride light around (well, push off of it anyway). However, photons carry very little momentum for their energy, so such engines tend to have efficiency problems. --Tardis 23:36, 16 October 2006 (UTC)[reply]
I actually think light propulsion systems (eg. using a laser) would have extremely high efficiencies - the problem would be the maximum power - if you tried to pump a gigawatt through a pen laser, it would just explode - however, öthe milliwatt that pen lasers do shoot out the end is probably transferred very efficiently into kinetic energy. You just need a super-high-capacity laser. --18.214.1.158 05:09, 17 October 2006 (UTC)[reply]
See nuclear photonic rocket, which I recently rewrote to address some of these questions. Besides, the energy of the photons that you eject does not correspond to the kinetic energy that you get in any direct fashion; energy is not a vector quantity, so there are no third law considerations. --Tardis 20:02, 18 October 2006 (UTC)[reply]

I can't understand your question (learn to use sentences, they are your friend !). However, maybe you are talking about a high power ground station sending microwaves to the underside of a spaceship, as a way to launch the spaceship. I believe this is a viable method, but does have quite a few drawbacks, like potentially cooking any occupants. StuRat 23:50, 16 October 2006 (UTC)[reply]

StuRa: He asked whether there is a reaction (physics) force on an emitter of electromagnetic waves. The answer is Yes, as Tardis already pointed out. EM waves have a momentum, but in most cases the momentum transfer is to tiny to notice. Simon A. 12:55, 17 October 2006 (UTC)[reply]
Additional remarks: For an example where the momentum of EM rardiation becomes significant, see the Mössbauer effect and its application, Mössbauer spectroscopy. For the idea to use the momentum transfer due to the reflection of light off a mirror to drive a spacecraft, see solar sail. There, it seems we are really talking about using the momentum of light in the visible frequency range. For the idea to use the momentum of photons emitted by the spacecraft, this has also been discussed, but as the power required would blow apart a laser device, they envisioned a nuclear reactor: see nuclear photonic rocket. (I just read about this for the first time in the article, so no idea whether it is accurate.) Whithin an athmosphere, there is also the idea to let a spacecraft absorb a collimated microwave beam (maybe from a ground-based maser) which heats up the air below the aborption plate. The air then pushes the aircraft further on. I think they did somewhere some not too succesful experiments but I forgot how they called it. Simon A. 13:07, 17 October 2006 (UTC)[reply]

Angle of reflection for light.

This is a homework problem I'd like to help a student with (I don't want the answer, but how to do it):

A beam of light is emitted in a pool of water from a depth of 65.0cm. How far away, relative to the spot directly above it, must it strike the air-water interface in order that the light does not exit the water?
I am really confused on this problem. I've started it by using the equation nsin(theta) = nsin(r).
I know that the index of refraction for water is 1.33 and air is about 1. But I don't know where to go from there? I'd appreciate any help.

How do we do this problem ? StuRat 23:29, 16 October 2006 (UTC)[reply]

See total internal reflection. In short, you want to find the angle of refraction to be 90 degrees (or worse, to be undefined), so the light never really makes it out into the other medium. You also want to make sure you have Snell's law correct; I don't understand your notation, although the indices are right. --Tardis 23:39, 16 October 2006 (UTC)[reply]

Thanks ! StuRat 00:22, 17 October 2006 (UTC)[reply]

The answer is 73.70cm :-). I suppose that you'll still need to show the calculations to the teacher. So this shouldn't hurt.--Wikicheng 06:50, 17 October 2006 (UTC)[reply]
Then you replaced 1.33 by 4/3. If you use 1.33 the answer is more like 74.13 cm. Our page List of indices of refraction gives 1.333, which is closer to 4/3 and produces the answer 73.75 cm. You can round all to 74 cm.  --LambiamTalk 14:37, 17 October 2006 (UTC)[reply]
Your expression for the angle of refraction will contain an arcsin. You want the argument of that arcsin function to exceed 1. Arbitrary username 12:28, 17 October 2006 (UTC)[reply]

When I do the math, I get 57 cm:

theta = arcsin(n2/n1)

theta = arcsin(1.00/1.33)

theta = arsin(0.752)

theta = 48.75 degrees

     x
  +-----+ <-water
  |  49/    surf
  |   /
65|  / <-light
cm| /    beam
  |/
  + <-light
      source

tan(48.75) = 65cm/x

1.14 = 65cm/x

x = 57cm

Did I make a mistake ? StuRat 15:00, 18 October 2006 (UTC)[reply]

Just a trivial one: the angles in Snell's law, etc., are measured from the normal to the surface, not the surface itself. So the angle in your triangle should be the complementary angle: 41.25°. Hope this helps. --Tardis 17:34, 18 October 2006 (UTC)[reply]
Thanks, that clears it up. StuRat 20:52, 18 October 2006 (UTC)[reply]

October 17

Variables

I don't understand constants, the control, independent and dependent varibles. Or what role do they play in a science fair project? Please explain. Thank You. Molly Smith

Hi Molly. The dependent variable is the variable that the experimenter cannot control, the indepedent one is the one you can control. So if my experiment is meant to answer a question like, "what color of paper attracts more bees to it?", I can change the color of the paper directly, but I can't change the bees directly. Thus my experiment would consist of me changing the independent variable (the paper) in order to observe the change in the dependent variable (the bees).
Constants in this context probably refer to the fact that there are some variables you don't want to change. For example, in my paper/bee experiment, I would want to make sure I was doing all of these experiments at around the same time of the day. Otherwise any effect I saw could potentially be due mostly to the changing time of the day it was — perhaps bees are more active at one time of the day than another, and so I wouldn't really be measuring the dependent variable purely if some complicated factor like that was also at play.
Controls are not always possible — it depends on the type of experiment it is. In my color/bee experiment there could be no control. Controls are only possible in experiments where you have a very definite causal hypothesis; for example, if I were a doctor and I wanted to a study of a new medicine, I would want to make sure there were some patients I was monitoring who were not taking the medicine, or were taking a placebo (which looks like medicine but has no biological effect). That way I would be able to tell the difference between the medicine I was testing and no medicine at all; the people who were not taking medicine would be the "control" to my study. Again, not all experiments can have, or really need, a control — it depends on the type of question being asked.
You might find the Scientific method article at Simple a good resource; it is much more readable and less pedantic than the scientific method article on Wikipedia. Hope that is helpful. --Fastfission 01:42, 17 October 2006 (UTC)[reply]
This is just to clarify an already INCREDIBLE explanation.
  • The constants are the things kept the same across all of the experiments. The only things that you should change from one experiment to the next are the Independent Variables. Example: If you were comparing the length of time it takes to bake cakes using different types of flower, you would want to make sure that the other ingredients used were of the same brand for all of the experiments. You would also want to make sure the amounts were the same, you used the same type and brand of cake pan, and they should all be cooked in the same oven at the same temperature. The purpose of keeping these things constant is so the effect of the Independent Variable can be isolated.
  • The control is the group in your experiment that has no Independent Variables. It is what all of the experimental groups are compared to. My cake experiment would also lack a control. If you were comparing the effect of different types of soda on tooth decay, the control would be a tooth kept out of soda. The purpose of a control group is so that you can show that the Independent Variable was (not) causing the change. The control group shows what would have happened naturally, without any outside interference.
  • The Independent Variable is what you change in the experiment. In my cake example, the Independent Variables would be the different types of flour, and in my soda example the IV would have been the type/brand of soda. The independent variables are what make an experiment an experiment. The IV should be the only thing to change from group to group.
  • The Dependant Variable is the reaction to the IV. In my cake example the DV would be the amount of time it took to bake the cake, and in my soda example the DV would be the rate of tooth decay. This is what you are doing the experiment to find. If you are comparing the results of multiple experiments, you would compare there dependant variables; depending on the type of experiment you may compare the IVs as well, but you will always compare the DVs. If you are doing just one experiment, such as how long does it take to bake a cake with X brand flour, or how long does it take to dissolve a tooth using Coca-Cola, this would be your answer.

I hope that we have been able to help you, and good luck in the science fair. THL 05:02, 17 October 2006 (UTC)[reply]

Combustion of BIO Gas.....

I'm working on an anaerobic digester design and i found that the CH4:CO2 ratio is between 1:1 and 3:2. In addition it may contain Hydrogen Sulphide, Nitrogen, Amonia and Hydrogen in small amounts.

I'm planning to utilize this gas to produce CO2 to be used in another industry. So i would like to know further information about following issues.

1. will it be effective to combust methane+CO2+other trace gasses mixture to produce CO2

 (will the excess CO2 affect the complete combustion?)

2. What will be the products after combustion of the above gas mixture?

3. what will happen to NH3 and H2S after combustion?

It is planned to combust the mixture in lower temperature (Less than 200 C)

Appreciate any suggestion to success my effort and any links regarding this process will be really helpful to me...!! Thank you.

--192.248.8.100 04:27, 17 October 2006 (UTC)Sithara from Sri Lanka[reply]

A few questions about the human brain

  • How much of the energy used by the body is actually consumed by the brain? Hoping for an answer in kcal/day or something like that. A similar question is: how much of the body's oxygen does the brain use? (I think the answer to this is around 25%, but I can't find any sources that give a figure, let alone one that supports mine)
  • Someone told me recently that the brain cannot burn fat or protein to supply itself with energy. I think this is probably not true, but I can't find a source either way. Does anyone know?

Thanks in advance for any answers!

N Shar 04:47, 17 October 2006 (UTC)[reply]

the brain uses ~1760 kJ/day, pretty much entirely from glucose. the brain will burn glucose as its fuel of choice, if little glucose is availible from the diet, gluconeogenesis in the liver is used to synthesise the required glucose from other materials. in prolonged starvation the brain can use other fuel sources such as ketone bodies, but not very well. further info[30]. Xcomradex 05:28, 17 October 2006 (UTC)[reply]
Yay! Thank you! N Shar 05:30, 17 October 2006 (UTC)[reply]
That's 20 W, in case someone wanted to know. —Bromskloss 11:31, 17 October 2006 (UTC)[reply]
and apparently the brain uses ~20% of the oxygen for your body, but that is from a snake oil pusher[31]. Xcomradex 05:38, 17 October 2006 (UTC)[reply]

I would expect the ratio would vary dramatically, based on whether you are sedentary but engage in intellectually challenging pursuits, or meditate while exercising strenuously. StuRat 14:53, 17 October 2006 (UTC)[reply]

Right. The fact that the brain needs more energy when thinking is a most valuable thing for researchers studying the brain with positron emission tomography: The patient get radioactively marked glucose, and the PET scanner then shows where this glucose is used. And, BTW, as nobody has commented on it so far: If I am not mistaken the percentage of energy and the precentage of oxygen that the brain uses as compared to the total organism should be the same. After all, a cell always needs the same amount of oxygen in order to use a given amount of energy stored in food. But I'm not sure. The answer should be hidden somewhere in the citric acid cycle. Simon A. 16:05, 17 October 2006 (UTC)[reply]
  • As for the second question: It is true. The brain can't burn fat or protein. It gets all its nourishment from the glucose in your bloodstream. This is one of the main reasons you have a liver - to store glycogen and maintain your blood-sugar level. It's also why it gets tough to think straight when your blood sugar is low. Once your liver glycogen is depleted, the brain starts consuming ketone bodies (acetone and similar) from your blood, which your body produces from catabolizing the proteins of its own tissue. This is why your breath starts to smell of acetone when you start starving or if you run for about 30 km or so - prolonged exersize follows the same process as starvation, but at a faster rate. The body needs to produce these ketone bodies because nerve tissue doesn't have the ability to break down protein itself. The amount of energy used by the brain or any other organ isn't constant of course, in particular once you start starving. Then the body starts shutting parts down to conserve energy. --BluePlatypus 23:02, 17 October 2006 (UTC)[reply]

Snake Bite

What's the worst snake to be bitten by ?

The Fierce Snake is the worst land snake, but there is at least one sea snake which is more venomous. Not many people get bitten though. --liquidGhoul 05:11, 17 October 2006 (UTC)[reply]
What about the overall death experience? A slow painfull death would be much worse than a quick death from a deadlier snake. Gaboon vipers have two inch fangs and actually eat small antalope!
Then I would say getting eaten by a python would be the worst. It's a matter of opinion. --liquidGhoul 05:37, 17 October 2006 (UTC)[reply]
Is there some sort of pain index pertaining to snakes, as there is for insects with the Schmidt Sting Pain Index?  freshofftheufoΓΛĿЌ  06:08, 17 October 2006 (UTC)[reply]
That involved Schmidt subjecting himself to insect stings. Noone is stupid enough to do that for snakes. --liquidGhoul 06:18, 17 October 2006 (UTC)[reply]
I don't see how it couldn't be done from case studies, at least as an "approximation".  freshofftheufoΓΛĿЌ  05:34, 19 October 2006 (UTC)[reply]

I have heard that many deaths due to snake bites are mainly due to fear and panic rather than the poison itself !. -- Wikicheng 06:54, 17 October 2006 (UTC)[reply]

You don't die from fear, maybe panic if you run off a cliff or something, but no. Most deaths from snake bite are from the venom. A large proportion of snake bites are after they have been picked up by someone. It is amazing how stupid some people can be. --liquidGhoul 07:36, 17 October 2006 (UTC)[reply]
Well, snake oil is just so tasty. — X [Mac Davis] (SUPERDESK|Help me improve)08:38, 17 October 2006 (UTC)[reply]
I'd never even heard of that term before, and now it is used in two threads in a row! --liquidGhoul 11:17, 17 October 2006 (UTC)[reply]

Reversing a Black Hole...

This question might be a bit long-winded, but, here goes. Hyperthetically, imagine a very young, 'borderline' Black Hole: A supernova 'just' went-off and produced said black hole. By 'borderline', I mean that had the original "sun" been JUST A BIT less massive, it would have produced a neutron star instead. Now, would it be possible to FORCE-FEED this black hole with enough matter to change it into a neutron star? For that to happen, would the matter have to be "fed" at faster than the speed of light? (of cousre impossible, hence, hyperthetical). Once a black hole, ALWAYS a black hole? (ignoring evaporation). Just something to 'chew' on. Thanks, Dave152.163.100.74 06:55, 17 October 2006 (UTC)[reply]

Addendum...What I really meant to ask was, is a SINGULARITY reversible? 152.163.100.74 07:12, 17 October 2006 (UTC)[reply]
No one really knows except Stephen Hawking. And he's not telling 8-)--Light current 07:27, 17 October 2006 (UTC)[reply]
You have it confused—black holes have greater mass than neutron stars. You could feed a neutron star and squeeze it until it became a black hole. — X [Mac Davis] (SUPERDESK|Help me improve)08:42, 17 October 2006 (UTC)[reply]
However, a small singularity can "evaporate" due to Hawking radiation. StuRat 14:42, 17 October 2006 (UTC)[reply]

No I don`t Mac Davis. Perhaps I should have posed my question thusly: Can a black hole, or even a neutron star, for that matter, be 'force-fed' to return to main-sequence 'status'? Touche about Stephen, Light! 8)Dave 152.163.100.74 14:29, 17 October 2006 (UTC)[reply]

Well, there's this: if a black hole is rotating fast enough, and you skim objects across it at just the right trajectories, you can extract as much as ~29% of its mass/energy through the Penrose process before its rotation slows to zero. But this process doesn't decrease the area of the event horizon, so it can't destroy the singularity. Melchoir 15:11, 17 October 2006 (UTC)[reply]
So yes, once a black hole, always a black hole (ignoring Hawking radiation). Removing mass is not possible even if you can travel faster than light. --Bowlhover 00:28, 18 October 2006 (UTC)[reply]

Amount of Water in the World

Is there a set amount of water in the world? I understand the process of condensation and all that but I wonder, if it's always a circular movement, then is there a certain amount of water thats continually moving around or is water created and destroyed in some way, if so, how?

Thank You

Should stay about the same, until the next comet strike that is. 152.163.100.74 07:06, 17 October 2006 (UTC)[reply]
Water is not created or destroyed. It falls as rain, drains into rivers and seas, is evaporated into the atmosphere and falls as rain again. Quite clever really! It should be called the water cycle. Dont know if it is. 8-)--Light current 07:25, 17 October 2006 (UTC)[reply]
Not strictly true, you forgot chemical rxns. eg in hydrogen cars 2H2 + O2 => 2H2O  ; or burning of carbohydrates and hydrocarbons nCH2O + O2 => CO2 + H2O , to pick 2 important cases. (H2O is a greenhouse gas) Water is destroyed (split) by plants in photosynthesis, added to CO2, to make those carbohydrates. Next question is whether number of C H and O atoms is constant on Earth. Care to guess? ==GangofOne 07:48, 17 October 2006 (UTC)[reply]
I think what you are talking about is more strictly to do with the carbon cycle. Im ignoring H2 cars etc.--Light current 08:09, 17 October 2006 (UTC)[reply]
There's a fair bit of water in space, eg in comets and other flying chunks of ice. There is even a theory that all the water on Earth came from those colliding with Earth. Of course there is much less of them left now, but they will still add to the amount of water on Earth, possibly negligible, but still. Conversely, won't some water evaporate into space? I remember wondering about that when I was a kid, and I never got an answer. DirkvdM 10:51, 17 October 2006 (UTC)[reply]
Water vapor, at a relatively hefty 18 amu, has a horrid time escaping Earth's gravity. Using the Maxwell distribution, a conservative value for Earth's escape velocity in the upper atmosphere, and the Integrator (as a high-precision error function evaluator), I get that at 300 K, about of water molecules (as in, one in every ) will have escape speed at any given time. (Of course, many of those will be moving downward! But this is just a factor of 2 or 3.) Supposing the entire mass of the Earth to be water vapor making these attempts (that's molecules), that they each get to try once per mean collision time (which I approximate as their mean free path divided by the expected particle velocty ), and that all attempts with the appropriate speed are magically directed "out" and encounter no collisions during the escape, I get an attempt frequency of . Of course, there's still an expected success interval of because of the ludricously low probability of escaping per try. (At a not-unreasonable upper-atmosphere temperature of 1000 K, the much better odds of having the escape energy () and somewhat shorter mean collision time predict 480 trillion escapes per second; however, this is still only 14 nanograms of water per second, and that's still with the whole Earth of water vapor with magic escaping skills.) In short, no. The water isn't going anywhere. --Tardis 05:51, 18 October 2006 (UTC)[reply]

A huge amount of water gets entrained in subduction zones. There is actually a geologic water (and carbon) cycle, which can be tracked in geologic time. The amount of water in the biosphere varies over millions of years. --Zeizmic 11:58, 17 October 2006 (UTC)[reply]

So, to answer the question, yes, there is more or less a constant amount of water on Earth. While it can be created or broken back down to hydrogen and oxygen, these processes are more or less in equilibrium, so there is no net increase or decrease over time. StuRat 14:21, 17 October 2006 (UTC)[reply]

To DirkdvM...No, I don`t think so. Most of the humidity, water, is contained in the troposphere, and to a much lesser extent, in the stratosphere. That`s where all the 'weather' is. THAT part of the atmosphere which would/could evaporate would be 'dry'. Hope this answers your long-ago childhood question! : ) Dave 152.163.100.74 15:49, 17 October 2006 (UTC)[reply]
To add...if you meant wouldn`t some water escape/evaporate into space during/following a cometary COLLISION, then, I`d have to say, "Of course!". But, I think there would have to be an over-all GAIN, assuming the comet would have brought megatons of water with it! Dave 152.163.100.74 16:09, 17 October 2006 (UTC)[reply]
To StuRat: "these processes are more or less in equilibrium,"; That's only your unsupported assumption. Surely burning down half of the Amazon basin is a noticable change. --GangofOne 21:11, 17 October 2006 (UTC)[reply]
Not relative to the total amount of water on Earth, it isn't. StuRat 05:42, 18 October 2006 (UTC)[reply]

Burning (or almost any other process in the troposphere) will have a negligible effect. There are however geological effects that sequeater water by reacting it with magmea to from other kinds of rock. In the very long term, this water will be recycled into he troposphere. Thare is a larger effect in the upper atmosphere. Sunlight "cracks" water into hydrogen and oxygen,and the hydrogen escapes into space relatively more easily than does oxygen. (sorry, no references: this is from memory of magazine articles from long ago.) - 70.177.166.201 00:20, 18 October 2006 (UTC)[reply]

Human evolution and immunity to venoms

It is oftern said that the most poisonous animals live in Australia. Our articles on venoms, i.e., animal poisons, seem to agree with that and also list quite some dangerous animals in South America. Here, in Europe, is is nearly impossible to meet animals with deadly venom, and also African and Asian animals seem to have a rather hard time killing a human. This looks to me as if the human body is well adapted to deal with the kind of poisonous animals to which the species Homo sapiens was exposed during the longest time of its evolution, which happened, as we know, mainly in Northern Africa and then spread out from there, South America being populated rather late and Australia very late. So, do you think that we humans have evolved the immunitary capability to deal with Old World venoms and hence, New World venoms are more dangerous? A counter argument would be if other large mammals were also more easily killed by, say, Australian snake species than be European ones, even if they are marsupials that evolved in Australia. Does anybody happen to know about this stuff? Simon A. 11:51, 17 October 2006 (UTC)[reply]

I would argue that, over time, people killed off most dangerous animals in populated areas. For example, rattlesnakes are now being killed off in the south-west US (or at least those that can be found, which are the ones with loud rattles). People have a rather low tolerance for the existence of animals which can kill them. StuRat 14:13, 17 October 2006 (UTC)[reply]
For that to hold true, humans must have wiped them out completely, which is rather unlikely. Humans may have eradicated quite a few se=pecies, but not the little critters that tend to be poisonous.
Looking at the issue the other way around, why would spiders develop a venom that kills humans? They can't eat us. So it must be an unfortunate (for us) side-effect. So if that happened so often elsewhere, then why indeed did it fortunatley not happen in the places where humans evolved. Indeed, it sounds like too big a coincidence and I can't think of any other explanation. DirkvdM 08:48, 18 October 2006 (UTC)[reply]
People definitely can kill off something like snakes, as is happening now in the rattlesnake example I gave. StuRat 14:28, 18 October 2006 (UTC)[reply]
As is happening now. So hasn't happened yet. After thousands of years of cohabitation. QED. DirkvdM 18:24, 18 October 2006 (UTC)[reply]
For most of human existence, what is now the US southwest was sparsely populated, because it's largely desert. Only quite recently has it started to become populated to the point where people would pose a threat to native species, due largely to the Hoover Dam, which provides sufficient water and power for people to live there. I will be sure to update you once the extinction is complete. StuRat 20:40, 18 October 2006 (UTC)[reply]
Ok, point taken. But I was also alluding to the fact that we now have much better capabilities to track and kill animals. DirkvdM 07:56, 19 October 2006 (UTC)[reply]

End correction

Can anyone please give me information regarding end correction in resonating air columns ? I would like to know what causes it and how it can be calculated . Thanking You.

Tried resonance in pipes? Heres a nice link [32]--Light current 13:38, 17 October 2006 (UTC)[reply]

What frequency of microwaves can create an overpressure shockwave?

Microwaves, as i have heard can impart heat and momentum to air molecules, much like what happens in a shockwave during an explosion. The air is propelled into objects and as it is traveling it is heated up and expands causing even more pressure against what it hits. Does anyone know what frequency microwaves would be needed to create the same effect without useing chemical explosives? Is such a thing possible?

Robin

Never heard of it, and the physics doesn't seem right. Microwaves could cause a water pocket to steam burst (mini boiler explosion), and this could cause a standard shock wave. --Zeizmic 16:13, 17 October 2006 (UTC)[reply]

Is there any kind of EM wave which could impart enough momentum to air to cause a shockwave, or heat air up enough, causing it to expand quickly causeing a shockwave? Robin

Microwaves can't heat air to any significant extent. To absorb microwaves and heat up, a molecule needs to have a dipole moment. Almost none of the components of air have a dipole moment, the exception being water. Water has a dipole moment, which is why the water-rich food in your microwave gets hot, but the plate and air stay relatively cool. --BluePlatypus 22:47, 17 October 2006 (UTC)[reply]
A photon obeys the relationship , where E is energy, p is momentum, and c is the speed of light. It's hard to compare different dimensions like energy and momentum, but c can generally be trusted to be "large" even though it can of course be written as , where d is "1000 times the speed of light". The point is that photons have much "more" energy than momentum, and so unless the energy is irrelevant somehow (as in a solar sail, where it is reflected) the energy effects will dominate. In this case, it simply means that you will turn the air into a violent, turbulent plasma that explodes simply due to overpressure long before you will do anything special pushing it around: the same light (of any variety) that heats a parcel of air by 1 Kelvin will only propel it at 105 meters per year. (Note that the same light can't do both; light in random directions can heat without propelling at all, but no light can propel without heating at all, due to relatively advanced conservation of momentum considerations.) And if you're content with heating the air, why bother with microwaves? TNT usually does the trick, or just a shorted-out capacitor if you want to stick to electricity. --Tardis 05:10, 18 October 2006 (UTC)[reply]

adding a word to search list

How do I add a link from an existing wikipedia article to a search list?

Specifically, the word 'Teazle' should be linked to the article Teazel (http://en.wikipedia.org/wiki/Teazel) and then appear with high relevence in a search-list for the word 'teazle'-- teazle is the more common spelling in the UK

John Barrs 17:14, 17 October 2006 (UTC)82.19.174.20

You create a redirect, like this: Teazle. StuRat 18:08, 17 October 2006 (UTC)[reply]
This is what it looks like:

#Redirect [[teazel]]

StuRat 18:08, 17 October 2006 (UTC)[reply]
I see that gave us a double redirect, so I changed it to redirect only once:

#Redirect [[dipsacus]]

StuRat 18:12, 17 October 2006 (UTC)[reply]

Jogging and calorie consumption

I ran for 20 minutes today at a slow pace. I guessed it to be about 4 miles per hour, but it could have been a bit faster. I weigh about 12 stone (168 pounds or 76 kilograms). Some questions:


a) How fast is a slow jog considered to be?
b) How many calories did I burn?
c) What is considered normal walking speed?


If I knew c) then I could work out how fast I run and then measure how far I've gone and my calorie consumption. Cheers!

The rate of forward motion shouldn't matter much. Even jogging in place should burn up just as many calories. Note, however, that jogging uphill will make you do significantly more work than jogging in place or on level ground. StuRat 18:25, 17 October 2006 (UTC)[reply]
Surely running at 7 mph burns more calories than at 4??
Not if your arms and legs are moving about just as much, as in "power walking". StuRat 18:59, 17 October 2006 (UTC)[reply]
It takes approximately the same calories to travel a mile at 1mph and 2mph, because it takes half as long. Calories per distnace run are approximately constant (note: approximately). RJFJR 20:05, 17 October 2006 (UTC)[reply]

"Jog" means to "run slow" in the first place. 4mph would qualify as jogging rather than running, and since it is quite easy to walk 4mph, you could classify it as a slow jog. People walk at different speeds, but 2 to 4 mph would be normal. -THB 22:22, 17 October 2006 (UTC)[reply]

Try this calorie calculators:[this] and [this]. --Auximines 22:40, 17 October 2006 (UTC)[reply]
Basically, if you make sure you have both of your feet off the ground at one point you're fine. Not doing so automatically classifies you as a walker, or a power walker.  freshofftheufoΓΛĿЌ  05:31, 19 October 2006 (UTC)[reply]

solubility of capsaicin in edible oil

I shall be extremely thankful if you can provide me the following information:-

  1. How much capsaicin is released from chili in edible oil in terms of percentage of the total quantity present when powder of chili is mixed in oil?
  2. What is the optimum time and temperature to get the maximum quantity of capsaicin released into the oil?
  3. Does green and dry red form of chili make a difference for the release of optimum quantity of capsaicin into the oil with reference to 2 above?
  4. Is there is a difference between the solubility of capsaicin and capsacinoids in oil?

Regards,

59.95.99.138 17:32, 17 October 2006 (UTC)S k sethi[reply]

What kind of oil? Do your textbook have information on the solubility of capsaicin in oil? Surely your professor doesn't expect you just to know these things and the underlying info must be in your textbook. Get started on the problem and come back here for assistance. -THB 22:15, 17 October 2006 (UTC)[reply]
This doesn't sound like a homework problem... Taking a look at Capsaicin#Capsaicinoids, I can see that they are all very similar. They have an identical ortho disubstituted benzene group and a hydrocarbon tail that varies only in length. The length of the tail will tend to increase its tendency towards hydrophobicity, making it more soluble in the standard oils used in food preparation. All of the tails differ by a carbon or two, so the degree of difference in hydrophobicity wouldn't be great. As for an optimum solubility of capsacinoids, the species of pepper you use shouldn't make much difference beyond the amount of capsacinoids they contain, so you could whichever pepper have the most in it; I would think that dry would be better, if only because it has more surface area when it's crushed, allowing for more efficient migration of the solute into the oil. Sorry I don't have any hard numbers. – ClockworkSoul 01:50, 18 October 2006 (UTC)[reply]

Feta

This question my be a bit dumb, but just why doesn't feta melt when hot like every other decent cheese? --Janneman 19:20, 17 October 2006 (UTC)[reply]

  • it does get softer. my guess is it just has a higher melting point. try torching it!
  • Hehe, that's agressive cooking! :-) —Bromskloss 22:23, 17 October 2006 (UTC)[reply]
But not uncommon: Flambé. DirkvdM 08:55, 18 October 2006 (UTC)[reply]
  • It's because it hs a low fat content. Englishnerd 21:06, 17 October 2006 (UTC)[reply]

Suicide pill

Im looking for an easy and peaceful way out. i dont have the guts to throw myself off a bridge or anything like that. i was thinking about overdosing on over-the-counter drugs but im afraid that would cause slow and painful death or just make me realy sick. where can i obtain a suicide pill (doesn't have to be specifically made for that). i need something that would do it painlessly and easy to obtain (in Canada) or buy online.

p.s. please dont point me to suicide prevention bullshit cause those people are just stupid. my problem wont ever go away and im going to end this. its just a matter of how.. i dont want to have to run in front of a bus or something.

What physical illness do you have? —Centrxtalk • 20:10, 17 October 2006 (UTC)[reply]
It certainly is sad to hear that you want to kill yourself. Why do you feel that suicide is necessary? None of us here, and perhaps nobody where you are in Canada knows what you are going through, and certainly you are in a rough place if you want to kill yourself, but this is a decision that you will not be able to go back on. If you have a physical illness, have you had a second opinion? -a fifth opinion? Maybe someone here can tell you about research or give you hope for the future. Did you have a falling-out with a loved one? That is something that only time can heal, but YOU WILL FEEL BETTER as time passes.
You asked that nobody point you to suicide prevention bullshit, and I will honor that, but have you considered seeing a psychologist of psychiatrist? You wouldn't even have to tell them that you were suicidal, and sometimes just talking helps.
Nobody here is going to tell you how to kill yourself. Even in this tiny community which represents a tiny portion of a corner of the internet, you would be missed. Your addition to this help desk, in the Feta cheese article above, helped us out, even if it was just a little bit. Please take care of yourself, do whatever you need to do to get yourself feeling better, and find help. And if you need help finding help, just ask; ask us, ask a doctor, ask someone. I think I speak for all of us when I say I hope we hear from you again.Tuckerekcut 21:07, 17 October 2006 (UTC)[reply]
I've had some really tough bad things and loss happening in my life in the past, and I've felt suicidal, but now I'm reasonably happy. Try studying the stoic philosophy. Get more detached from life, don't take it so personally. Try helping others.
Antyhing that makes you want to kill yourself isnt beneficial to you, so whatever your problem is, its hurting you to worry about it that much. So yeh I'd recomend speaking to someone with wisdom in this area about your problems. Philc TECI 22:09, 17 October 2006 (UTC)[reply]

Unfortunately, an overdose can screw up and leave you brain-damaged or in a coma. It would be interesting to know what is so irrevocable and permanent and it would help answer your questions. -THB 22:12, 17 October 2006 (UTC)[reply]

The classical resource is Compassion & Choices, formerly known as The Hemlock Society, but I think you'll find they're more focused towards people who are near the end of their lives due to medical problems. Like everyone else here, if that situation doesn't apply to you, I'd encourage you to seek other help than theirs.
Atlant 22:51, 17 October 2006 (UTC)[reply]
Are you *absolutely* sure that you want to die? A long time ago, I thought I did too. I got drunk and sat on my bed with my revolver, playing Russian roulette. Five clicks and nothing. When it came to pull the trigger for the sixth time and face a 1/1 chance of a bullet to the head, I just couldn't do it. My finger literally wouldn't pull the trigger, no matter how hard I tried. It was only then that I realized that there was a part of me inside that really wanted to live. --WineBob 23:07, 17 October 2006 (UTC)[reply]

There is another way to leave your problems behind, just get up and leave. Go somewhere where nobody knows you, your past, or your problems, and start life over. Of course, this doesn't apply if your problem is a terminal disease, but applies to just about anything else. You might say you would feel guilty about leaving everyone you know, but you would be doing that anyway if you killed yourself. You can either tell people you are leaving or just skip out of town, your choice. StuRat 00:06, 18 October 2006 (UTC)[reply]

The loss of you would greatly diminish each of us. Everyone I know has had times when it didn't seem like life was worth living: your loved one is gone, or the job is lost, the stress is too great, or the health is impaired. But we have found that as time went by, other things came along which made life worth living. Don't give up. Medication can help you get out of the biochemical pit of depression. ""All mankind is of one author, and is one volume; when one man dies, one chapter is not torn out of the book, but translated into a better language; and every chapter must be so translated...As therefore the bell that rings to a sermon, calls not upon the preacher only, but upon the congregation to come: so this bell calls us all: but how much more me, who am brought so near the door by this sickness....No man is an island, entire of itself...any man's death diminishes me, because I am involved in mankind; and therefore never send to know for whom the bell tolls; it tolls for thee." John Donne. Edison 05:39, 18 October 2006 (UTC)[reply]

Not quite a faq, but it's been asked before, when I recommended a heroin overdose. But I also suggested something like what StuRat said. Moving elsewhere might take away the reasons you now have to kill yourself (depending on what they are). But to expand on that, if you've already made up your mind, then basically you're already dead. That opens up a whole new range of possibilities. You can now do whatever you like. Who cares? You're dead already, aren't you? So sit down and think what you would really like to do. No inhibitions, just go for it. With some luck you'll like it. DirkvdM 09:21, 18 October 2006 (UTC)[reply]
I personally knew someone from high school who tried to kill himself by jumping on a railroad. I believe that the main thing that drove him to do it was stress at university. He didn't manage to kill himself, but did lose an arm. He since made a recovery from being suicidal, and is now making something of his life, pursuing a career as an artist. When I got to meet him at a high school reunion a few years ago he seemed to have a very positive outlook, though obviously he'd be better off now if he still had both his arms. Now obviously I don't know what you're going through, but still I think there's a lesson from his experience that may be applicable more widely, and maybe to your situation: that however much suicide may currently seem to you like the only option, suicidal feelings can be a passing phase, and the less damage you suffer in the meantime the better it will be for you in years to come. All the best to you — you're worth it. Arbitrary username 12:47, 18 October 2006 (UTC)[reply]
Actually, the lost arm may have been the shock that made him change his life. Not that I'd advise the questioneer to cut off his arm, though. :) DirkvdM 18:29, 18 October 2006 (UTC)[reply]
P.S. I see you posted a separate message about how to use your phone as an MP3 player. (At least I assume it's your message; it was posted from the same computer just a few minutes earlier.) I'm really sorry I don't have the answer to that question. But obviously you enjoy listening to music. That's great. As you're feeling down at the moment, I reckon now's a good time to treat yourself. Go out and buy yourself a recording from one of your favorite groups / composers / whatever. Arbitrary username 13:08, 18 October 2006 (UTC)[reply]
Are you really so intent on ruining the lives of your friends and family? Do you really want to be the dagger in the hearts of those who put you in this world and brought you up? Is that not quite selfish? I beg you to read the Gospel (start with that of Matthew) and realise that there is much joy to be had if you so wish. God bless you, 82.65.102.5 12:57, 18 October 2006 (UTC)[reply]
Blimey! If he weren't suicidal already, he'd be now. After all this crap I wonder if he's still reading this. DirkvdM 18:29, 18 October 2006 (UTC)[reply]
Ah, religion. The thing that gives you false hope by bringing you into a denial. Also: "If you don't do this and you don't do this, you'll be tortured forever in a place called hell." Now I really want to die. --Bowlhover 19:34, 18 October 2006 (UTC)[reply]
Yeah all this feely emotional crap is a little goober but "getting religion" is pretty much the stereotypical turnaround for suicidals so don't discount its effect --frothT C 19:10, 18 October 2006 (UTC)[reply]
He's right; religion might not be the answer for most of us (especially here on RD), but there are loads of people that manage to find their happiness in that little book, and who's to judge them for that? I for one don't see the problem with at least one plea to "find Jesus, Mohammad, or Buddha" among all of the other comments. Heck, he's probably a lot more likely to be picked up by some psycho cult with his mindset, so the least we can do is point him towards someone (a priest, rabbi, monk, or imam) who might make an honest effort to help him.  freshofftheufoΓΛĿЌ  05:26, 19 October 2006 (UTC)[reply]
He specifically stated he didn't want any of that sort of help. Also, religion might just as well turn him off. The fact that he comes to the Wikipedia ref desk says something about him. And the science ref desk no less. Like you said, people here aren't generally into religion, and that may be precisely what he was looking for. Also, Bowlhover is amongst us probably the greatest expert on this subject. DirkvdM 08:09, 19 October 2006 (UTC)[reply]

Purpose of Viruses

While some bacteria are harmful, most bacteria serve a beneficial purpose. Higher lifeforms would not exist on earth without the presence of helper bacteria. Viruses on the other hand seem to be, at best, innocuous and, at worst, highly destructive. What biological purpose do viruses serve (other than killing off the weak)? Are there any "good" viruses?

"Purpose" is all in your mind, or at best an observed side-effect of "living things reproduce" and "living things adapt to utilize the resources and conditions that are available". DMacks 20:24, 17 October 2006 (UTC)[reply]
Thinning the herd? Ƶ§œš¹ [aɪm ˈfɻɛ̃ⁿdˡi] 20:28, 17 October 2006 (UTC)[reply]
No 'life' if you include viruses in that serve any purpose, they are just chains of chemicals created through coincedence that in the enviroment they were in somehow self replicated, variations through mistakes in this process led to variation, ones that couldnt replicate died, those that could survived. Thats life and evolution for you, on an incredibly basic level though. Philc TECI 20:42, 17 October 2006 (UTC)[reply]
Then why have bacteria evolved synergistic relationshipship with other life forms but viruses have not?
Because of the way we define them, virusses by definiton of the term, have no means of self reproduction, and so, in all cases I know of, hijack and destroy a host cell instead, it is very dificult to have a relationship with a parasite, when all this thing is capable of is destroying your cells. Keeping in mind, virusses aren't complex enough to perform any other functions than replication. Whereas bacteria are complex enough organism to perform some (sometimes useful) function in their life span, and reproduce without killing host cells, this makes them far more useful. Philc TECI 22:06, 17 October 2006 (UTC)[reply]
We have been using Bacteriophages for years, but have proven less effective than antiobiotics, but such things could be deemed as synergistic, in a sort of common enemy sort of way. Philc TECI 22:37, 17 October 2006 (UTC)[reply]
It doesn't make a lot of sense to ask "why" a given evolutionary result has not been seen. Maybe there's no good route from "where life has been thus far" to where you want to go; but maybe in the future it might. Maybe we just isn't imaginative enough to figure out an evolutionary niche for something...Nature and random-chance/trial&error are sometimes at least as smart as scientists:) What if one virus protected us from some other worse virus?[33] DMacks 22:15, 17 October 2006 (UTC)[reply]

Viruses move genes around and thus provide a source of diversity to then be acted upon by selection; ie they speed up evolution. However the words 'good' or 'bad' do not apply. --GangofOne 20:59, 17 October 2006 (UTC)[reply]

Some viruses are desirable: Good Viruses -THB 22:09, 17 October 2006 (UTC)[reply]

Viruses have also given us wonderful ways of studying the structure of proteins and DNA. So maybe that's their higher purpose. ;-) --Fastfission 22:15, 17 October 2006 (UTC)[reply]

In the future, we may be able to change our genetic codes using a virus to deliver the new DNA. This is obviously quite dangerous, but could be enormously beneficial, as well. StuRat 23:54, 17 October 2006 (UTC)[reply]

As to why viruses have never developed symbiotic relationships with host cells... some have speculated that the evolutionary origin of the intracellular organelles of eucaryotes is as a symbiotic viral infection...--Mark Bornfeld DDS 00:01, 18 October 2006 (UTC)[reply]

"they speed up evolution. However the words 'good' or 'bad' do not apply" <-- The term "good" is used here in the context of what is beneficial for virus-infected organisms. Theoretical biologists such as Stuart Kauffman have discussed the importance of evolvability as a trait. In addition to playing an important role in the rate of evolution, some hypotheses suggest that viruses played a fundamental role in the origin of cellular life as we know it; see: Three RNA cells for ribosomal lineages and three DNA viruses to replicate their genomes: A hypothesis for the origin of cellular domain by Patrick Forterre. --JWSchmidt 01:51, 18 October 2006 (UTC)[reply]

"Why does X exist?" is a teleological question. It exists. Edison 05:42, 18 October 2006 (UTC)[reply]

What does cannabis smoke smell like?

I'm wondering if someone I know smokes it. The smoke is somewhere between the smell of tobaco smoke and incense sticks. I'm not going to turn her over to the cops, just curious.

She could also be smoking Kreteks or another type of scented cigarette. The fragrance of cannabis depends greatly on what type and form is being smoked, but the odor is usually more pungent than that of incense. It could also be a combination of (cigarette) smoke and the air freshener she's using to mask the smell.---Sluzzelin 22:00, 17 October 2006 (UTC)[reply]
It has a unique smell, very hard to describe, and it can vary, as above. I also agree that it sounds more like a clove cigarette or even a bidi.
Have you considered asking her if it is marijuana smoke? You'll be able to tell whether she tells the truth or not. -THB 22:05, 17 October 2006 (UTC)[reply]
She may just be burning inscence sticks to remove the nasty smell of the tobacco smoke. Cannabis has a very distinct pungent smell, it would be difficult to think that anyone would be deliberately making that smell, so I doubt any inscence sticks smell like it, however, they are regulalrly used to mask its use, as with normal smoking. Philc TECI 22:30, 17 October 2006 (UTC)[reply]

I think she would be very unlikely to smoke any kind of scented cigarette, clove cigarette, or bidi. I don't want to ask her as she would think I was being hostile. Does cannabis smell unpleasant, or pleasant? What exactly is meant by it smelling "pungent" for example?

It is an extremely dense, sickly-sweet smell, not a thousand miles from the smell of sweat in many cases. If you think you smell weed, chances are you do.

It has been described as the smell of burning leaves. Expensive pot, like the smell of expensive burning leaves.Edison 05:43, 18 October 2006 (UTC)[reply]

Or rather burning flowers, because that's what it is. Marijuana, that is, which indeed smells rather sweet (in the literal sense). But hasjisj has a much sharper smell. Are there no 'coffee shops' or such where you live, where you could go and have a sniff? DirkvdM 09:42, 18 October 2006 (UTC)[reply]

EM wind

I heard something on this before, but am still not sure so shall just ask directly and to the point, Em waves can cause momentum transfer with air molecules right? so that means that it is , at least in theory , possible to create EM wind, that is, air currents caused by interactions with Em waves. Is this correct?

I'm not an expert, but I don't see why not. --Allen 23:14, 17 October 2006 (UTC)[reply]
Yes; if you shine a strong laser through a sealed glass box, you will get a small current flowing along the laser beam (then around the sides of the box back to its entry point) because of the recoil from the slight absorption in the air. But you will also heat the air so affected, and I would not be at all surprised if the resulting convection entirely swamped the effect of radiation pressure on the air. Does that help? --Tardis 04:27, 18 October 2006 (UTC)[reply]
Ayup. Compare Crookes radiometer (thermal effects) to Nichols radiometer and solar sail (radiation pressure). DMacks 04:36, 18 October 2006 (UTC)[reply]

So of this is possible then a EM caused overpressure shockwave would be possible as well if the air could be accelerated to supersonic speeds right? Robin

protozoa - animal report

Helping with a forth grade report and have had problems finding the run donw from kigdom to species. Kingdom, phylum, class, order, family, genus, species, common name? I would appreciate any help.

Our articles on binomial nomenclature, taxonomy, and Linnaean taxonomy ought to get you started. TenOfAllTrades(talk) 23:08, 17 October 2006 (UTC)[reply]
Did you read our protozoa article ? StuRat 23:46, 17 October 2006 (UTC)[reply]

Equine Medicine

I own a 5 year old thoroughbred mare who is presently in training for horse racing. At home she behaves well and even though she had an older mare in the next paddock does not come into season. However as soon she is placed in a professional training environment with other horses, particularly indoors, she immediately comes into season and begins behaving very badly. We have tried deprovera injections without success. I have heard that implanting a glass marble in the uterus can help. Does anyone out there have any knowledge of this problem and possible remedies?

Your help would be appreciated.

Regards

Tony

Maybe equinechronicle can give you some more information on treatment in general. It says the mentioned marble technique lacks references in scientific literature, but Equisearch quotes a study conducted at Auburn University with partially positive results for the method of placing a glass ball in the mare's uterus. Though this technique sounds invasive, no damage was found in the mares' uteral tissue upon removing the glass marble after three months.---Sluzzelin 01:25, 18 October 2006 (UTC)[reply]

Many thanks. This is most helpful. Mumzwon 03:05, 18 October 2006 (UTC)[reply]

Sorry about the apparent name change. I am a first time user and just learnt how to sign my user name (which, by the way, is the racing name of the horse). Regards Tony. Mumzwon 03:09, 18 October 2006 (UTC)[reply]

Please note that people answering medical, legal, or veterinary questions on Wikipedia may have no qualifications whatsoever, and you are always advised to consult an appropriate expert in the field in question. Good luck with your mare. Edison 05:47, 18 October 2006 (UTC)[reply]

Thinking

Is it harder to think straight/remember things when you have a cold?--Light current 23:39, 17 October 2006 (UTC)[reply]

When I have a head cold I just have problems staying awake and problems with external stimuli. That could be a problem. Lots of orange juice for you Mr. Light Current. AMP'd 00:02, 18 October 2006 (UTC)[reply]
Maybe not so much for the reason that you have a cold but more consequential of symptoms such as fatigue. IMO, it's a bit hard to concentrate on a test if you're all about your runny nose, your sneezing, your coughing, etc and unable to focus completely on the task at hand. Thinking straight - might be difficult with lack of focus. Remembering things - it's not in my common knowledge that physical symptoms (aside from distraction) would change the way you retrieve information from the brain, but I'm no expert. Sybil Gray 00:03, 18 October 2006 (UTC)[reply]
On the Red Square (Krasnaya Plochad) , James sneezed. He extended his hand towards the blonde russian spy's and began : - My name is ..." -- DLL .. T 19:01, 18 October 2006 (UTC)[reply]

October 18

How many seagulls...

...would I need to get to stand very close together in order to create sufficiant mass to form a black hole? --Kurt Shaped Box 00:36, 18 October 2006 (UTC)[reply]

Posting on behalf of User:Ziyi_cai841117: let me count...one, two three, four, five...two hundred billion one thousand and one...two hundred billion one thousand and two...four hundred billion...four hundred trillion...four billion billion billion... --Bowlhover 03:28, 18 October 2006 (UTC)[reply]
How heavy is a seagull? 500 grams? If so, you'd need about 1.2 x 10^31 seagulls (see Tolman-Oppenheimer-Volkoff limit). By the way, why are satirical questions about seagulls often asked? How did this joke originate? --Bowlhover 00:57, 18 October 2006 (UTC)[reply]
Your average gull weighs about a kilo - the biggest gulls weigh up to 3kg. As for the questions about gulls - I think I started that one off. I have an interest in gulls and started asking a few questions about them. A few people saw this and started taking the mickey... :) --Kurt Shaped Box 01:01, 18 October 2006 (UTC)[reply]
Ah I remember that --frothT C 16:34, 18 October 2006 (UTC)[reply]
But would you have to compress them as well? There are plenty of stars more massive than 6 x 10^30 kg (~3 solar masses) which are not black holes. There are stars around 20 times as massive. How about an upper bound for the problem. A rough back of the envelope calculation shows that a sphere of radius ~ 2.68 AU at the same density of water would be roughly the same size as its Schwarzschild Radius. That's a mass of about 2.7×1038 kg.Richard B 22:17, 18 October 2006 (UTC)[reply]
I think you could make a black hole with just a few. But to be on the safe side, I would gather the complete worlds population, and pack them together rather tightly. THen you need to surround the gull mass with a very powerful exploding (or imploding) mechanism. Say an H bomb. Under the massive compressive forces, its just possible you could get a very small BH. On the positive side tho, that would be the end of ANY future gull questions on WP 8-)).--Light current 01:28, 18 October 2006 (UTC)[reply]
A H bomb is not at all powerful enough. You'd just be creating a very big mess. In order to create a black hole, you'd have to overcome the strong nuclear force, which is very strong. --Bowlhover 02:09, 18 October 2006 (UTC)[reply]
You're not related to L. Ron Hubbard by any chance, are you? ;) --Kurt Shaped Box 02:04, 18 October 2006 (UTC)[reply]
You gather a bunch of gulls together. Then surround them with a flock of swallows (African or European - your choice) and get them to throw the coconuts they're carrying at them at close to the speed of light. Voila! Clarityfiend 02:40, 18 October 2006 (UTC)[reply]
Interesting question. I've often likened the "big crunch" scenario to the mannerisms of a large group of seagulls converging wildly on a stray piece of bread on the ground.  freshofftheufoΓΛĿЌ  05:07, 19 October 2006 (UTC)[reply]

What kind of bird is this?

This bird flew out of a bush in the middle of Manhattan. What is it? grendel|khan 00:46, 18 October 2006 (UTC)[reply]

It's a common starling. In my homeland we have many. --Kurt Shaped Box 00:50, 18 October 2006 (UTC)[reply]
Blame Eugene Schieffelin. MeltBanana 02:38, 18 October 2006 (UTC)[reply]
You can send some of yours back over here if you like - they've started to decline in the UK. Also, has anyone ever considered trapping for the pet trade? Check out those talking starling videos on YouTube... ;) --Kurt Shaped Box 05:55, 18 October 2006 (UTC)[reply]
Nice photograph. —Bromskloss 09:15, 18 October 2006 (UTC)[reply]

Time machine

How can two teenagers build a time machine? --Bowlhover 03:24, 18 October 2006 (UTC)[reply]

You'll need some paper clips, a piece of string, and tachyon-enriched uranium. —Centrxtalk • 03:29, 18 October 2006 (UTC)[reply]
It's easier when you find a guy named Rufus who will lend it to you, preferably disguised as a phone booth. ---Sluzzelin 03:43, 18 October 2006 (UTC)[reply]
It's really easy [34]. Clarityfiend 04:37, 18 October 2006 (UTC)[reply]
A police box would do nicely as well. – ClockworkSoul 04:38, 18 October 2006 (UTC)[reply]
Eat some mushrooms and all will become clear. Vespine 05:01, 18 October 2006 (UTC)[reply]
Forget the paper clips and uranium. You can find out all about time machines here. B00P 05:46, 18 October 2006 (UTC)[reply]
Or, alternatively, you could buy the plans on the Internet. Just be careful not to injure your vital organs in the process :) --Robert Merkel 07:07, 18 October 2006 (UTC)[reply]
It's quite easy ready... 1. Get a cardboard box, old circuit boards, egg time and some glowing fuel (tip: Paint rock with glow-in-the-dark paint). 2. Set up box and glue on circuit boards and 'fuel' 3. get inside cardboard box and set egg timer for the amount of time you wish to travel into the future. 4. Wait for egg timer to countdown, and only then exit your time travel box. 5. You've now sucessfully travelled the set time into the future. Benbread 08:30, 18 October 2006 (UTC)[reply]
Don't forget to write "TIM MASHIN" on the side! Confusing Manifestation 12:04, 18 October 2006 (UTC)[reply]
How about deciding that when you build the machine you will come back to today from the future and give yourselves the plans. If you don't want to risk meeting yourselves then you can work out where you can leave the plans so that you will find them. Write it on your calendar now so that you won't forget ! Now go and look for those plans ! Gandalf61 08:35, 18 October 2006 (UTC)[reply]
Make sure you bring it to the 24 hour all you can eat car wash. --Russoc4 18:13, 18 October 2006 (UTC)[reply]
Ask yourself. The yourself that already built it and went back into the past to tell you about it, that is.  freshofftheufoΓΛĿЌ  05:02, 19 October 2006 (UTC)[reply]

dirigibles

designs for commercial use; design principles; performance and capacity. Thanks. SKalmutz--Skalmutz 03:52, 18 October 2006 (UTC)Bold text[reply]

See WP's article on airships, and also the external links mentioned there. Have a look at the P-791 for a recent example of design.---Sluzzelin 04:18, 18 October 2006 (UTC)[reply]

Regarding Earth rotation and revolution

If gravity is a force towards centre of sun, why doesn't the earth fall straight into sun instead of rotating around its own axis and revolving around sun? One explaination given by stephen hawkins in brief history of time says that actually earth is travelling strainght in space-time but in 3 dimensional space it looks curved as a result of removal of time dimension.Does somebody have any simpler answer to understand?

If the earth were to stop moving through space for an instant, it would indeed start moving in a straight line directly towards the sun. And if the sun's gravity stopped for an instant, the earth would continue moving in a straight line instead of orbitting. But you know that neither of those "if"s happen...the earth is in orbit, always tending to move "sideways" relative to "directly towards the sun" and the sun's gravity keeps pulling it closer. These effects are balanced--the gravitational pull is strong enough to keep the earth curving around instead of zooming off but not strong enough to overcome the earth's inertia and pull it any closer. See centripetal force and planetary orbit, and remember that actually the earth and sun orbit each other (but the sun is so much more massive, that result is as if just the earth is orbitting a virtually stationary sun). DMacks 09:05, 18 October 2006 (UTC)[reply]
(edit conflict) Hi! Please sign your comments with "~~~~" at the end. We can forget about the earth rotating around its own axis since that is not related (or its impact is at least very small). An object doesn't have to move in the direction of the force that is acting on it if the object is already moving in another direction. Think of a car that rolls past you. If you give it a nudge, it doesn't immediately change its movement and start moving in the direction you pushed, right? If you continue to push it in the same direction for a long time, however, with nothing holding against, it would more an more in that direction. But the sun does actually not pull the earth in the same direction all the time because toward the sun is not the same direction all the time – it depends on where the earth is at the moment. A more familiar example is when you have an object in a string that you swing around in a circle. Let's say you do it in empty space so we don't have to think about air resistance. The only force on the object is the string pulling it and that force works toward the center, where your hand is. Still, the object don't end up in your hand, but continues in a circle. —Bromskloss 09:08, 18 October 2006 (UTC)[reply]
Just to expand on things a little, there are two answers to your question. The first is based on Newtonian gravitation - that gravity is a force, which pulls the Earth to the Sun. But, as pointed out above, the Earth is travelling in another direction, so the force acts to change that direction. Imagine you've got a big cannon on top of a big hill. If you fire the cannon with a small charge, the cannonball will go forwards a bit and fall at the same time, making a parabolic path. If you use a lot more charge, it will go further and further. If, theoretically speaking, you used a massive amount of charge, you could shoot it so fast that by the time it's fallen a certain distance, the curvature of the Earth means it's still at roughly the same height. As long as it keeps falling down as fast as it's moving forwards, it will track out a circular path around the Earth. This is a (very much) simplified way of making something orbit the Earth, like a satellite. Now, imagine that on a bigger scale, and you have the Earth orbiting the Sun.
Unfortunately, it turns out Newton didn't get things perfectly right, and when Einstein came along and invented General Relativity he made the universe look a whole lot different. In particular, in GR gravity isn't a "force", it's a distortion in the shape of spacetime. Think about standing on top of a sphere, like the Earth. From our scale, it looks like the Earth is flat. But if you tried walking straight forwards, eventually you'd come back to where you started, because in fact the Earth is curved. Now imagine space is like that - curved, but in more complicated ways. Anything moving in space that doesn't have a force acting on it (and remember, gravity isn't a force any more) will move forwards in its little bit of spacetime, which will always look flat on a small scale (and in the universe, the scales get pretty big), but whenever you have something massive, like a star, that makes the overall shape of spacetime curved, so even though you may think you're going straight forward, someone standing far away will watch you trace out a curved path - in particular, spacetime around the Sun is shaped so that the path the planets make is roughly circular - i.e. an orbit. Confusing Manifestation 12:03, 18 October 2006 (UTC)[reply]

sinisoidal RC phase shift oscillators

how does a sinusoidal RC oscillator prodduce a sinusoidal output??? I know that an oscillator produces a out put signal even withought a input signal bcause it amplifies the noise signals but noise signals are not always sinosoidal are they?

An oscillator consists of gain more than unity to compensate for losses.So the noise automatically gets amplified,and due to resonant nature of circuit only a particular frequency gets amplified causing suistained oscillations.
The RC produces 60 deg phase shift, so Supposing the output of the amp is 180 out of phase, three such RC networks would eventually produce zero phase shift or in phase oscillations for feedback,thus positive feedback in short.
See Wien bridge oscillator for one answer. Another is the "Twin-T" oscillator, described briefly at RC oscillator.
Atlant 17:03, 18 October 2006 (UTC)[reply]

How can I study theoretical physics in one of the US universities?

I want to study theoretical physics (Quantum mechanics) in one of the US universities. I am nineteen years old. I live in Egypt. What shall I do? How much will this cost me? Is there a way I can get a free scholarship? I have an account under the name Meno25 talk. I sent this message from a public computer because my computer is currently broken down. By the way, I am currently having a wikiholiday.

--196.218.105.36 11:32, 18 October 2006 (UTC)[reply]

First of all, you cannot study just theoretical physics. The canon of undergraduate physics is more or less the same everywhere, and usually contains, within in the first three years (in many conuntries, that is up to the Bachelor of Science degree) courses in: calculus, linear algebra, general physics, mechanics, electrodynamics, lab practices, a bit of quantum mechanics and relativity. If you are then thrilled by it, you'd contnue your studies up to an Honours degree, a Master of Science, or even a PhD, depending on the country and your stamina. But I'm afraid I am not an expert for your main question, because I studied physics in Germany and currently work as physicist in Austria, so I do not know much about scholarships in the USA. But I could tell you about Germany or Austrai, of course, in case you are interested. Simon A. 11:56, 18 October 2006 (UTC)[reply]

Here are some types of scholarships:

  • Academic scholarships for exceptional students. If you are at the top of your class and score extremely well on tests, you might be able to get one of these.
  • Athletic scholarships for excellent athletes. Somehow I doubt if many physics majors have athletic scholarships, but there may be a few.
  • Need-based scholarships. These provide money to those who are poor. Generally speaking, they are only for those of the same nationality as the scholarship fund.
  • Special group scholarships. You get these by being a member of some class (blind, deaf, black, etc.) or organization.

As for the cost, that will vary widely. However, I would say US$10,000-$20,000/year would be a bare minimum for tuition and expenses. Some are far more expensive. One option is to take the first two years at a community college, which can be much less expensive.

As for getting into the US, you would apply for a student visa, at the American embassy. So long as nobody in your family is a member of a terrorist organization like the Muslim Brotherhood, you can probably get one. However, if you ever overstay your student visa, you are likely to be deported and banned from ever entering the US again.

StuRat 14:02, 18 October 2006 (UTC)[reply]

I don't know about the US, but in the UK, plenty of universities, including some prestigious ones such as Imperial College London and University of St Andrews, offer theoretical physics as a possible course[35]. It tends to overlap normal physics but with some modules changed (more maths, less lab work). It's probably too late to apply to a UK university this year (the 2007 UCAS application season is now in full swing), but registration for 2008 should start up soon, and with physics application rates so low, universities will be competing to accept you! As far as scholarships and bursaries are concerned, unfortunately UK universities tend to ask that overseas students sustain themselves (for more info, see International Students on the UCAS website). Laïka 15:27, 18 October 2006 (UTC)[reply]

Well I'm paying 28000 USD per year here in america. I was a geek child so no scholarships for me I'm afraid :) If you do decide to study in america you'll be paying it off for many many years. IIRC britain has free higher education (or was that canada?) but you might have to be a citizen. --frothT C 16:24, 18 October 2006 (UTC)[reply]

Canada does not have free higher education (it has severely reduced higher education but international students have to pay twice the amount domestic students do. However becoming a citizen is easier in Canada.) Sweden and other socialist countries do have free higher education but I forget the requirements. ColourBurst 16:43, 18 October 2006 (UTC)[reply]
The UK did have free Uni education. The universities can now charge up to £3000 (~$5000) per year in Top-up fees, but that still sounds a heck of a lot cheaper than a lot of countries. But an international student would have to pay full fees (Cambridge, Leeds and Imperial all charge around the £10,000 (~$18,000) mark for scientific courses to international students). Laïka 19:02, 18 October 2006 (UTC)[reply]
Doesn't cambridge have like individual dorms and fireplaces and leather furniture in the rooms? At 18 grand? Geez what am I doing in the US? --frothT C 19:16, 18 October 2006 (UTC)[reply]
Some of the rooms are pretty sweet; the best room at Corpus Christi College, Cambridge has ensuite and a private living room! Of course, you need to get at least a first to even be entered into the lottery to earn the room... Laïka 20:39, 18 October 2006 (UTC)[reply]
As you asked on my Talk page, some infor on German universities: University education used to be free, but this year, most Länder have introduced fees. Their amount is not too clear yet but it should settle below €1000 per year. But note that, as you are not a EU citizen you will AFAIK be required to prove that you own enough money to afford living in Germany in order to get a student visa. Maybe I point out a few differences between German and US universities. Many other countries in northern continental Europe (Austria, Switzerland, Benelux, Scandinavia, ...) are comparable to what I say about Germany.
  • German universities have no entrance exams, and besides the most popular subjects (which does not include physics) anybody with an Abitur may start studying at any university. Foreigners have to show that they finished secondary education (i.e. high school) at a level comparable to a German Gymnasium.
  • To compensate for the lack of entrance exams, in physics, the exams after the first and second year are designed to sieve, i.e. to let only those pass who are good enough to study physics. More than a third of the freshmen leave physics after failing these exams!
  • In US universities most undergrads start with one or two terms of general studies before deciding on a major. German universities require you to decide for your major at the beginning. Also, the starting level is higher due to the longer high school time and our three-tier school system.
  • While the US has a few elite universities and many medicore ones, the quality of education differs not that much from one German university to another one, and lies below the top US universities but clearly above the average US university. Still, the physics departments of some German universities are much better than average.
  • Undergrad courses are taught in German! Most foreigners need a year of full time language courses before thay can really start to study. But as every German physicist is usually more or less fluent in English, some universities are considering to offer undergard courses in English. I don't know whether this has already left planning stage.
  • German freshmen are older than US freshmen and treated more as adults. You are expected to organize your live and your studies yourself. While there is counseling offers you have to actively seek them -- nobody will notice if you are left behind. And few universities have a campus with on-campus housing,
  • Traditinally, physics studies take five to six years and finish with the Diplom degree, which is comparable to the Master of Science of British or US universities. However, due to the Bologna process, this is now being changed to a two stage scheme: three years to a Bachelor of Sciene, than two years to a Master of Science degree. The PhD is considered not as a part of the studies but as the first employment of a physiscist at university.
If you have more questions or need pointers to information, just ask. And have a look at the web site of the German Academic Exchange Service (DAAD), which represents German universities abroad: [36] Simon A. 21:19, 18 October 2006 (UTC)[reply]

In the U.S. top private colleges it will cost $40,000 to $45,000 per year for tuition, housing, food, books, fees and other expenses, not including airfare. There may be restrictions on you working if you have a student visa, but perhaps some part time work is allowed. You might check on that. Many foreign governments have sponsored their top students or those related to the people in power to study in the U.S. Some foreign students get the general education courses at a cheap community college, whole they also polish their English. There are tests of English proficiency (TOELF) required by many colleges. Check with your government for possible sponsorship. Good luck! Edison 23:24, 18 October 2006 (UTC)[reply]

Science, I think

I am a Marine currently in Iraq. I found a Phillips PYE Unicam PU 4023 Refractive Index Detector, a PYE Unicam LCXP gradient Programmer, and a Hitachi 150-20 Data Processor. I have researched the 3 items and found that they are most likely used together. What I am trying to figure out is what would the owner use the items for.

Sounds like HPLC. DMacks 15:36, 18 October 2006 (UTC)[reply]
Kill you and your mates most likely if he had a chance. DirkvdM 18:32, 18 October 2006 (UTC)[reply]

Are planets in other solar systems on a parallel orbital plane to our planets? Also why is it that pictures of multiple deep space objects (like that hubble deep field scan) seem to show everything scattered about helter-skelter? If planets and moons form in the same plane then why don't universe-level objects like galaxies? --frothT C 16:19, 18 October 2006 (UTC)[reply]

1. No.
2. Because they are scattered about.
3. Your question betrays a misunderstanding. While the planets of the Solar System revolve around the Sun in (pretty much) the same plane, their axes of rotation are all different (Earth's being tilted by 23½°). So why shouldn't galaxies be oriented in any direction?
B00P 07:04, 19 October 2006 (UTC)[reply]


Formation and evolution of the Solar System should clear this up. Essentially, the process that forced everything onto the same plane at the formation of the Solar System was a completely local process, neither affected by nor affecting other star systems, and not controlled by any larger order. In the absence of a reason for things to line up, they don't. Melchoir 16:49, 18 October 2006 (UTC)[reply]
I'm not sure, but I assume it's because the planets' rotation around the Sun and the moons' rotation around the planets forming at the same time, in the same protoplanetary disk, caused them to be in the same plane. Why that disk is not in the same plane as the milky way, I don't know, but what I said suggests that the two disks did not form at the same time. Just the other night I saw in a documentary that the Sun bobs up and down through the Milky Way's disk, almost as if it rotates around something else in the disk (at least, that's what the visualisation loked like). As for larger structures, it's still a mystery why they formed in the first place. What caused structure in the Universe after the Big Bang that resulted in the accretion that resulted in galaxies and such. Sorry to have asked more questions than given answers. :) DirkvdM 18:50, 18 October 2006 (UTC)[reply]

It has been noted that the arrangement of galaxies resembles the arrangement of a pile of bubbles, with clumps of galaxies where the "bubbles" come together, more sparsely placed galaxies along the "skins" of the bubbles, and inter-galactic voids in the center of the bubbles. To me, this implies that something at the center of the bubbles repels normal matter. This would also help to explain why the expansion rate of the universe is accelerating. StuRat 19:09, 18 October 2006 (UTC)[reply]

Or that there's something near the skins that pulls other bubbles to it, like the Great Attractor or something.. hmmm --frothT C 19:18, 18 October 2006 (UTC)[reply]
But that wouldn't explain why the expansion rate of the universe is accelerating. StuRat 20:33, 18 October 2006 (UTC)[reply]
I think the OP has asked an interesting question. Planetary systems and spiral galaxies (the most common type of observed galaxy) exhibit preferred orbital planes, due to the interaction between conservation of angular momentum and gravtitational attraction. Are there any signs of preferred orbital planes in larger scale structures, such as groups, clusters and superclusters ? If not, why not ? Gandalf61 08:34, 19 October 2006 (UTC)[reply]

Flu vaccine

The university I work for is providing flu vaccines for all employees, with no foreseen shortages. Should I as a healthy, fit, not-very-flu-susceptible, 24-year-old male get the vaccine?

My concerns are:

  1. People in more at-risk groups ought to go ahead of me, even though the uni says they have enough (if they have spare, would the spares go back to the hospital?)
  2. If I'm not at risk, I don't want to put unnecessary medicine in my body
  3. I believe in letting my body's own immune system do the work, and hopefully become stronger.

So obviously I'm leaning towards not getting one (I've never gotten one before), but I am wondering if

  1. My beliefs are mistaken or out-dated
  2. I am selfishly allowing flu to spread by not vaccinating myself.

What do people think?

Thanks! -Sam

You don't seem to be a member of any priority group defined by the ACIP. So basically do as you wish. Don't do so thinking that getting the flu will make your immune system "stronger", though, that's just silly. And yes, by not getting the vaccine you are making it ever-so-slightly more likely for others, especially those you live with, to get flu, but it's a marginal difference. - Nunh-huh 17:39, 18 October 2006 (UTC)[reply]
The defined priority groups are:
People at high risk for complications from the flu, including:
Children aged 6 months until their 5th birthday,
Pregnant women,
People 50 years of age and older, and
People of any age with certain chronic medical conditions;
People who live in nursing homes and other long term care facilities.
People who live with or care for those at high risk for complications from flu, including:
Household contacts of persons at high risk for complications from the flu (see above)
Household contacts and out of home caregivers of children less than 6 months of age (these children are too young to be vaccinated)
Healthcare workers. - Nunh-huh 17:39, 18 October 2006 (UTC)[reply]
Right: my question was based on the fact that the university said that they would have no shortages, and so wondered whether non-priority groups ought to also get the vaccine. --Sam
Yes. You should if you don't want to get the flu, with the risks that entails, risks which are greater than those caused by the vaccine. If you don't care whether you get the flu, or spread it to others, you needn't get the vaccine. - Nunh-huh 17:48, 18 October 2006 (UTC)[reply]

There is a shortage of flu vaccine for the nation, (the US, but probably in other countries as well), and it is a real problem. Companies and individuals with the necessary funds purchase huge amounts of vaccine and then resell it to anyone who has money, regardless of risk factors, in "flu clinics" all over the place. This year, the entire stock of flu vaccine made by Sanofi Pasteur was purchased within the first 14 hours it was on sale. Out of the many units, only about 10% went to actual medical facilities (I'm sorry I can't give exact numbers here). This year, in the city I live in in the US, we have 7,500 DPH-appropriated units for the entire priority population (about 20,000 individuals). Some will seek immunization outside of DPH-funded clinics, others will get sick, some of them will die... and their care will cost the city much more than a few vials of vaccine. That said, your university has said that they have a surplus, and they probably do. If I were you, I would ask them where the extra units will go if they are not used at the school. If the answer is "the trash" then I would get one. Otherwise, I would probaly not. And if you do happen to get sick, stay home if you can. In any case, I would tell them that it is not cool that they purchased so much vaccine. It is certainly unfortunate, Sam, that more people are not as thoughtful as you when they are asked if they want a flu shot, then we might just have enough for the people that need it.Tuckerekcut 22:18, 18 October 2006 (UTC)[reply]

There is no flu vaccine shortage. 40 million doses have been shipped to date, the most ever at this point in the season. [37] [38]. There may be distribution problems, which will be corrected. But just like eating all the food on your plate won't do anything to alleviate the suffering of children starving in Africa, foregoing a flu shot when it's available won't do anything to make it available to others elsewhere. - Nunh-huh 00:54, 19 October 2006 (UTC)[reply]

I regularly get flu shots and I'd suggest you do the same. People who have to interact with lots of people on a regular basis (like at your uni) have a vested interest in doing everything they can to prevent the flu from climbing aboard their body. On the one hand, you'll be in contact with people from over a wide area in a somewhat cramped environment, making it very very likely that you will come in contact with some bugs. On the other hand, lots of people from all over will be coming in contact with you - while you're shedding viruses, you'll potentially infect a lot of the folks around you. My understanding is that the one good reason not to get the shot is if you're allergic to eggs, which are used to incubate the vaccine. See our article for more details. To answer your specific question, yes, I think your views are a little out-dated, but asking questions is the best way to reverse that! If it's the shot that's really bothering you, there's also a nasal mist, which is apparently more effective. Matt Deres 01:24, 19 October 2006 (UTC)[reply]

There is no flu vaccine shortage this year. I just got the annual brochure from the DOH and it makes it very clear that there is no shortage and that it should be given to everybody who wants it. -THB 02:22, 19 October 2006 (UTC)[reply]

membrane capacitive current?

Hello. I can´t understand exactly what is meant by a capacitive outward current across a cell membrane, apparently in the opposite direction of an ionic inward current. Is it because of the departure of positive ions from the extracellular space into the intracellular space, that one defines the "lack" of positive charges outside the cell as a current "across" the membrane? Or is capacitive current a real, physical current from the intracellular space to the extracellular one? Thank you for any help. Christian, medical student, Denmark

DFT bins

I often read terms like transform bin or DFT bin in papers concerning DFT and other transforms.. Can you explain me the meaning?

I think you want to see discrete fourier transform. --Kainaw (talk) 20:03, 18 October 2006 (UTC)[reply]

I did, but on en.wiiki I found that therm in Goertzel Algorithm only, without definiton

Spacetime

One interesting interpretation I heard about relativity is that as you move faster through space you move slower through time. So photons dont actually move through time at all, and if you are stationary in space you are moving at maximum rate through time. Another way to look at, albeit from my laymans viewpoint, is that you have to apply energy to 'brake' your progress through time. Anyway, does this mean that everything travels at the same rate through spacetime, and if so what is that rate mmatt 20:09, 18 October 2006 (UTC)[reply]

Yes: in a very real sense, everything travels at the speed of light through spacetime. (The proper dimensions of spacetime are somewhat hard to define; it really makes just as much sense to say that time and space are the same thing and call the speed of light 1. This makes sense, because for a stationary object, the only speed is "time-speed", and it seems only fair to call that "one second per second" or just 1.) Of course, the very notion of "speed" when time is relative to the observer is somewhat odd; perhaps you'd like to read about proper time, which is one sort of realization of the "constant speed" idea. --Tardis 20:14, 18 October 2006 (UTC)[reply]
According to Einstein, everything is always going at the speed of light it just happens to be that most of that speed is through the fourth dimension (time). When one travels at significant portions of the speed of light through space (say in a fast rocket ship), it's simply transfering some of this speed away from time to space so that you're still going at the speed of light. No faster, no slower. Ƶ§œš¹ [aɪm ˈfɻɛ̃ⁿdˡi] 21:01, 18 October 2006 (UTC)[reply]

Male marsupial nipples ?

Male placental mammals have nonfunctional nipples, so do male marsupials have them, as well ? In the females, those nipples are inside the pouch. Do males also have a partial pouch which contains the nipples ? StuRat 20:18, 18 October 2006 (UTC)[reply]

Better ask User:JackofOz from marsupial land! 8-)--Light current 02:41, 19 October 2006 (UTC)[reply]
I'm flattered, but I have no idea, sorry. What I do know is that marsupials are also found in New Guinea and some adjacent islands; South America; and North America. JackofOz 07:16, 19 October 2006 (UTC)[reply]
Yeah, but you could go and check. Don't you have koalas hanging around in your back yard? Or are you afraid that groping in their pouches to feel their nipples would be seen as sexual harrassment? DirkvdM 08:32, 19 October 2006 (UTC)[reply]
If nobody can find out, I know some people in the marsupial lab at uni and can ask them. I may see them tomorrow, but otherwise next Wednesday. So remember to check this post then. Thanks. --liquidGhoul 08:14, 19 October 2006 (UTC)[reply]
Darwin had something to say about this:
[...] long after the progenitors of the whole mammalian class had ceased to be androgynous, both sexes yielded milk, and thus nourished their young; and in the case of marsupials, that both sexes carried their young in marsupial sacks.
Everything I can find on the internets tell me that male marsupials should have nips too, but I can't find anyone saying "YEP, IT'S TRUE, MR. WALLABY HAS LITTLE BOOBS JUST LIKE YOU AND ME", so I guess you're going to have to take this one on faith.  freshofftheufoΓΛĿЌ  09:59, 19 October 2006 (UTC)[reply]

electromagnetic interaction and contact for force fields?

I have read recently in a book called "a brief history of pretty much everything" it calimed that all physical interactions are actually electrons repeling each other, would it be at all possible at least in theory if not in practice at this stage in technology to create a field or beam of electrons with the same properties as matter to move or repel matter like a force field? is this why we cant make one coz we cant make a powerfull enough or dense enough electron cloud? hopefull

I think you would be more likely to melt it than move it.mmatt 21:00, 18 October 2006 (UTC)[reply]
In the absense of atomic nucleii, the electrons tend to fly apart from one another. They can be kept together temporarily, as in a television with a cathod ray tube. --Gerry Ashton 21:04, 18 October 2006 (UTC)[reply]

but if you could make them group them together in the right way they would form a solid force, like a stream or solid force? hopefull

cathode ray tube. Xcomradex 21:16, 18 October 2006 (UTC)[reply]

That didnt answer my question about if electrons could be arranged to form a force that is solid like normal things are if electron repulsion causes things to be solid? Hopefull

The usual way that electrons are "arranged to form a force that is solid like normal things are" is to, well, use normal things, like, say, a forklift. If you could somehow arrange electrons to make a smooth surface, you could perhaps push on objects with it. But the objects would push back, so you'd have to hold the electrons in their formation somehow. What better way to do that than to stick a bunch of nuclei in with them (to bind them to a spot) and let the resulting atoms bond together to hold a shape? (This, of course, is once again "use normal matter".) It is not unreasonable to charge a piece of metal and then manipulate it with an electric field, or run a current through it and then manipulate it with a magnetic field. The latter is more common since you don't need to keep anything charged, and is the principle behind all sorts of things like rail guns and magnetic levitation. Does that help? --Tardis 22:22, 18 October 2006 (UTC)[reply]

I think so , so the answer is "yes you could make a solid electron field if you could keep them charged and stationary" this would move stuff if it touched it, or stop things moving towards it? is that much correct? would it be possible to make a stronger repulsion than normal, so instead of just pushing it repulses away, like magnets? sorry to ask another question on top of that Hopefull

Domestic Poultry

Why do they loose lose their pigments in captivity and turn white? (ie: chickens, turkeys, ducks, flamingos, etc). --Russoc4 22:39, 18 October 2006 (UTC)[reply]

Their loose pigments fall off ? :-) StuRat 23:36, 18 October 2006 (UTC)[reply]
Ah, you got me. --Russoc4 23:41, 18 October 2006 (UTC)[reply]
I think there are multiple answers. Flamingos lose their colour because they get their colour from the food they eat; when they eat different micro-organisms, they lose their pink-ness. ISTR that zoos and other places actually introduce a reddish dye to their food so that they look 'right'. For the others, I'm not sure if there's an actual trend to whiteness, if it's just happenstance that white happened to be bred in barnfowl, or if some of the same answer applies as with flamingos. Matt Deres 01:32, 19 October 2006 (UTC)[reply]

Flamingos need carotenoids in their diet to develop their pink/red color. This is mentioned in the Flamingo article under "color". It is possible that there are similar effects for other birds. Though, I would consider this more likely to be an effect of breeding. I.e. I've seen brown chicken in captivity. There are different chicken types/races. Lukas 01:47, 19 October 2006 (UTC)[reply]

Perhaps white poultry has been intentionally bred because they are easier to find if they get loose. StuRat 01:52, 19 October 2006 (UTC)[reply]
Right. Anyway, the question is still unanswered. Also, is there a reason why lab rats are white? --Russoc4 02:11, 19 October 2006 (UTC)[reply]
I also don't think they 'lose' their pigment, I think they've been bread that way. Maybe because brown chickens camouflage better, so are harder to find if they escape the coop, as opposed to white birds which would stand out more. Also, white feathers are more aesthetically pleasing to use as stuffing for pillows and duvets. There are probably more reasons. Vespine 02:14, 19 October 2006 (UTC)[reply]
Is the poultry bred to be breaded poultry ? :-) StuRat 02:25, 19 October 2006 (UTC)[reply]
Follow on: As for lab rats my guess is to tell them apart from wild rats? Vespine 02:15, 19 October 2006 (UTC)[reply]
So...they're white "bread". So I guess the brown ones are whole wheat? Clarityfiend 02:27, 19 October 2006 (UTC)[reply]
haha actually, can I say the pun was intended? No, it was a typo;) read:bred ;) Vespine 02:34, 19 October 2006 (UTC)[reply]
Aren't lab rats albinos? Lab animals are often bred to have specific deficiencies. About poultry, there are plenty non-white domesticated ones and there are white wild birds. But there do seem to be more white domesticated ones, so you still have a point. DirkvdM 08:46, 19 October 2006 (UTC)[reply]

Electromagnetic Pulse

How does an electromagnetic pulse short-circuit electronic devices? Note: I am not planning on building an E-bomb. There is just something about being arrested for terrorism that scares me. THL 23:38, 18 October 2006 (UTC)[reply]

See induction. Basically, the EM field induces currents in the circuitry, which can cause a number of different problems, particularly when components are exposed to a greater-than-intended voltage, reverse-biased voltage, or an SCR is switched on (see latch-up).
Over voltages and excessive currents basically destroy the electronic components and devices.--Light current 02:02, 19 October 2006 (UTC)[reply]

October 19

How to Read the Periodic Table of the Elements

How do you read the Periodic table of the Elements? From a grade 6 learning the table in science class

The periodic table is quite a complex representation of the fundamental building blocks of matter. This is a pretty decent one online. I have no idea what level of knowledge grade 6 kids have on this topic so I'll explain the VERY basic. The 'ATOMIC NUMBER' is the primary determining factor of what an element is. The atomic number is the number of protons within the atomic nucleus of the element's atoms. The periodic table starts with Hydrogen which has 1 proton, then helium which has 2, lithium has 3 etc... The reason why they are not always together, i.e. 4 is not near 5, is because the elements are not only listed in order of atomic number but also they are grouped with elements which share common properties, i.e. 2 Helium, 10 neon, 18 Argon, 36 Krypton are all noble gasses and form the right hand column, 3 lithium, 11 sodium, 19 potassium are alkali metals and form the left hand column... There is plently more information packed into the table, like atomic weight, radius, density, boiling point, which you can learn about more later, but the above should be more then enough to get you started. Vespine 04:38, 19 October 2006 (UTC)[reply]
The simple Wikipedia article on this really fails at being simple and is probably just as difficult for a 6th grader to understand. I'll see if I can fix that up in a little bit.  freshofftheufoΓΛĿЌ  04:46, 19 October 2006 (UTC)[reply]

Producing Manganese (III)

I'm wondering how its possible to prepare a compound with manganese in a 3+ oxidation state. I've read on the internet of 1. either acidifying manganese sulfate in solution with sulfuric acid and then adding permaganate or 2. adding base to a manganese sulfate solution and waiting while air oxides the precipitate formed (or then one can add sulfuric acid to dissolve the precipitate, leaving the ion in solution). I also read that the first method produces a orangey colour solution and the second produces a dark, brown/black solid.
Does anyone know how well these methods work? And how stable would my product be, i.e. could the resultant solution or solid be put into a bottle for use later in other reactions?
Thanks in Advance. and P.S. Just in Case, no this is not a homework question, nor does it particularly, I think, sound like say a textbook question. 72.56.169.205 00:46, 19 October 2006 (UTC)[reply]

i would worry you would get manganese dioxide (a Mn(IV) compound) from permanganate, but it might be controllable i guess. typically Mn(III)-salen complexes are made by bubbling air through a solution of the corresponding Mn(II)-salen complex in ethanol. i imagine this procedure would be broadly applicable (it's also used to prepare some Co(III) complexes in the same way). an example is in J. Org. Chem. (2006) 1449-57; there are many more out there. Xcomradex 01:08, 19 October 2006 (UTC)[reply]
Traditionally unstable oxidation states of any transition metal (like manganese(III)) can be stabilised by precipitation or by complexing. Check the electrode potentials, but I am sure that you will find that, in aqueous solution, manganese(III) is unstable with respect to disproportionation. --G N Frykman 07:47, 19 October 2006 (UTC)[reply]

Time Dilation Confusion

I don't understand what a 1G acceleration means in the time dilation article. Here it is: Indeed, a constant 1G acceleration would permit humans to circumnavigate the known universe (with a radius of some 13.7 billion light years) in one human lifetime.

1 g is the acceleration due to gravity at Earth's surface: 9.8 m/s2. —Keenan Pepper 02:05, 19 October 2006 (UTC)[reply]
I don't really understand either. Let's say you're travelling at 299 792 457 m/s. Since it's impossible to gain more than 1 m/s of speed, what does it mean to accelerate at 1 g? How can you accelerate at a constant 9.8 m/s when doing so would mean travelling faster than light? --Bowlhover 03:20, 19 October 2006 (UTC)[reply]
It sounds to me like a weak statment making a lot of assumptions and ifs, i.e. "if it was somehow possible to accelerate infinitely and if there was enough energy to propel you and if you didn't have to worry about relativistic effects and if you could somehow turn at relatively extreme angles without worrying about dying, then it could be possible".  freshofftheufoΓΛĿЌ  04:43, 19 October 2006 (UTC)[reply]
If you kept relativistic effects then you only need to scoff at the idea of enough energy and the idea of turning at relativistic speeds. But if you accelerated fast enough, you could travel the length of the universe's circumference several times over before you died. Ƶ§œš¹ [aɪm ˈfɻɛ̃ⁿdˡi] 05:45, 19 October 2006 (UTC)[reply]
Bowlhover might think of it in terms of Einstein's thought-experiment of an elevator in an infinitly long shaft. A steady acceleration of 1G is what would make it feel as if the elevator was were not moving and you were on the surface of the Earth.
Now, here's the difference between Newtonian and Eisteinian physics. In Classical physics, a steady acceleration results in a straight-line growth of your velocity; Relativistic physics has it that as you approach the speed of light, a steady acceleration increases your velocity by smaller and smaller increments, and so you never get to go faster than light.
B00P 07:28, 19 October 2006 (UTC)[reply]
Of course, a 1G acceleration would never get you off the ground. Assuming you start from the surface of the Earth. But even if you start from space, you'd have to navigate such that you always follow a line that lies where the gravity of surrounding objects cancels out (sort of follow a gravitational ridge), because else you'd eventually get caught by some star. DirkvdM 08:57, 19 October 2006 (UTC)[reply]

Shape of the Universe...

In a previous post, there was mention of circumnnavigation of the Universe. I suppose the "poster" is assuming the Universe is spherical? Could the Universe in fact be a near-perfect sphere? If so, wouldn`t that make it a CLOSED Universe? I tend to think, that with all the evidence about the Universe so far, that it is in fact an OPEN Universe. Anyone care to comment? Thanks Dave 205.188.116.74 02:44, 19 October 2006 (UTC)[reply]

See [39] and [40], as well as our article on "shape of the universe". Basically, although the CMB data does not rule out the possibility of a near-spherical universe, it does not support it either. The evidence also doesn't strongly support any other model of the Universe. --Bowlhover 03:38, 19 October 2006 (UTC)[reply]
Fairly interesting stuff Bowlhover. I guess we`ll just have to wait for new evidence promised in the next decade. TYVM Dave 205.188.116.74 04:02, 19 October 2006 (UTC)[reply]

PHYSICS,Mechanical Advantage&related topics pertaining to Pulleys

In a single Pulley system where the Pulley with the Load(L) is hung by a string whose one end is tied to a fixed rigid support and the other end is being pulled upward. Now,it is observed that when the Load is pulled vertically up by a distance d,the string is pulled up by the Effort(E) to a distance 2d upward.

Kndly help me understand mathematically how the 2d vertcal shift can be proved as against d vertical shift of the Load L.

Regards,

S Ghosh India

Have you read this: "pulley"? I'm not sure what you mean by mathematically proved? E x 2d = L x d, or 2Ed = Ld therefore E = L/2 perhaps? Vespine 04:45, 19 October 2006 (UTC)[reply]

Lightning Protection

Any idea on how UPS such as APC UPS provide Lightning Protection to PC'S through Electric lines and telephone lines???

Have you read our article on surge protector? --Shantavira 08:24, 19 October 2006 (UTC)[reply]